SlideShare a Scribd company logo
1 of 64
THÁI NGUYÊN, 06/2017
Tải tài liệu tại sividoc.com
Viết đề tài giá sinh viên – ZALO:0973.287.149-TEAMLUANVAN.COM
ĐẠI HOC THÁI NGUYÊN
TRƯ NG ĐẠI HOC KHOA HOC
——————–o0o——————–
MÔNG THANH HANG
PHƯƠNG PHÁP LƯ NG GIÁC
GIẢI PHƯƠNG TRÌNH ĐA THỨC
VÀ M T SO DẠNG TOÁN
LU N VĂN THẠC SỸ TOÁN HOC
Chuyên nghành: PHƯƠNG PHÁP TOÁN SƠ CAP
Mã so 60 46 01 13
THÁI NGUYÊN, 06/2017
Tải tài liệu tại sividoc.com
Viết đề tài giá sinh viên – ZALO:0973.287.149-TEAMLUANVAN.COM
ĐẠI HOC THÁI NGUYÊN
TRƯ NG ĐẠI HOC KHOA HOC
——————–o0o——————–
MÔNG THANH HANG
PHƯƠNG PHÁP LƯ NG GIÁC
GIẢI PHƯƠNG TRÌNH ĐA THỨC
VÀ M T SO DẠNG TOÁN
LU N VĂN THẠC SỸ TOÁN HOC
Chuyên nghành: PHƯƠNG PHÁP TOÁN SƠ CAP
Mã so 60 46 01 13
Ngư i hư ng dȁn khoa hoc
GS. TS. LÊ TH± THANH NHÀN
1
Tải tài liệu tại sividoc.com
Mnc lnc
M đau . . . . . . . . . . . . . . . . . . . . . . . . . . . . . 2
Chương 1. M t so đang thfíc lư ng giác và đang thfíc đại so
sinh b i h thfíc lư ng giác 4
1.1 M®t so tính chat của đa thác lượng giác . . . . . . . . . . 4
1.2 M®t so đong nhat thác dạng đại so - lượng giác . . . . . . . 9
1.3 Đa thác Chebyshev...................................................................... 17
1.3.1 Các định nghĩa................................................................. 17
1.3.2 Tính chat của các đa thác Chebyshev............................. 17
Chương 2. Phương pháp lư ng giác giải phương trình b c ba
và b c bon 20
2.1 Giải phương trình b c ba ..........................................................20
2.1.1 Giải và bi n lu n phương trình b c ba ...........................20
2.1.2 Phương trình b c ba nh n các yeu to trong tam giác
là nghi m........................................................................28
2.2 Giải phương trình b c bon ........................................................32
2.3 M®t so h phương trình đưa ve phương trình b c ba và b c
bon......................................................................................................37
Chương 3. Phương pháp lư ng giác giải phương trình đa thfíc
b c cao 39
3.1 Phương trình đa thác b c cao ...................................................39
3.2 H phương trình đa thác b c cao ..............................................49
Chương 4. M t so dạng toán liên quan 51
4.1 Phép the lượng giác..........................................................................51
4.1.1 Phép the lượng giác trong bat đȁng thác........................ 51
4.1.2 Phép the lượng giác trong dãy so.......................................53
4.2 M®t so dạng toán tà các đe thi Olympic sả dụng phương
pháp lượng giác.................................................................................55
Ket lu n............................................................................. 60
Tài li u tham khảo............................................................ 61
2
Tải tài liệu tại sividoc.com
M đau
Các chuyên đe đa thác và lượng giác và nhǎng van đe liên quan là
m®t phan quan trong của đại so và giải tích toán hoc. Các hoc sinh thường
phải đoi m t với nhieu dạng toán loại khó liên quan đen hai chuyên đe này.
Các dạng toán ve phương trình đa thác luôn luôn xuat hi n trong chương
trình toán tà b c THCS đen THPT...
Trong hau het các kỳ thi hoc sinh giỏi quoc gia, Olympic Toán khu
vực và quoc te, Olympic sinh viên giǎa các trường đại hoc và cao đȁng, các
bài toán liên quan đen đa thác rat hay được đe c p và thu®c loại khó và
rat khó.
Các bài toán ve khảo sát phương trình và bat phương trình đa thác
bang phương pháp lượng giác là m®t dạng chuyên đe chon loc can thiet cho
giáo viên và hoc sinh b c trung hoc phő thông và năm đau b c đại hoc. Sả
dụng lượng giác ta có the thiet l p được nhieu đong nhat thác đại so mới,
đe tà đó cho phép giải các phương trình b c ba, b c bon và m®t so dạng
phương trình đa thác b c cao với h so thực m®t cách trực tiep, không can
vi n trợ đen so phác.
Chính vì v y, và cũng đe đáp áng cho nhu cau giảng dạy và hoc t p,
tác giả chon đe tài lu n văn ve "Phương pháp lượng giác giải phương trình
đa thác và m®t so dạng toán". Đây là chuyên đe có ý nghĩa thực tien trong
công vi c giảng dạy, nó cho ta sự nhìn nh n nhat quán ve các bài toán giải
và bi n lu n phương trình đa thác và các dạng toán liên quan đen bat đȁng
thác và cực trị m®t so lớp đa thác m®t bien.
Cau trúc lu n văn gom 4 chương:
Chương 1. M®t so đȁng thác lượng giác và đȁng thác đại so sinh bởi
h thác lượng giác.
Chương 2. Phương pháp lượng giác giải phương trình b c ba và b c
bon.
Chương 3. Phương pháp lượng giác giải phương trình b c cao.
Chương 4. M®t so dạng toán liên quan.
M®t so dạng ví dụ và bài t p được chon loc là các đe ra của các kỳ thi
3
Tải tài liệu tại sividoc.com
hoc sinh giỏi quoc gia và Olympic quoc te. M®t so các bài toán minh hoạ
khác được trích tà các tài li u tham khảo [1-5].
Tôi xin bày tỏ lòng cảm ơn sâu sac tới GS.TS. Lê Thị Thanh Nhàn
người thay đã trực tiep hướng dan và giúp đơ đe tác giả hoàn thành bản
lu n văn này.
Tôi xin chân thành cảm ơn các thay, cô giáo trong khoa Toán - Tin,
phòng Đào tạo trường Đại hoc Khoa hoc - Đại hoc Thái Nguyên, Trường
THPT Sơn Dương, huy n Sơn Dương, tỉnh Tuyên Quang và bạn bè đong
nghi p đã giúp đơ tạo đieu ki n cho tôi hoàn thành bản lu n văn này.
Thái Nguyên, 01 tháng 05 năm 2017
Mông Thanh Hang
4
Tải tài liệu tại sividoc.com
Σ
∈ { }
∈ { }
≤ { }
Chương 1. M t so đang thfíc lư ng
giác và đang thfíc đại so sinh b i h
thfíc lư ng giác
1.1 M t so tính chat của đa thfíc lư ng giác
Định nghĩa 1.1 (xem [3]). Bieu thác
n
Ln(x) = a0 + (ak cos kx + bk sin kx), (1.1)
k=1
trong đó: a0, ak, bk ∈ R (k ∈ {1, 2, . . . , n}); |an| + |bn| /= 0 (n ∈ N∗
),
được goi là đa thác lượng giác b c n (cap n) với các h so a0, ak, bk (k ∈
{1, 2, . . . , n}).
Định nghĩa 1.2 (xem [3]). Neu trong đa thác (1.1) tat cả các h so bk
(k 1, 2, . . . , n ) đeu bang 0 thì ta có đa thác lượng giác cap n thuan
cos:
Cn(x) = a0 + a1 cos x + a2 cos 2x + · · · + an cos nx (an 0) (1.2)
Neu trong (1.1) tat cả các h so ak (k 1, 2, . . . , n ) đeu bang 0 thì ta có
đa thác lượng giác cap n thuan sin:
Sn(x) = b0 + b1 sin x + b2 sin 2x + · · · + bn sin nx (bn /= 0). (1.3)
Tính chat 1.1. Cho Sn(x) và Sm
∗
(x) là hai đa thác lượng giác. Khi đó:
a) Sn(x) + Sm
∗
(x) là đa thác lượng giác b c k với k max n, m .
b) Sn(x).Sm
∗
(x) là đa thác lượng giác b c n + m.
Tính chat 1.2. Với moi đa thác lượng giác Ln(x) dạng (1.1) luôn ton tại
các đa thác đại so Pn(t) và Qn−1(t) sao cho
Ln(x) = Pn(cos x) + sin xQn−1(cos x).
5
Tải tài liệu tại sividoc.com
Σ
Σ Σ
Tính chat 1.3. Với moi Sn(x) dạng (1.3) luôn luôn ton tại đa thác đại so
Qn−1(t) đe
Ln(x) = b0 + sin x.Qn−1(cos x).
Tính chat 1.4. Với moi đa thác Cn(x) dạng (1.2) ta đeu có
Cn(x) = Pn(cos x),
trong đó Pn(t) là đa thác b c n đoi với t và có h so b c cao nhat là an2n−1
.
Ngược lại, với moi đa thác Pn(t) với h so chính bang 1 thì tà phép đ t
ȁn phụ t = cos x ta đeu bien đői ve được đa thác Cn(x) dạng (1.2) với
an = 21−n
.
Bài toán 1.1. Cho đa thác
k
f(x) = a0 + (aj cos jx + bj sin jx) (k ≥ 1) (1.4)
j=1
và cho so α thoả mãn đieu ki n nα = 2π với n > k. Cháng minh rang
f(x + α) + f(x + 2α) + . . . + f(x + nα) = na0. (1.5)
Lài giai.
Nh n xét rang tő hợp tuyen tính của các đa thác dạng (1.4) cũng là
m®t đa thác có dạng đó. Vì v y không mat tính tőng quát ta chỉ can cháng
minh (1.5) cho trường hợp đa thác dạng f (x) = sin mx và f (x) = cos mx
là đủ. M t khác, ta có
n
cos(α + kβ) = 0,
k=1
n
sin(α + kβ) = 0
k=1
đúng với moi α ∈ R, 0 /= β < 2π và nβ . 2π. Tà đó ta có ngay đȁng thác
(1.5) là đúng.
Bài toán 1.2. Cho đa thác
f(x) = b0 + b1 sin x + b2 sin 2x + · · · + bn sin nx, bn =
/ 0,
thoả mãn đieu ki n
Cháng minh rang
|f(x)| ≤ | sin x|, ∀x ∈ R.
|b1 + 2b2 + 3b3 + · · · + nbn| ≤ 1. (1.6)
6
Tải tài liệu tại sividoc.com
. .. . . .
. ..
= |f′(0)| = . lim
f(x) − f(0)
. ≤ lim .
f(x) − f(0)
.
x→0 sin x
đieu phải cháng minh.
x
. x→0
.
x
.
P = 1 − √
2
− R cos 2 α − β +
4
Lài giai. Ta có
|b1 + 2b2 + 3b3 + · · · + nbn| =
f(x) sin x f(x)
= lim = lim ≤ 1,
Bài toán 1.3. Cho các so thực a, b, A, B sao cho đa thác lượng giác
f(x) = 1 − a cos x − b sin x − A cos 2x − B sin 2x
thoả mãn đieu ki n
f(x) ≥ 0, ∀x ∈ R.
Cháng minh rang
a2
+ b2
≤ 2, A2
+ B2
≤ 1.
Lài giai. Đ t
√
a2 + b2 = r;
√
A2 + B2 = R. Khi đó ton tại α, β đe
Tà đó suy ra
a = r cos α; b = r sin α,
a cos x + b sin x = r cos(x − α),
A = R cos 2β; B = R sin 2β,
A cos 2x + B sin 2x = R cos 2(x − β).
f(x) = 1 − r cos(x − α) − R cos 2(x − β).
Đ t
f(α +
π
4
) = P, f(α −
π
) = Q.
4
Khi đó, ta có các đȁng thác
r π
r
Q = 1 − √
2
− R cos 2 π
α − β −
4
.
Neu r2
> 2 thì 1 − √
r < 0.
x
x→0
x
x→0
2
,
7
Tải tài liệu tại sividoc.com
4
4
≤ ≤
≥ ∀ ∈
−
— −
Σ
i i n n
Trị tuy t đoi của hi u hai góc 2(α − β + π
) và 2(α − β − π
) bang π
4 4
nên các cosin của chúng trái dau. Bởi v y, trong hai bieu thác
R cos 2 α − β +
π
R cos 2 α − β −
π
có m®t bieu thác không âm.
Tà đó dan đen trong hai so P và Q có m®t so âm. V y ít nhat m®t
trong hai giá trị f(α + π
) và f(α − π
) là so âm. Đieu đó là vô lý (do giả
4 4
thiet f(x) 0, x R).
V y r2
2 suy ra a2
+ b2
2.
Tương tự ta có
f(β) = 1 − r cos(β − α) − R cos 0 = 1 − r cos(β − α) − R;
f(β + π) = 1 − r cos(β − α + π) − R.
Neu xảy ra trường hợp R > 1 thì 1 R < 0 và do hi u của 2 góc β α + π
và β α bang π nên l p lu n tương tự như trên ta thu được m®t trong hai
so f(β) và f(β + π) là so âm, vô lý.
V y A2
+ B2
≤ 1, đieu phải cháng minh.
Nh n xét 1.1. Bài toán trên là truờng hợp đ c bi t của định lý ve đa thác
lượng giác nh n giá trị không âm:
Neu đa thác
n
f(x) = 1 + (ak cos kx + bk sin kx)
k=1
không âm với moi x ∈ R thì
a2
+ b2
≤ 2, ∀i ∈ {1, 2, . . . , (n − 1)}, a2
+ b2
≤ 1.
Bài toán 1.4. Cháng minh rang với m = 2n
− 1, đa thác lưong giác
f(x) = cos 2n
x + a1 cos(2n
− 1)x + a2 cos(2n
− 2)x + . . . + am cos x (1.7)
không the chỉ nh n giá trị cùng dau.
Lài giai. Giả sả f(x) chỉ nh n giá trị dương. Khi đó
1
f1(x) := (f(x) + f(x + π)) > 0
2
và
8
Tải tài liệu tại sividoc.com
∈
4
Σ
∈ ∀ ∈
n
với moi x ∈ R.
Do cos(x + kπ) = (−1)k
cos x nên đa thác
f1(x) = cos 2n
x + a2 cos(2n
− 2)x + . . . + am−2 cos 2x
dương với moi x R.
Do đó đa thác
1 1
f2(x) :=
2
(f1(x) + f1(x +
2
π))
dương với moi x ∈ R.
Tương tự như trên ta cũng thu được
f2(x) = cos 2n
x + a4 cos(2n
− 4)x + . . . + am−4 cos 4x.
V y
f(x) =
1
f
2 2(x) + f2 x +
1
π
dương với moi x ∈ R. L p lại quá trình trên, sau hǎu hạn bước ta thu được
đa thác cos 2n
x > 0 với moi x ∈ R. Đieu đó không xảy ra.
Nh n xét 1.2. Neu sả dụng đ c trưng tuan hoàn của các nguyên hàm
F (x) của f(x) dạng (1.7) thì F (x) không the là hàm thực sự đơn đi u và
do đó đạo hàm của nó (chính là f(x)) không the luôn luôn cùng dau.
Bài toán 1.5. Cho đa thác
n
fn(x) = a0 + (ak cos kx + bk sin kx),
k=1
trong đó các so thực a0, ak, bk R thoả mãn đieu ki n fn(x) > 0, x R,
a2
+ b2
= 1 (i = 1, 2, . . . , n).
i i
Cháng minh rang
fn(x) − n
a0
≤ 1, ∀x ∈ R.
Lài giai. Ta có
fn(x) ≤ a0 +
Σ q
a2
+ b2
= a0 + n. (1.8)
Xét nguyên hàm của fn(x)
i i
i=1
F (x) = a0x +
Σ
sin ix − cos ix .
ai bi
i i
i=1
n
9
Tải tài liệu tại sividoc.com
Do F′(x) = fn(x) > 0 với moi x ∈ R nên F(x) là hàm tăng trên R. Suy ra
F(2π) > F(0) tác a0 > 0. Ket hợp với (8) ta thu được
fn(x) − n
a0
≤ 1, ∀x ∈ R.
1.2 M t so đong nhat thfíc dạng đại so - lư ng giác
Nh n xét rang đȁng thác cơ bản đe dan đen sự phong phú của h thong
các đong nhat thác lượng giác là công thác
sin2
t + cos2
t = 1, ∀t ∈ R. (1.9)
Gan với h thác (1.9) là đong nhat thác Lagrange
(2x)2
+ (1 − x2
)2
= (1 + x2
)2
, ∀x ∈ R. (1.10)
Hai công thác (đong nhat thác) (1.9) và (1.10) là hai cách viet của m®t
t
h thác. Neu ta thay x = tan
2
ngược lại.
vào (1.10) thì de dàng thu được (1.9) và
Như v y là moi công thác lượng giác sě tương áng với m®t đong nhat
thác đại so tương áng. Đieu đó cũng th t de hieu neu chúng ta nhớ lại quá
trình dan dat đen định nghĩa các hàm so lượng giác cơ bản đoi với góc nhon
được mô tả dựa theo Định lý Pytago:
Trong tam giác vuông ABC với cạnh huyen BC ta luôn có h thúc
(AB)2
+ (AC)2
= (BC)2
.
Tuy nhiên, với so lượng các công thác bien đői lượng giác quá nhieu,
bản thân các h thác lượng giác tạo thành m®t chuyên đe có tính đ®c l p
tương đoi, dan tách hȁn cơ sở đại so của nó, đã làm cho chúng ta quên đi
m®t lượng lớn các h thác đại so có cùng xuat sá tà m®t h thác lượng giác
quen biet. Đ c bi t, trong chương trình toán b c phő thông hi n nay, các
hàm so lượng giác ngược, hàm lượng giác hyperbolic,... không nam trong
phan kien thác bat bu®c thì nhǎng bài toán liên quan đen chúng sě là m®t
thách thác lớn đoi với hoc sinh.
Ta nhac lại công thác Euler quen biet
eiα
= cos α + i sin α, α ∈ R.
10
Tải tài liệu tại sividoc.com
/∈ −
2 a
2 a2 2 a
2 a
2 a5 2 a 2 a3 2 a2
−
Khi đó
cos α =
sin α =
eiα + e−iα
,
2
eiα e−iα
.
2i
Rõ ràng khi khảo sát hàm so cos t thì ít ai nghĩ trong đau rang nó có dạng
1
a +
1
vì khi đó a không còn là m®t so thực. Nhưng neu ta chú ý đen
bieu thác
eα + e−α
2
, α ∈ R,
thì đó chính là cos(iα) (= cosh α) và vì v y, ve m t hình thác, ta sě có
nhieu bien đői thu được tà các công thác liên quan đen bien x [ 1, 1]
giong như công thác đoi với hàm cos t.
Ví dn 1.1. H thác đại so áng với công thác
cos 2t = 2 cos2
t − 1
chính là công thác
1
a2
+
1
= 2
h1
a +
1 i2
−1.
Ví dn 1.2. H thác đại so áng với công thác
cos 3t = 4 cos3
t − 3 cos t
chính là công thác
1
a3
+
1
= 4
h1
a +
1 i3
−3
h1
a +
1 i
,
2 a3 2 a 2 a
hay
4x3
− 3x =
1
a3
+
1
2 a3
với
x =
1
a +
1
, a /= 0.
Ví dn 1.3. H thác đại so áng với công thác
cos 5t + cos t = 2 cos 3t cos 2t
chính là công thác
1
a5
+
1
+
1
a +
1
= 2
h1
a3
+
1 ih1
a2
+
1 i
.
11
Tải tài liệu tại sividoc.com
2
−
2 a
2 q3
2 q 2
Tà ví dụ trên, sả dụng ket quả khai trien các hàm lượng giác cos 3t và
cos 2t, ta thu được đong nhat thác đại so dạng b c 5.
1
a5
+
1
= −m + 2(4m3
− 3m)(2m2
− 1),
2
trong đó
a5
m =
1
a +
1
.
Ví dn 1.4. Cho so thực m với |m| > 1. Tính giá trị của bieu thác
M = 8x3
− 6x,
trong đó
x =
1
q
3
m +
√
m2 − 1 +
q
3
m −
√
m2 − 1 .
Lài giai. Đe ý rang, do |m| > 1 nên ton tại so thực q đe có h thác
m =
1
q3
+
1
.
Ta chỉ can chon
là đủ. Khi đó
q =
q
3
m +
√
m2 − 1
1
q +
1
=
1
q
3
m +
√
m2 − 1 +
q
3
m −
√
m2 − 1 = x.
Theo Ví dụ 1.2 thì
nên M = 2m.
4x3
− 3x = m
Ví dn 1.5. Không dùng máy tính, tìm giá trị đúng của góc nhon x thoả
mãn
cos x = q
1 + (
√
6 +
1
√
2 −
√
3 − 2)2
Lài giai. Xét
A =
√
6 +
√
2 −
√
3 − 2.
Ta có
A = (
√
3 −
√
2)(
√
2 − 1) =
√
3
√
2
√
2 + 1
,
.
12
Tải tài liệu tại sividoc.com
√
r
— −
hay √
3
√
2 π π
A = 2
−
2
2 1
cos
6
− cos
4
= π π
sin + sin
π
= tan .
24
V y nên
2
+
2 4 6
1 + A2
= 1 + tan2 π
=
1
π .
Suy ra
24 cos2
24
hay
cos x = cos2
π
24
π
= cos ,
24
π
x = ±
24
+ k2π, k ∈ Z.
π
Do x là góc nhon nên x = .
24
Cách 2. Tà h thác đã cho
1
ta thu được
cos x = q
1 + (
√
6 +
√
2 −
√
3 − 2)2
1 + (
√
6 +
√
2
√
3 2)2
=
1
cos2 x
= 1 + tan2
x.
Do đó
tan2
x = (
√
6 +
√
2 −
√
3 − 2)2
,
hay
tan x =
√
6 +
√
2 −
√
3 − 2 > 0, do x là góc nhon.
Tiep theo ta sả dụng h thác góc nhân đôi đoi với hàm so tang ho c hàm
so cosin, ta thu được công thác tính góc nhon x.
Bây giờ ta chuyen sang xét các h thác đại so liên quan đen hàm so
sin t. Tà công thác Euler, ta thu được h thác
eit − e−it
i sin t = .
2
Tà đây suy ra bieu thác i sin(it) nh n giá trị thực. Đieu này gợi ý cho ta
cách chuyen đői các đong nhat thác đoi với hàm so sin sang các đong nhat
thác đại so.
,
13
Tải tài liệu tại sividoc.com
2 a5 2 a 2 a3 2 a2
2 a
Ví dn 1.6. Xét công thác khai trien
sin 3t = 3 sin t − 4 sin3
t.
Tà đây ta thu được công thác (hình thác)
i sin i(3t) = 3(i sin it) + 4(i sin it)3
.
H thác đại so áng với công thác trên chính là đong nhat thác
1
a3
−
1
= 3
h1
a −
1 i
+4
h1
a −
1 i3
,
hay
2 a3 2 a 2 a
4x3
+ 3x =
1
a3
−
1
với
2 a3
x =
1
a −
1
, a 0.
2 a
Ví dn 1.7. Xét công thác bien đői
sin 5t + sin t = 2 sin 3t(1 − 2 sin2
t). (1.11)
Ta viet lại công thác (1.11) dưới dạng
i sin i(5t) + i sin it = 2i sin i(3t)(1 + 2(i sin it)2
.
H thác đại so áng với công thác trên chính là đong nhat thác
1
a5
−
1
+
1
a −
1
= 2
h1
a3
−
1 ih
1 +
1
a −
1 2i
.
Tà ví dụ trên, sả dụng ket quả khai trien các hàm lượng giác sin 3t và
sin 2t, ta thu được đong nhat thác đại so dạng như ở ví dụ sau đây.
Ví dn 1.8.
1
a5
−
1
= −m + 2(4m3
+ 3m)(2m2
+ 1),
2
trong đó
a5
m =
1
a −
1
.
14
Tải tài liệu tại sividoc.com
2
√ √
√ √
√
2 q3
2 q 2
√
(1 + z2)(1 + x2) −
√
1 + z2 −
√
1 + x2
Ví dn 1.9. Cho so thực m. Tính giá trị của bieu thác
M = x3
+
3
x,
4
trong đó
x =
1
q
3
m +
√
m2 + 1 +
q
3
m −
√
m2 + 1 .
Lài giai. Đe ý rang, với moi m đeu ton tại so thực q đe
m =
1
q3
−
1
.
Ta chỉ can chon
là đủ. Khi đó
q =
q
3
m +
√
m2 + 1
1
q −
1
=
1
q
3 m +
√
m2 + 1 +
q
3
m −
√
m2 + 1 = x.
Theo Ví dụ 1.6 thì
1
4x3
+ 3x = m
nên M = m.
4
Tà nhǎng ket quả nh n được, ta có the giải và bi n lu n được nhieu
dạng phương trình đại so b c cao và công thác tính giá trị của m®t so bieu
thác cháa căn thác.
Bài toán 1.6. Cho các so dương x, y, z thỏa mãn đieu ki n x+y +z = xyz.
Cháng minh rang
(1 + y2)(1 + z2) − 1 + y2 −
√
1 + z2
yz
+
zx
(1 + x2)(1 + y2) − 1 + x2 − 1 + y2
xy
= 0.
Lài giai. Đ t
x = tan α, y = tan β, z = tan γ
+
15
Tải tài liệu tại sividoc.com
2
√ √
√ √ √
√
√ √
với
Do x + y + z = xyz, nên
α, β, γ ∈ 0;
π
.
tan α + tan β + tan γ = tan αtan βtan γ
⇔ tan α + tan β = tan γ(tan αtan β − 1)
tan α + tan β
⇔
1 − tan αtan β
= − tan γ (de thay tan α tan β =
/ 1)
⇔ α + β = −γ + kπ ⇔ α + β + γ = kπ, (k ∈ Z)
Do α + β + γ ∈ 0;
3π
, suy ra 0 < kπ <
3π
, mà k ∈ Z nên k = 1.
2 2
V y nên α + β + γ = π. Ta suy ra
(1 + y2)(1 + z2) − 1 + y2 −
√
1 + z2
yz
(1 + tan2
β)(1 + tan2
γ) − 1 + tan2
β − 1 + tan2
γ
tan βtan γ
1 1 1 1
cos β cos γ
−
cos β
−
cos γ
=
sin β sin γ
cos β cos γ
=
1 − (cos β + cos γ)
.
sin β sin γ
Tương tự, ta cũng có
(1 + z2)(1 + x2) −
√
1 + z2 −
√
1 + x2
zx
=
1 − (cos γ + cos α)
sin γ sin α
và
(1 + x2)(1 + y2) −
√
1 + x2 − 1 + y2
xy
=
1 − (cos α + cos β)
.
sin α sin β
Khi đó ve trái của đȁng thác can cháng minh bang
1 − (cos β + cos γ)
+
1 − (cos γ + cos α)
+
1 − (cos α + cos β)
sin β sin γ sin γ sin α sin α sin β
=
sin α + sin β + sin γ − sin(α + γ) − sin(α + β) − sin(β + γ)
sin α sin β sin γ
= 0,
đieu phải cháng minh.
=
16
Tải tài liệu tại sividoc.com
Bài toán 1.7. Cho xy /= −1, yz /= −1, zx /= −1. Cháng minh rang
x − y
1 + xy
+
y − z
1 + yz
+
z − x
1 + zx
=
x − y
1 + xy
.
y − z
1 + yz
.
z − x
.
1 + zx
Lài giai. Đ t x = tan α, y = tan β, z = tan γ, với α, β, γ ∈ −
π
;
π
.
Khi đó
x − y
1 + xy
+
y − z
1 + yz
2 2
+
z − x
1 + zx
=
tan α − tan β
1 + tan αtan β
+
tan β − tan γ
1 + tan βtan γ
+
tan γ − tan α
1 + tan αtan γ
= tan(α − β) + tan(β − γ) + tan(γ − α)
và
x − y y − z z − x
1 + xy
.
1 + yz
.
1 + zx
= tan(α − β) tan(β − γ) tan(γ − α).
Ta cháng minh đong nhat thác
tan(α − β) + tan(β − γ) + tan(γ − α) = tan(α − β) tan(β − γ) tan(γ − α).
Th t v y, ta có
nên
tan a + tan b
1 − tan a tan b
= tan(a + b)
Suy ra
tan a + tan b = tan(a + b)(1 − tan a tan b).
tan(α − β) + tan(β − γ) + tan(γ − α)
= tan(α − β + β − γ)[1 − tan(α − β) tan(β − γ)] + tan(γ − α)
= tan(α − γ) + tan(γ − α) + tan(α − β) tan(β − γ) tan(γ − α)
= tan(α − β) tan(β − γ) tan(γ − α).
Do đó
x − y
1 + xy
+
y − z
1 + yz
+
z − x
1 + zx
=
x − y
1 + xy
.
y − z
1 + yz
.
z − x
,
1 + zx
đieu phải cháng minh.
17
Tải tài liệu tại sividoc.com
∈
∈
−
1.3 Đa thfíc Chebyshev
1.3.1 Các định nghĩa
Định nghĩa 1.3 (xem [3]). Các đa thác Tn(x) (n N) được xác định như
sau
T0(x) = 1; T1(x) = x,
Tn+1(x) = 2xTn(x) − Tn−1(x), ∀n > 1
được goi là các đa thác Chebyshev (loại 1).
Định nghĩa 1.4 (xem [3]). Các đa thác Un(x) (n ∈ N) xác định như sau
U0(x) = 0; U1(x) = 1,
Un+1(x) = 2xUn(x) − Un−1(x), ∀n > 1
được goi là các đa thác Chebyshev (loại 2).
1.3.2 Tính chat của các đa thfíc Chebyshev
Tính chat 1.5. Tn(x) = cos(n arccos x) với moi x ∈ [−1, 1]
Tính chat 1.6. Tn(x) Z[x] b c n có h so b c cao nhat bang 2n−1
và là
hàm chȁn khi n chȁn; là hàm lẻ khi n lẻ.
Tính chat 1.7. Tn(x) có đúng n nghi m phân bi t trên [-1, 1 ] là
2k + 1
xk = cos
2n
π (k = 0, 1, . . . , n − 1).
kπ
Tính chat 1.8. |Tn(x)| ≤ 1, ∀x ∈ [−1, 1] và |Tn(x)| = 1 khi x = cos
n
,
k ∈ Z.
Tính chat 1.9. Đa thác T ∗(x) = 21−n
Tn(x) là đa thác b c n với h so b c
cao nhat bang 1 và có đ® l ch so với 0 trên [ 1, 1] là nhỏ nhat trong tat cả
các đa thác b c n với h so b c cao nhat bang 1.
Tính chat 1.10. Un(x) =
sin(n arccos x)
√
1 − x2
với moi x ∈ (−1, 1).
18
Tải tài liệu tại sividoc.com
−
n
n n
sin t
1 sin nt
Tính chat 1.11. U (x) = T ′ (x) = , cos t = x, đa thác b c n − 1
có h so b c cao nhat bang 2n−1
và là hàm chȁn khi n lẻ; là hàm lẻ khi n
chȁn.
Tính chat 1.12. Tn(x) có đúng n nghi m phân bi t trên [-1, 1 ] là
2k + 1
xk = cos
2n
π (k = 0, 1, . . . , n − 1).
Tính chat 1.13. |Un(x)| ≤ n, ∀x ∈ [−1, 1] và |Tn
′
(x)| ≤ n2
, ∀x ∈ [−1, 1].
Xét các hàm so
sinh x =
1
(ex
e−x
), cosh x =
2
1
(ex
+ e−x
).
2
Khi đó với |x| > 1 thì
Tn(x) = cosh(nt); Un(x) =
trong đó x = cosh t.
sinh(nt)
, (1.12)
sinh t
Bài toán 1.8. Cháng minh rang đa thác Un(x) có đúng n − 1 nghi m thực
khác nhau trong (−1, 1).
Lài giai. Sả dụng Tính chat 1.3 và do Tn(x) có đúng n nghi m thực phân
bi t trên [−1, 1] nên theo Định lý Rolle ta có ngay đieu phải cháng minh.
Bài toán 1.9. Cháng minh rang
Un(x) = xUn−1(x) + Tn−1(x), ∀n ∈ N∗; x ∈ R.
Lài giai. Ta cháng minh cho trường hợp x ∈ (−1, 1). Đ t x = cos t
(0 < t < π) và sả dụng các tính chat của Tn(x) và Un(x) ta thu được đieu
phải cháng minh.
Với |x| > 1 thì ta sả dụng ket quả của đȁng thác (1.12).
Bài toán 1.10. Cháng minh rang
Tn+1(x) = xTn(x) − (1 − x2
)Un(x), ∀x ∈ R, n ∈ N. (1.13)
Lài giai. Sả dụng phương pháp quy nạp, ta sě thu được (1.13)
19
Tải tài liệu tại sividoc.com
| |
∈
Bài toán 1.11. Cháng minh rang
(1 − x2
)Tn
′′
(x) − xTn
′
(x) + n2
Tn(x) = 0, ∀n ∈ N, x ∈ R.
Lài giai. Với x ∈ [−1, 1], ta sả dụng tính chat 1.2 của Tn(x) và thay trực
tiep bieu thác của Tn(x) vào đieu ki n bài ra ta thay đȁng thác đúng.
Với x > 1 thì sả dụng (1.13) và các tính chat của các hàm so sinh x
và cosh x.
Bài toán 1.12. Cháng minh rang với m, n ∈ N; n ≥ m và x ∈ R thì
Tn+m(x) + Tn−m(x) = 2Tn(x)Tm(x).
Lài giai. Sả dụng định nghĩa và phương pháp quy nạp ho c sả dụng các
công thác
cos(n + m)x + cos(n − m)x = 2 cos nx cos mx
và
cosh(n + m)x + cosh(n − m)x = 2 cosh(nx) cosh(mx).
Bài toán 1.13. Cháng minh rang
Tm(Tn(x)) = Tmn(x), ∀x ∈ R, m, n ∈ N. (1.14)
Lài giai. Ta cháng minh (1.14) bang phương pháp quy nạp theo m. Với
n co định tuỳ ý và m = 0, ta có T0(Tn(x)) = 1 = T0n(x) (theo định nghĩa
Tn(x)). V y (1.14) đúng với m = 0; n N.
Giả sả (1.14) đúng tới m. Khi đó
Tm+l(Tn(x)) = 2Tn(x)Tm(Tn(x))
= 2Tn(x)Tmn(x) − T(m−l)n(x) (theo giả thiet quy nạp)
= Tn+mn(x) + Tmn−n(x) − Tmn−n(x) (theo Bài toán1.3 )
= T(m+1)n(x).
V y
Tm(Tn(x)) = Tmn(x), ∀x ∈ R, m, n ∈ N.
20
Tải tài liệu tại sividoc.com
0
√
0 0
∆ = (ax0 + b)2
− 4a(ax2
+ bx0 + c) > 0
0
Chương 2. Phương pháp lư ng giác
giải phương trình b c ba và b c bon
2.1 Giải phương trình b c ba
Trong phan này ta nêu phương pháp giải phương trình b c ba với h
so thực tùy ý :
ax3
+ bx2
+ cx + d = 0, a /= 0. (2.1)
2.1.1 Giải và bi n lu n phương trình b c ba
Bài toán 2.1. Giải phương trình (2.1) khi biet m®t nghi m x = x0.
Lài giai. Theo giả thiet thì ax3
+ bx2
+ cx0 + d = 0
0 0
(2.1) ⇔ ax3
+ bx2
+ cx + d = ax3
+ bx2
+ cx0 + d
0 0
3 3 2 2
⇔ a(x x0) + b(x
2
— x0) + c(x − x0) = 0
⇔ (x − x0)[ax + (ax0 + b)x + ax0 + bx0 + c] = 0.
1) Neu ∆ = (ax0 + b)2
− 4a(ax2
+ bx0 + c) < 0 thì phương trình (2.1)
có nghi m duy nhat x = x0.
2) Neu ∆ ≥ 0 thì phương trình (2.1) có các nghi m
x = x0
x =
−(ax0 + b) ± ∆
2a
H quả 2.1. 1) Neu x0 là nghi m của phương trình (2.1) thì đieu ki n can
và đủ đe phương trình (2.1) có ba nghi m phân bi t là
ax2
+ (ax0 + b)x0 + ax2
+ bx0 + c 0
2
−
21
Tải tài liệu tại sividoc.com
−
0
0
3
c
0
x1 + x2 + x3 = −
b
d
b
3
2
√
2) Neu x0 là nghi m của (2.1) thì có the phân tích :
ax3
+ bx2
+ cx + d = (x − x0)f(x). (2.2)
trong đó f(x) là m®t tam thác b c hai xác định.
3) Neu x1, x2, x3 là các nghi m của phương trình (2.1) thì
ax3
+ bx2
+ cx + d = a(x − x1)(x − x2)(x − x3)
và có công thác Viete
a
c
x1x2 + x2x3 + x3x1 =
a
x1x2x3 =
a
Bài toán 2.2. Giải phương trình
với
ax3
+ bx2
+ cx + d = 0 (2.3)
ac3
= db3
. (2.4)
(Khi đó phương trình (2.3) (2.4) có tên goi là phương trình quy hoi b c
ba)
Lài giai. Tà (2.4), suy ra
1) Neu c = 0 thì b = 0 và
(2.4) ⇔ ax3
+ d = 0 ⇔ x =
r
3
d
−
a
,
d
2) Neu c /= 0 thì b /= 0 và
c
= ( )3
.
c a b
Đ t = −x0 thì c = −bx0, d = −ax3
.
The vào (2.3), ta được
ax3
+ bx2
− bx0x − ax3
= 0
⇔ a(x − x0) + bx(x − x0) = 0
2
⇔ (x − x0)[ax + (ax0 + b)x + ax ] = 0.
V y x = x0 = −
b
là m®t nghi m.
Neu ∆ = (ax0 + b)2
− 4a2
x2
≥ 0 thì phương trình còn có nghi m
0
x =
−(ax0 + b) ± ∆
.
2a
22
Tải tài liệu tại sividoc.com
— ±
−
. .
1
0
. .
1
3
1
3
2
Bài toán 2.3. Giải phương trình: 4x3
3x = m với m = 1.
Lài giai. Khi m = 1 thì phương trình có dạng
4x3
− 3x = 1 ⇔ (x − 1)(2x + 1)2
= 0
1
nên phương trình có nghi m đơn x = 1 và nghi m kép x = −
2
.
Khi m = −1 thì phương trình có dạng
4x3
− 3x = −1 ⇔ (x + 1)(2x − 1)2
= 0
1
nên phương trình có nghi m đơn x = −1 và nghi m kép x =
2
.
Bài toán 2.4. Giải phương trình: 4x3
− 3x = m với |m| < 1.
Lài giai. Đ t m = cos α = cos(α ± 2π). Khi đó
cos α = cos(3.
α
) = 4 cos3
α 3 cos α.
3
Do v y phương trình có ba nghi m
x1
α
= cos 3
x2,3 = cos
α ± 2π
3
Bài toán 2.5. Giải phương trình
4x3
− 3x = m, |m| > 1.
Lài giai. Ta cháng minh phương trình có nghi m duy nhat.
Th t v y, phương trình không có nghi m trong [−1, 1] vì neu
x = x0 ∈ [−1, 1] là nghi m thì ta đ t x0 = cos β.
Khi đó 4x3
− 3x0 = |cos 3β| ≤ 1 /= m.
Giả sả phương trình có nghi m x = x1 với |x1| > 1. Khi đó 4x3
−3x1 =
m. V y có phương trình
4x3
− 3x = 4x3
− 3x1
⇔ 4(x − x1) − 3(x − x1) = 0
2
⇔ (x − x1)(4x + 4x1x + 4x − 3) = 0
∆ = 4x2
− 4(4x2
− 3) = 12 − 12x2
< 0.
1 1 1
V y x = x1 là nghi m duy nhat.
23
Tải tài liệu tại sividoc.com
0
0
2 a3
2 a3
a) Neu p = 0 thì phương trình có nghi m duy nhat y = 3 q.
3
Đ t m =
1
(a3
+
1
) với a3
= m ±
√
m2 − 1.
Khi đó, phương trình có nghi m duy nhat
1 1
x = (a + ) =
2 a
1
q
3
m +
√
m2 − 1 +
q
3
m −
√
m2 − 1 .
Bài toán 2.6. Giải phương trình
4x3
+ 3x = m.
Lài giai. Nh n xét rang neu x = x0 là nghi m của phương trình thì đó là
nghi m duy nhat.
Th t v y, xét x > x0 khi đó
4x3
+ 3x > 4x3
+ 3x0 = m.
Tương tự, với x < x0 thì 4x3
+ 3x < 4x3
+ 3x0 = m.
1 1
Đ t x = (a − ).
2 a
Khi đó de dàng kiem tra đȁng thác
4x3
+ 3x =
1
a3
−
1
. (2.5)
2 a3
Suy ra cách giải phương trình như sau.
Đ t m =
1
(a3
−
1
) với a3
= m ±
√
m2 + 1.
Khi đó theo (2.5) ta có nghi m duy nhat của phương trình :
x = (a − ) =
q
1 1 1 3 m +
√
m2 + 1 +
q
3
m −
√
m2 + 1 .
2 a 2
Bài toán 2.7. Giải và bi n lu n phương trình
t3
+ at2
+ bt + c = 0.
a
Lài giai. Đ t t = y −
3
. Khi đó ta có the viet phương trình dưới dạng
(y −
a
)3
+ a(y −
a
)2
+ b(y −
a
) + c = 0
3 3 3
⇔ y − py = q,
a2
2a3
ab
trong đó p =
3
− b; q = −
27
+
3
− c.
√
2
24
Tải tài liệu tại sividoc.com
± −
3
2 d3
3
b) Neu p > 0. Đ t y = 2
r
p
x. Khi đó ta được phương trình 4x3
−3x =
3
√
3q
m với m =
2p
√
p
.
-) |m| ≤ 1. Đ t m = cos α. Khi đó phương trình có ba nghi m:
x = cos
α
; x = cos
α ± 2π
.
3 3
-) |m| > 1. Đ t m =
1
(d3
+
1
) với d3
= m
√
m2 1.
2 d3
Khi đó phương trình có nghi m duy nhat
1 1
x = (d + ) =
2 d
1
q
3
m +
√
m2 − 1 +
q
3
m −
√
m2 − 1 .
c) Neu p < 0. Đ t y = 2
r
−
p
x ta được phương trình 4x3
+ 3x = m.
Đ t m =
1
(d3
−
1
) với d3
= m ±
√
m2 + 1.
Khi đó phương trình có nghi m duy nhat
x = (d − ) =
q
1 1 1 3 m +
√
m2 + 1 +
q
3
m −
√
m2 + 1 .
2 d 2
Bài toán 2.8. Cho x1, x2, x3 là nghi m của phương trình
x3
+ ax2
+ x + b = 0 (2.6)
và b 0. Cháng minh rang
1
(x1 −
x
1
)(x2 −
x
1
) + (x2 −
x
1
)(x3 −
x
1
) + (x3 −
x
1
)(x1 −
x
) = 4.
Lài giai. Vì x1, x2, x3 là nghi m của phương trình (2.6) nên
x1x2 + x2x3 + x3x1 = 1
hay
x1x2x3 = −b 0
x1x2 + x2x3 + x3x1 = 1
x1, x2, x3 0
2
1 2 2 3 3 1
25
Tải tài liệu tại sividoc.com
1
−
1 2 2 3 3
Đ t x = tan α, x = tan β, x = tan γ với α, β, γ ∈ −
π
;
π
.
1 2 3
2 2
Ta có
tan α tan β + tan β tan γ + tan γ tan α = 1
⇔ 1 − tan α tan β = (tan α + tan β) tan γ (2.7)
Neu tan α tan β = 1 thì tan α + tan β = 0 ho c tan γ = 0.
lý.
Neu tan α + tan β = 0, thì tan α = − tan β, khi đó − tan2
β = 1, vô
Neu tan γ = 0 thì x3 = 0, vô lý.
V y tan α tan β /= 1. Khi đó tà (2.7), ta có
tan α + tan β
1 − tan α tan β
. tan γ = 1
π
⇔ tan(α + β) = tan(
2
− γ)
π
⇔ α + β + γ =
2
+ kπ, (k ∈ Z)
và
1
(x1 −
x
1
)(x2 −
x
1
) + (x2 −
x
1
)(x3 −
x
1
) + (x3 −
x
1
)(x1 −
x
)
= (tan α − cot α)(tan β − cot β)
+ (tan β − cot β)(tan γ − cot γ)
+ (tan γ − cot γ)(tan α − cot α)
= 4 cot 2α cot 2β + 4 cot 2γ(cot 2α + cot 2β)
= 4cot 2α cot 2β − 4 cot(2α + 2β)(cot 2α + cot 2β)
( do 2γ = π − (2α + 2β + k2π))
= 4cot 2α cot 2β 4
cot 2α cot 2β − 1
(cot 2α + cot 2β) = 4.
cot 2α + cot 2β
V y
1 1 1 1 1 1
(x1 −
x
)(x2 −
x
) + (x2 −
x
)(x3 −
x
) + (x3 −
x
)(x1 −
x
) = 4,
đieu phải cháng minh.
1 2 2 3 3
1
26
Tải tài liệu tại sividoc.com
— −
≤
4
−
4 8
3
Bài toán 2.9. Cho phương trình x3
px2
+ qx p = 0 với p, q là các so
dương. Cháng minh rang neu phương trình đã cho có ba nghi m phân bi t
x1, x2, x3 lớn hơn ho c bang 1, thì ta luôn có
p ≥
1
+
√
2
!
(q + 3). (2.8)
Lài giai. Giả sả x1 < x2 < x3. Theo định lý Vieete, ta có
x1 + x2 + x3 = p
x1x2 + x2x3 + x1x3 = q
x1x2x3 = p
Đ t x1 = tan A, x2 = tan B, x3 = tan C với A, B, C là ba góc của tam giác
và
π
A, B, C <
4
π
2
. Không giảm tőng quát, ta giả sả A = min{A, B, C}
π π
thì ≤ A ≤ .
Khi đó (2.8) tương đương với
2 +
√
2
tan A tan B tan C ≥
2 +
√
2
(tan A tan B + tan B tan C + tan C tan A + 3)
8
⇔ 1 ≥ (cot A + cot B + cot C + 3 cot A cot B cot C)
8
⇔ 8 − 4
√
2 ≥ cot A + cot B + cot C + 3 cot A cot B cot C. (2.9)
Đe cháng minh (2.9), ta chú ý
cot A + cot B + cot C + 3 cot A cot B cot C
2 sin A
= cot A +
cos(B − C) + cos A
+ 3 cot A
cos(B − C) + cos(B + C)
cos(B − C) − cos(B + C)
cot A +
2 sin A
1 + cos A
+ 3 cot A
1 − cos A
,
1 + cos A
1 − tan2 A
1
cot A =
tan A
1
=
A
=
2 tan
2
1 tan2 A
2
2 ,
A
2 tan
2
≤
27
Tải tài liệu tại sividoc.com
≤
2
2 2
2 sin A
=
1 + cos A
2.2sin
A A
cos
2 2
A
A
= 2 tan 2
2 cos2
2
và
1 − tan2 A
2 sin2 A
3 cot A
1 − cos A
= 3
1 + cos A
2
A
2 tan
2
2
2 cos2
A
2
1 − tan2 A
= 3
A
2
2 tan 2
tan2 A
2
1 − tan2 A
tan
A
tan
A
− tan3 A
= 3
nên ta có
2 2
= 3 2 2 ,
2 2
cot A + cot B + cot C + 3 cot A cot B cot C
1 − tan2 A
tan
A
− tan3 A
hay
≤
A
2
2 tan
2
A
+ 2 tan + 3
2 2
cot A + cot B + cot C + 3 cot A cot B cot C
≤
1
+ 3 tan
A
−
3
tan3 A
A
2 tan
2
π π
2 2 2
π A π
M t khác vì
4
≤ A ≤
3
⇒
8
≤ nên ta có
2 6
Xét hàm so
√
2 − 1 ≤ tan
A 1
√
3
.
3 3
f(t) = −
2
t
1
+ 3t +
2t
, t ∈
1
2 − 1; √
3
.
≤
√
28
Tải tài liệu tại sividoc.com
2
r
−
— −
— − −
√ 1
π
−
— −
2
≤ 0,
Ta có
f′(t) =
3
t2
2
1 9t4
+ 6t2
1
+ 3 −
2t2
=
2t2
= −
(3t2
− 1)2
2t
với ∀t 0 nên f(t) nghịch bien trên 2 − 1; √
3
. Suy ra
f(t) ≤ f(
√
2 − 1) = 8 − 4
√
2,
ta có đieu phải cháng minh.
cos(B − C) = 1
3π
B = C = ,
Dau "=" xảy ra khi A
tan =
2
√
2 − 1
hay 8
A =
4
.
2.1.2 Phương trình b c ba nh n các yeu to trong tam giác là
nghi m
Trong phan này ta xét các ∆ABC với đ® dài ba cạnh BC = a, CA =
b, AB = c. Ký hi u p là nảa chu vi, r và R là bán kính các đường tròn
n®i, ngoại tiep, ra, rb, rc tương áng là bán kính các đường tròn bàng tiep,
ha, hb, hc tương áng là đ® dài các đường cao,. . .
Bài toán 2.10 (Phương trình b c ba theo đ® dài các cạnh của tam giác).
Cháng minh rang a, b, c là ba nghi m của phương trình b c ba dưới đây:
x3
− 2px2
+ (p2
+ r2
+ 4Rr)x − 4Rrp = 0. (2.10)
A
Lài giai. Tà h thác tan =
2
r
p − a
và a = 2R sin A, suy ra
A
hay
2 tan
a = 2R 2
1 + tan2 A
2
r
a = 4R
p − a
1 +
p − a
= 4Rr
p − a
.
r2 + (p − a)2
Như v y, ta có quan h a(a2
2pa + p2
+ r2
) = 4Rr(p a) hay a3
2pa2
+
(p2
+ r2
+ 4Rr)a 4Rrp = 0. Do đó a là m®t nghi m của x3
2px2
+(p2
+
r2
+ 4Rr)x 4Rrp = 0. Tương tự, b và c cũng là nghi m của phương trình
này.
29
Tải tài liệu tại sividoc.com
— − −
— − −
−
Bài toán 2.11. Cháng minh rang
1 1 1
, ,
a b c
là nghi m của phương trình
3 p2
+ r2
+ 4Rr 2 1 1
x x
4pRr
1
+
2Rr
x −
4pRr
= 0. (2.11)
Lài giai. Thay x bởi , ta có ngay đieu phải cháng minh.
x
Bài toán 2.12. Cháng minh rang p a, p b, p c là các nghi m của
phương trình
x3
− px2
+ r(r + 4R)x − pr2
= 0.
Lài giai. Theo công thác di n tích ta có
S2
= p(p − a)(p − b)(p − c) = p2
r2
⇒ (p − a)(p − b)(p − c) = pr2
.
p − a + p − b + p − c = 3p − (a + b + c) = p.
Giờ ta cháng minh
(p − a)(p − b) + (p − b)(p − c) + (p − c)(p − a) = r(r + 4R)
⇔ (p − a)(p − b) + (p − b)(p − c) + (p − c)(p − a) =
⇔ (p − a)(p − b) + (p − b)(p − c) + (p − c)(p − a)
=
abc
+
(p − a)(p − b)(p − c)
S abc S
( + )
p S p
p p
boxTac
abc
+
(p − a)(p − b)(p − c)
p p
abc + p3
− p2
a − p2
b − p2
c + pab + pbc + pca − abc
p
= p2
− pa − pb − pc + ab + bc + ca
= 3p2
− pa − pb − pc + ab + bc + ca − 2p2
= 3p2
− pa − pb − pc + ab + bc + ca − p(a + b + c)
= p2
− pa − pb + ab + p2
− pb − pc + bc + p2
− pc − pa + ca
= (p − a)(p − b) + (p − b)(p − c) + (p − c)(p − a).
V y đȁng thác được cháng minh. Ta có p a, p b, p c là ba nghi m của
phuơng trình
x3
− px2
+ r(r + 4R)x − pr2
= 0.
=
30
Tải tài liệu tại sividoc.com
3
−
Bài toán 2.13.
1
,
p − a
1
,
p − b
1
p − c
là ba nghi m của phương trình
x3
−
4R + r
x2
+
1
x −
1 = 0.
pr r2 pr2
1
Lài giai. Thay x bởi , ta có đieu phải cháng minh.
x
Bài toán 2.14 (Phương trình b c ba theo các đường cao). Chúng minh
rang ha, hb, hc là các nghi m của phương trình
p2
+ r2
+ 4Rr
x x2
+
2R
2p2
r
R
x −
2p2
r2
= 0.
R
1
Lài giai. Theo công thúc tính di n tích tam giác, ta có S =
2
aha =
pr ⇒ ha =
2pr
a
. Tương tự ta có hb =
2pr
b
, hc =
2pr
c
. Nên ha + hb + hc =
1 1
2pr( +
a b
1
+ ) và
c
ha + hb + hc = 2pr
p2
+ r2
+ 4Rr p2
+ r2
+ 4Rr
= .
4pRr 2R
h h + h h + h h 2pr 2pr 2pr 2pr 2pr 2pr
= + + = 4p2
r2
(
1 1 1
+ + ),
a b b c c a
a b b c c a
2 2 1 2p2
r
ab bc ca
hahb + hbhc + hcha = 4p r = .
2Rr R
hahbhc
= 8p3
r3 1
abc
. Ta có
hahbhc = 8p3 r3 1
=
4pRr
2p2
r2
.
R
V¾y ha, hb, hc là nghi m của phương trình
x3
−
Bài toán 2.15.
p2
+ r2
+ 4Rr
x2
+
2R
2p2
r
R
x −
2p2
r2
= 0.
R
1 1 1
, ,
ha hb hc
31
Tải tài liệu tại sividoc.com
là nghi m của phương trình
3 1 2 p2
+ r2
+ 4Rr 2R
x −
r
x +
1 4p2r2
x −
4p2r2
= 0.
Lài giai. Thay x bởi
x
ta có ngay đieu phải cháng minh.
Bài toán 2.16 (Phương trình b c ba theo bán kính đường tròn bàng tiep).
Cháng minh rang ra, rb, rc là nghi m của phương trình
x3
− (4R + r)x2
+ p2
x − p2
r = 0.
Lài giai. Tà S = pr = ra(p − a) = rb(p − b) = rc(p − c) suy ra
ra =
hay
pr
p − a
, rb =
pr
p − b
, rc =
p3r3
pr
p − c
rarbrc =
(p − a)(p − b)(p − c)
p4r3 p4r3
2
Ta có
= =
p(p − a)(p − b)(p − c) S2
= p r.
1 1 1
ra + rb + rc = pr(
p − a
+
p − b
+
p − c
)
4R + r
= pr
pr
= 4R + r.
và rarb +rbrc +rcra
= p2
r2
(
1
(p − a)(p − b)
1
+
(p − b)(p − c)
1
+ )
(p − c)(p − a)
= p2
r2 1
r2
= p2
.
V y ra, rb, rc là nghi m của phương trình
x3
− (4R + r)x2
+ p2
x − p2
r = 0.
Bài toán 2.17. Cháng minh rang
là nghi m của phương trình
1 1 1
, ,
ra rb rc
x3
−
1
x2
+
4R + r
x −
1 = 0.
r p2r
1
p2r
Lài giai. Thay x bởi
x
ta có ngay đieu phải cháng minh.
32
Tải tài liệu tại sividoc.com
d
2
2.2 Giải phương trình b c bon
Trong phan này ta sě nêu phương pháp chung đe phân tích m®t đa
thác b c bon tőng quát thành tích của hai tam thác b c hai. Đoi với m®t
so dạng đa thác b c bon đ c bi t có nhǎng phép bien đői phù hợp và đơn
giản hơn, không đòi hỏi phải v n dụng toàn b® thu t toán tőng quát.
Bài toán 2.18. Giải phương trình
ax4
+ bx3
+ cx2
+ dx + e = 0 (2.12)
với
ad2
= eb2
, a, e /= 0. (2.13)
(Phương trình (2.12)-(2.13) có tên goi : phương trình hoi quy b c bon).
Lài giai. Viet (2.13) dưới dạng
e
a
= ( )2
.
b
d
Đ t
b
= α có d = bα; e = a(α)2
. The vào phương trình (2.12), ta được:
ax4
+ bx3
+ cx2
+ bαx + a(α)2
= 0
⇔ a(x
2
— α ) + bx(x
2 2
+ α) + cα2
= 0
2
⇔ a(x + α) + bx(x + α) + (c − 2aα)x = 0
Nh n xét x = 0 không thỏa mãn phương trình. Chia hai ve của phương
trình cho x2
ta được:
với t =
x2
+ α
x
at2
+ bt + c − 2aα = 0
V y phương trình (2.12)-(2.13) đưa ve h :
at2
+ bt + c − 2aα = 0
x2
− tx + α = 0
(2.14)
Giải phương trình thá nhat của h (2.14) ta tính được t và tà phương
trình thá hai của h (2.14) ta tính được x.
4 2
33
Tải tài liệu tại sividoc.com
2 2 2
2 2 2 2
−
2
Bài toán 2.19. Giải phương trình
(x − a)4
+ (x − b)4
= c (2.15)
Lài giai Đ t x = t +
a + b
; α =
a − b
. Khi đó
2 2
(2.15) ⇔ (t + α)4
+ (t − α)4
= c
⇔ [(t + α) − (t − α)
2
]2
+ 2(t2
— α ) = c
2
⇔ 16α t + 2(t − α ) = c.
Ta nh n được phương trình trùng phương: 2t4
+ 12α2
t2
+ 2α4
− c = 0.
Bài toán 2.20. Giải phương trình
x4
= ax2
+ bx + c
với
b2
= 4(a + 2)(c + 1)
Lài giai Tam thác b c hai với
∆ = b2
− 4(a + 2)(c + 1) = 0
có dạng f(x) = (a + 2)x2
+ bx + c + 1
và f(x) = (a + 2)x2
+ bx + c + 1 có nghi m kép và
f(x) =
(a + 2)(x +
b
)2
neu a 2
2(a + 2)
c + 1 neu a = −2
Phương trình đã cho được viet dưới dạng
x4
+ 2x2
+ 1 = (a + 2)x2
+ bx + c + 1
2 2
⇔ (x + 1) = (a + 2)x + bx + c + 1
1) Neu a + 2 = 0(⇔ a = −2) thì b = 0 và ta được phương trình
(x2
+ 1)2
= c + 1
2 ) Neu a + 2 < 0 thì phương trình vô nghi m vì có
Ve trái ≥ 0
Ve phải < 0
34
Tải tài liệu tại sividoc.com
h i
/
≥
⇔
h i
⇔±
2
3) Neu a + 2 > 0(⇔ a > −2) thì c > −1 thì ta có the viet
(x2
+ 1)2
= (
√
a + 2x ±
√
c + 1)2
(dau + áng với b >√
0, dau - án
√
g với b < 0) và ta được các phương trình
b c hai x2
+ 1 = ±( a + 2x ± c + 1)
Tiep theo giải và bi n lu n theo a, c.
Bài toán 2.21. Giải phương trình
x4
= ax2
+ bx + c, b /= 0
Lài giai. Goi α là so thực thỏa mãn h thác
b2
= 4(a + 2α)(c + α2
) (2.16)
(ton tại ít nhat m®t giá trị α thỏa mãn (2.16)) vì (2.16) là phương trình
b c ba đoi với α)
Khi đó tam thác b c hai
f(x) = (a + 2α)x2
+ bx + (c + α2
)
có nghi m kép và
f(x) =
(a + 2α) x +
b 2
neu a + 2α = 0
2(a + 2α)
c + α2
neu a + 2α = 0
Viet phương trình đã cho dưới dạng
x4
+ 2αx2
+ α2
= f(x)
2
⇔ (x + α) = f(x) (2.17)
1) Neu a + 2α = 0 thì (2.17) (x2
+ α)2
= c + α2
2) Neu a + 2α < 0 thì Ve trái 0; Ve phải <0, nên phương trình vô
nghi m.
3) Neu a + 2α > 0 thì (2.17) x2
+ α =
√
a + 2α x +
b
.
2(a + 2α)
Bài toán 2.22. Giải phương trình
t4
+ αt3
+ βt2
+ γt + δ = 0
35
Tải tài liệu tại sividoc.com
−
/ −
1
a
⇔
Lài giai. Đ t t = x −
α
. Khi đó ta được phương trình:
4
(x −
α
)4
+ α(x −
α
)3
+ β(x −
α
)2
+ γ(x −
α
) + δ = 0
4 4 4
4 2 6α2
4
2α3
1 1 4
⇔ x = ax + bx + c với a =
42
; b = −
42
+
2
αβ − γ; c =
42
(3α −
42
βα2
+43
αγ 44
δ). Áp dụng bài toán trên ta tìm được nghi m của phương
trình.
H quả 2.2. Moi đa thác b c bon có nghi m thực đeu phân tích được
thành tích của hai tam thác b c hai với h so thực.
Bài toán 2.23. Cho α = 0. Khai trien bieu thác (1 α
√
x)8
+(1+α
√
x)8
=
P(x) ta được P(x) là m®t đa thác b c bon.
Giải phương trình P(x) = 0
Lài giai. Đ t α2
x = t, ta được phương trình
1 + 28t + 70t2
+ 28t3
+ t4
= 0 ⇔ (t +
1
1
)2
1
+ 28(t +
1
) + 68 = 0.
t
Đ t u = t +
t
ta được phương trình
u2
+ 28u + 68 = 0 ⇔ u1,2 = −14 ±
√
128 ⇒ t =
1
2
(u1,2 ±
q
u2
,2
−4).
Bài toán 2.24. Giải phương trình
a(ax2
+ bx + c)2
+ b(ax2
+ bx + c) + c = x.
Lài giai. Đ t ax2
+ bx + c = y ta được h
ax2
+ bx + c = y ax2
+ bx + c = y
ay2
+ by + c = x a(x2
− y2
) + (b + 1)(x − y) = 0
⇔
ax2
+ bx + c = y y = x
ho c
(x − y)[a(x + y) + b + 1] = 0 ax + (b − 1)x + c = 0
y = −
b + 1
− x
ax2
+ bx + c = −
b + 1
a
— x.
2
⇔
36
Tải tài liệu tại sividoc.com
— ⇔ −
√
√ √
"
− −
√
2 2
Giải moi phương trình b c hai ta được nghi m của h , tà đó ta suy ra
nghi m của phương trình.
Bài toán 2.25. Giải phương trình x4
= 3x2
+ 10x + 4.
Lài giai. Viet phương trình dưới dạng
(x2
+ α)2
= (3 + 2α)x2
+ 10x + 4 + α2
.
Chon α đe ∆′ = 25 (3 + 2α)(4 + α2
) = 0 2α3
+ 3α2
+ 8α 13 = 0
Ta thay α = 1 thỏa mãn. V y có the viet phương trình dưới dạng
(x2
+1)2
= 5x2
+10x+5 ⇔ (x +1)2
= [
√
5(x+1)]2
⇔
x2
+ 1 =
√
5(x + 1)
x2
+ 1 = −
√
5(x + 1)
Giải tàng trường hợp suy ra nghi m của phương trình đã cho là x1,2 =
5 ± 1 + 4 5
.
2
Bài toán 2.26. Giải phương trình x2
Lài giai. Đieu ki n x = 1.
Viet phương trình đã cho dưới dạng
x2
+
(x + 1)2
= 3.
x 2 x2
x2
2 x2
(x −
x + 1
) = 3 − 2.
x + 1
⇔ (
x + 1
) + 2.
x + 1
− 3 = 0
x2
x 1 = 0
⇔
x2
+ 3x + 3 = 0 (vô nghi m)
⇔ x1, x2 =
1 ±
√
5
.
2
V y phương trình đã cho có hai nghi m x1 , x2 =
1 ±
√
5
.
2
Bài toán 2.27 (VMO 1973). Giải phương trình x4
= 4x + 1.
Lài giai.Viet phương trình đã cho dưới dạng
x4
+ 2x2
+ 1 = 4x + 1 + 2x2
+ 1
(x2
+ 1)2
= 2(x + 1)2
⇔ (x2
+ 1)2
− [
√
2(x + 1)]2
= 0
⇔ [x + 1 +
√
2(x + 1)][x + 1 −
√
2(x + 1)] = 0
⇔ x2
+ 1 −
√
2(x + 1) = 0 ⇔ x
1,2
=
√
2p 4
√
2 − 2
.
2
"
2
37
Tải tài liệu tại sividoc.com
≤
2 2 2 2
2 2 2 2
3 x +
1
= 4 y +
1
= 5 z +
1
2.3 M t so h phương trình đưa ve phương trình
b c ba và b c bon
Nh n xét rang, neu như x2
+y2
= 1 thì ta sả dụng phép the x = sin α,
y = cos α. Trong nhieu trường hợp, phép the này sě tỏ ra hi u quả.
Bài toán 2.28. Giải h phương trình
(
x2
+ y2
= 1
.
4xy(2y2
− 1) = 1
Lài giai. Đ t x = sin α, y = cos α với 0 α < π thì ta được sin 4α = 1.
π
Nghĩa là 4α = + k2π với k = 0, 1, 2, 3. Tà đó có the de dàng suy ra
2
√
2 −
√
2
,
√
2 +
√
2
!
,
√
2 +
√
2
, −
√
2 −
√
2
!
,
−
√
2 −
√
2
, −
√
2 +
√
2
!
, −
√
2 +
√
2
,
√
2 −
√
2
!
.
Ta thay phép the này giải quyet bài toán rat nhanh gon, trong khi đó
các cách giải bài toán không sả dụng phép the lượng giác sě phác tạp hơn
nhieu.
Neu ta có h thác dạng xy + yz + zx = 1 thì ton tại α, β, γ sao cho
x = tan
α
, y = tan
2
β γ
, z = tan
2 2
α
và α + β + γ = π.
β
Th t v y, đ t x = tan
2
, y = tan
2
, (−π < α, β < π), thì
z = cot = tan − .
α + β π α + β
2 2 2
Tiep theo, đ t γ = π − (α + β).
Bài toán 2.29. Giải h phương trình
x y z
xy + yz + zx = 1
38
Tải tài liệu tại sividoc.com
−
3 2 3 2
xy + yz + zx = 1
Lài giai. Vì
x
3(x2 + 1)
=
y
4(y2 + 1)
=
z
5(z2 + 1)
nên x, y, z có cùng dau,
ngoài ra, neu (x, y, z) là nghi m của h thì (−x, −y, −z) cũng là nghi m.
Như v y, ta chỉ can đi tìm các nghi m dương. Đ t x = tan
α β
, y = tan ,
2 2
γ
z = tan
2
(0 < α, β, γ < π, α+β +γ = π), ta được
sin α
3
sin β
=
4
sin γ
= .
5
Tà định lý hàm so sin bây giờ suy ra α, β, γ là các góc của tam giác có
đ® dài các cạnh tương áng là 3, 4, 5. Tam giác này là tam giác vuông có
π
γ = , sin α =
2
3
, sin β =
5
4 α
. Vì the tan
5 2
1 β
= , tan
3 2
1 γ
= , tan
2 2 = 1.
Như v y đáp so của bài toán là
1
,
1
, 1 , −
1
, −
1
, −1 .
Bài toán 2.30. Cháng minh rang giǎa 7 so thực bat kỳ luôn tìm được hai
so x và y sao cho
0 <
x − y
1 + xy
1
√
3
.
Lài giai. Goi các so đã cho là a1, a2, . . . , a7. Với moi so thực a, ton
tại so α thu®c khoảng −
π
,
π
sao cho a = tan α. Giả sả a = tan α ,
2 2 1 1
a2 = tan α2, . . . , a7 = tan α7.
π
Trong 7 so α1, α2, . . . , α7 ton tại hai so có hi u không vượt quá
6
.
Giả sả hai so này là α và β, trong đó α > β. Khi đó
tan α − tan β π 1
0 <
1 + tan α tan β
= tan(α − β) ≤ tan
6
= √
3
.
Như v y các so x = tan α và y = tan β là các so can tìm.
Lời giải của bài toán trở nên hien nhiên sau khi ta chon được m®t phép
the lượng giác thích hợp. Bản chat của phép the lượng giác nam ở cho các
bien so có m t trong bài toán được xét như giá trị của các hàm lượng giác
nào đó. Trong đó các hàm so lượng giác can được chon đe bieu thác thu
được càng gon càng tot. Bài viet này nói ve các phép the lượng giác.
Bài toán 2.31. Giải h phương trình
1 x2
1 + x2
=
2y
1 + y2
=
1 − z2 .
1 + z2
≤
39
Tải tài liệu tại sividoc.com
1 = cos
1
5
5 5
2 3 3 3
Chương 3. Phương pháp lư ng giác
giải phương trình đa thfíc b c cao
Trong chương này ta xét m®t so dạng phương trình b c cao và h
phương trình tương áng giải được bang phương pháp lượng giác ho c sả
dụng các đȁng thác đại so sinh bởi các h thác lượng giác.
3.1 Phương trình đa thfíc b c cao
Tiep theo, ta xét m®t so áng dụng khác của đȁng thác đại so - lượng
giác.
1
Bài toán 3.1. Giải phương trình T5(x) =
Chebyshev loại 1 b c 5.
Nh n xét 3.3. Đe ý rang, tà đȁng thác
2
, trong đó T5(x) là đa thác
ta suy ra
cos 5t + cos t = 2 cos 3t cos 2t,
T5(x) = 16x5
− 20x3
+ 5x, x ∈ [−1, 1].
Lài giai. Đe ý rang
T5(x) = cos(5 arccos x), x ∈ [−1, 1]
và T5(x) < −1 khi x < −1, T5(x) > 1 khi x > 1.
V y nên chỉ can tìm nghi m của phương trình đã cho trong (−1, 1).
Ta có = cos = cos ± 2π = cos ± 4π .
1 π π π
Tà đó, suy ra phương trình đã cho có 5 nghi m phân bi t
π
x , x = cos
π3 ± 2π
, x = cos
π3 ± 4π
.
2,3 4,5
40
Tải tài liệu tại sividoc.com
−
5
2
+ )
a 2 a5
Bài toán 3.2. Giải phương trình 16x5
20x3
+ 5x =
5
.
2
Nh n xét 3.4. Đe ý rang, tà đȁng thác
cos 5t + cos t = 2 cos 3t cos 2t,
ta suy ra
16x5
− 20x3
+ 5x = T5(x), x ∈ [−1, 1].
Lài giai. Đe ý rang
T5(x) = cos(5 arccos x), x ∈ [−1, 1]
và T5(x) < −1 khi x < −1, T5(x) > 1 khi x > 1.
V y nên chỉ can tìm nghi m của phương trình đã cho trong (1, +∞).
5 1 1
Sả dụng các đȁng thác = 2 + và
4 2
T
1
(a
1
2
1
a5
+
1
,
suy ra phương trình có nghi m
x1 =
1 √
5
2
1
2 + √
5
2
.
Ta cháng minh phương trình đã cho có nghi m duy nhat trong (1, +∞).
Th t v y, ta thay T′(x) = 80x4
− 60x2
+ 5 > 0, ∀x ∈ (1, +∞) nên ve
trái của phương trình đã cho là hàm đong bien trong (1, +∞).
H quả 3.1. Xét phương trình
16x5
− 20x3
+ 5x = m. (∗)
Khi đó
- với m ∈ (−1, 1) thi phương trình (*) có 5 nghi m phân bi t,
- với m > 1 ho c m < −1 thi phương trình (*) có 1 nghi m duy nhat,
- với m =
√1 thì phương trình có 1 nghi m đơn x = 1 và 2 nghi m kép
x1,2 =
−1 ± 5
.
4
- với m = √
−1 thì phương trình có 1 nghi m đơn x = −1 và 2 nghi m
kép x1,2 =
1 ± 5
.
4
=
41
Tải tài liệu tại sividoc.com
2
2
2
Bài toán 3.3. Cháng minh rang phương trình
64x6
− 96x4
+ 36x2
− 3 = 0
có nghi m thực x0 thỏa mãn đieu ki n
q
2 +
√
2 +
√
2
< x0 <
q
2 +
√
2 +
√
3
.
Lài giai. Tà công thác cos2
α =
1 + cos 2α
2 (với 0 ≤ α ≤ π), ta suy ra
α
1 + cos
‚
.1 +
r
1 + cos α
q
α
cos
4
Khi α =
=
2
π
, ta có
2 =
, 2
=
1
2 +
√
2 + 2 cos α.
2 2
4
Khi α =
π
, ta có
π
cos
16
=
1
r
2 +
q
2 +
√
2.
6
M t khác, ta cũng có
π
cos
24
=
1
r
2 +
q
2 +
√
3.
Suy ra
cos 6t = 4 cos3
2t − 3 cos 2t
= 4(2 cos2
t − 1)3
− 3(2 cos2
t − 1)
= 32 cos6
t − 48 cos4
t + 18 cos2
t − 1
64 cos6
t − 96 cos4
t + 36 cos2
t − 3 = 2 cos 6t − 1.
Tà phương trình 64x6
− 96x4
+ 36x2
− 3 = 0, ta xét x ∈ [1; 1] và đ t
x = cos t, ta có 2 cos 6t − 1 = 0 ⇒ cos 6t =
π
1 π
2
⇒ t =
18
.
Do đó x0 = cos
18
là m®t nghi m của phương trình. M t khác
π π π
< <
24 18 16
2
,
42
Tải tài liệu tại sividoc.com
2
(x + n) +
— −
hay
π
cos
24
π
> cos
18
π
> cos .
16
V y phương trình 64x6
−96x4
+36x2
−3 = 0 luôn có m®t nghi m x0
thỏa mãn đieu ki n
π
= cos
18
q
2 +
√
2 +
√
2
< x0 <
q
2 +
√
2 +
√
3
.
Bài toán 3.4. Cho b® so m, n, p ∈ R. Giải phương trình
x3
+ m3
x3
+ n3
3 3 x − m x − n x − p
(x + m)3
+
3 x3+p3
(x+p)3
—
2
+
2 x + m x + n x + p
= 0.
Lài giai. Nh n xét rang
x3
+ m3
1 3 (x − m)2
(x + m)3
=
4
+
4 (x + m)2
.
Vì v y phương trình đã cho tương đương với phương trình sau
1 3 (x − m)2
1 3 (x − n)2
1 3 (x − p)2
4
+
4 (x + m)2
+
4
+
4 (x + n)2
+
4
+
4 (x + p)2
−
−
3
+
3 x − m x − n x − p = 0. (1)
2 2 x + m x + n x + p
Đ t
và đe ý rang
x − m
x + m
= a,
x − n
x + n
= b,
x − p
= c
x + p
1
+
3
a2
+
1
+
3
b2
+
1
+
3
c2
−
4 4 4 4 4 4
3 3
−
4
+
2
abc = 0
có the bien đői được ve dạng
(ab + c)2
= (1 − a2
)(1 − b2
). (2)
Thay các giá trị a, b, c theo bien x, m, n, p ta được
(ab + c)2
=
4[x3
+ (mn mp np)x]2
(x + m)2(x + n)2(x + p)2
,
2
43
Tải tài liệu tại sividoc.com
Σ
∈
1 − a2
=
4mx
, 1 − b2
=
4nx
.
V y (2) có dạng
(x + m)2 (x + n)2
x2
[x2
+2(x+p)
√
mn+mn−mp−np][x2
−2(x+p)
√
mn+mn−mp−np] = 0.
Giải ra ta được các nghi m của phương trình là
x1 = x2 = 0,
x3,4 = ±(
√
mp −
√
np) −
√
mn,
x5,6 =
√
mn ± (
√
mp +
√
np.
π
Bài toán 3.5. Cho 0 < α <
Q(x) ∈ R[x] b c n thì đa thác
n + 2
. Cháng minh rang với moi đa thác
P (x) = (x2
− 2x cos α + 1)Q(x)
không the có tat cả các h so đeu không âm.
Lài giai.
Giả sả
và
Khi đó
Q(x) = a0xn
+ a1xn−1
+ · · · + an−1x + an
P(x) = b0xn+2
+ b1xn+1
+ · · · + bn+1x + bn+2.
Suy ra
b0 = a0,
b1 = a1 − 2a0 cos α,
b2 = a2 + a0 − 2a1 cos α,
· · ·
bn+1 = an−1 − 2an cos α,
bn+2 = an.
bk = ak + ak−2 − 2ak−1 cos α, an+2 = an+1 = 0, a−1 = a−2 = 0
và
n+2
bk sin kα = 0.
k=0
Mà sin kα > 0 vì α 0,
π
nên ton tại h so b
n + 2 j
< 0.
44
Tải tài liệu tại sividoc.com
· · ·
2
√ √
0 0
1 1
n n
. . . . . . . . . . ..
Bài toán 3.6. Cho a0, a1, . . . , an là n + 1 so đôi m®t khác nhau. Giải h
phương trình sau
x0 + x1a0 + x2a2
· · · + xnan
= 0
x0 + x1a1 + x2a2
+ · · · + xnan
= 0
x0 + x1an + x2a2
+ · · · + xnan
= 0
Lài giai. Xét đa thác
f(y) = xnyn
+ xn−1yn−1
+ · · · + x1y + x0.
Ta có deg f ≤ n. Tà h (1) ta có
f(a0) = f(a1) = · · · = f(an) = 0,
nên f(y) có n + 1 nghi m phân bi t, do đó f(y) ≡ 0. Tà đó suy ra
x0 = x1 = · · · = xn = 0.
Thả lại ta thay x0 = x1 = = xn = 0 thoả mãn h đã cho. V y h có
nghi m duy nhat
(x0, x1, . . . , xn) = (0, 0, . . . , 0).
Bài toán 3.7. Giải phương trình
√
1 − x2 = 4x3
− 3x.
Lài giai. Đ t x = cos α, 0 ≤ α ≤ π. Ta được sin α = cos 3α, hay
cos 3α − cos
π
− α = 0.
Tà đó
2 sin
π
− 2α sin α +
π
= 0.
4
π
Đen đây ta có the de dàng tìm được α =
8
4
5π
ho c α =
8
ho c α =
3π
. Vì
4
π
cos =
8
1 + cos
π
4 =
2
2 + 2
,
2
‚
.
,
(1)
45
Tải tài liệu tại sividoc.com
−
4
2
=
2 − 2
,
2
‚
.
,1 + cos
5π √ √
cos
3π
√
2
4
= −
2
,
nên t p hợp
(√
2 +
√
2
, −
√
2 −
√
2
, −
√
2
)
là t p nghi m của phương
2 2 2
trình.
Bài toán 3.8. Giải phương trình
x 35
x + √
x2 − 1
=
1
Lài giai. Chú ý rang x > 1. Đ t x =
.
12
, 0 < α <
π
. Phương trình có
the viet lại thành
sin α 2
1 1 35
+ = .
sin α cos α 12
2 t2
− 1
Đ t sin α + cos α = t thì 1 + 2 sin α cos α = t
Thay vào ta được
, suy ra sin α cos α = .
2
2t 35
= 2 7 5
⇔ 35t − 24t − 35 = 0 ⇔ t = ∨ t = − .
t2 − 1 12 5 7
7
Vì t = sin α + cos α > 0 nên ta loại nghi m thá hai. Xét t =
5
12
thì tà đây
ta tính được sin α cos α =
trình
. Như v y sin α, cos α là nghi m của phương
25
X2
−
7
X +
12
= 0.
5 25
3 4
Tà đây ta tính được sin α =
5
5
ho c sin α =
5
. Tương áng ta được nghi m
5
của phương trình là x =
3
và x = .
4
Bài toán 3.9. Giải phương trình
8x3
− 4x − 1 =
√
3
6x + 1.
5π
cos
8
= −
46
Tải tài liệu tại sividoc.com
(
− −
(
− −
3
| | ≤ ∈
| | | − | | | −
x
2
— − −
2
Lài giai. Phương trình đã cho tương đương
(2x)3
− 4x − 1 =
√
3
2x + 4x + 1.
Đ t u = 2x, v =
√
3
2x + 4x + 1, ta có h phương trình
u3
4x 1 = v
v3
− 4x − 1 = u
⇔
u3
v3
= v u
u3
− 4x − 1 = v
⇔
(u − v)(u2
+ uv + v2
+ 1) = 0
u3
− 4x − 1 = v
u = v
u3
− 4x − 1 = u
⇔ 8x − 6x = 1. (1)
Neu x > 1 thì 8x3
6x = 2 x (4x2
3) > 2 nên (1) vô nghi m. Do v y
ta phải có x 1. Đieu này cho phép ta đ t x = cos t với t [0, π]. Khi
đó phương trình (1) có the viet lại thành
cos 3t =
1
2
⇔ t1 =
π
9
∨ t2 =
5π 7π
9
∨ t3 =
9
.
π 5π 7π
V y phương trình đã cho có ba nghi m x = cos , x = cos
9
, x = cos .
9 9
Bài toán 3.10. Tìm x ∈ (0; 1) thỏa mãn đieu ki n
32x(x2
− 1)(2x2
− 1)2
= 1 −
1
.
Lài giai. Vì x ∈ (0; 1) nên ta có the đ t x = cos α với α ∈ 0;
π
.
Ta có
32 cos α(cos2
α 1)(2 cos2
α 1)2
= 1
1
cos α
⇔ − 32 cos α sin α cos2
1
2α = 1 −
cos α
⇔ 8 sin
2
2α cos2
2α = 1 − cos α
⇔ 2 sin 4α = 1 − cos α
⇔ cos α = cos 8α.
Do đó α =
k2π
ho c α =
7
l2π
9
(với k, l ∈ Z.)
Vì rang α ∈ 0;
π
nên k = 1, l = 1 và l = 2.
(
(
⇔
2
2
47
Tải tài liệu tại sividoc.com
| | ≤
− −
Với t =
√
2 thì
π
2
π
2
3
— 2
V y phương trình đã cho có ba nghi m thu®c (0; 1) đó là
x = cos
2π
; x = cos
7
2π 4π
; x = cos .
9 9
Bài toán 3.11. Giải phương trình
x3
+
q
(1 − x2)3 = x
q
2(1 − x2).
Lài giai. Đieu ki n đehcác bieiu thác có nghĩa: −1 ≤ x ≤ 1.
sin3
α + cos3
α =
√
2 sin α cos α
⇔ (sin α + cos α)3
− 3 sin α cos α(sin α + cos α) −
√
2 sin α cos α = 0.
Đ t sin α + cos α =
√
2 sin
π
+ α = t với đieu ki n t
√
2.
4
t2
− 1
Suy ra sin α cos α =
2
và phương trình trở thành
t − 3
t2
1
2
t −
t2
1
2 = 0
2
⇔ t3
+
√
2t2
− 3t −
√
2 = 0
⇔ (t −
√
2)(t +
√
2 − 1)(t +
√
2 + 1) = 0
Suy ra t =
√
2 ho c t = 1 −
√
2 do |t| ≤
√
2 .
√
2 sin
π
+ α =
√
2 ⇔ sin
π
+ α = 1
hay α =
4 4
π
+ k2π.
4
Vì α ∈
h
−
π
;
π i π
nên α =
π
√
2
và do đó x = sin = .
hay
Với t = 1
2√2
, suy ra
4 4 2
sin α + cos α = 1 −
√
2
x +
√
1 − x2 = 1 −
√
2,
Đ t x = sin α (với α ∈ − ; ), thì phương trình trở thành
√
48
Tải tài liệu tại sividoc.com
2
— ∈
2
t
2
2 2 2 2
2 2 2 2
2
s
3
−
tác
x ≤ 1 −
√
2
⇔
x ≤ 1 −
√
2
1 − x2
= (1 −
√
2 − x)2
x2
− (1 −
√
2)x + (1 −
√
2) = 0
⇔ x =
1 −
√
2 +
√
2
√
2 − 1
.
V y phương trình đã cho có 2 nghi m là
x =
√
2
; x =
1 −
√
2 +
√
2
√
2 − 1
.
2 2
Bài toán 3.12. Giải phương trình
q
1 +
√
1 − x2
hq
(1 + x)3 −
q
(1 − x)3
i
= 2 +
√
1 − x2.
Lài giai. Đieu ki n có nghĩa: 1 ≤ x ≤ 1. Đ t x = cos t (với t [0; π]),
phương trình trở thành
hay
√
1 + sin t
q
(1 + cos t)3 −
q
(1 − cos t)3 = 2 +
√
1 − cos2 t
s
sin
t
+ cos
t
2
s
2 cos2
t
2 sin2
2
3
2
= 2 +
√
sin2
t
(3.1)
t
Vì t ∈ [0; π] nên
2 ∈
h
0;
πi
. Do đó
t t
sin
2
≥ 0, cos
2
≥ 0.
V y nên phương trình (3.1) tương đương với
2
√
2 sin
t
+ cos
t
cos3 t
− sin3 t
= 2 + sin t
⇔ 2
√
2 sin
t
+ cos
t
cos
t
− sin
t
×
49
Tải tài liệu tại sividoc.com
2
2
2
— −
≤ ≤
— ≤ ≤
x + 3y = 4y3
× cos2 t + sin
t t
cos
2 2
+ sin2 t
= 2 + sin t
2
⇔ 2
√
2 cos t 1 +
1
sin t = 2 + sin t ⇔
√
2 cos t(2 + sin t) = 2 + sin t
⇔ (
√
2 cos t − 1)(2 + sin t) = 0 ⇔ cos t =
√
2
⇔ x =
√
2
2
√
2
V y phương trình có nghi m là x = .
2
3.2 H phương trình đa thfíc b c cao
Phép the lượng giác thường có ích trong phép giải các h phương trình
hoán vị vòng quanh. Ta xem xét m®t ví dụ như v y.
Bài toán 3.13. H phương trình sau có bao nhiêu nghi m?
y + 3z = 4z3
.
z + 3x = 4x3
Lài giai. Ta viet lại h dưới dạng
x = 4y3
− 3y
y = 4z3
− 3z .
z = 4x3
− 3x
Ta cháng minh rang tat cả các so x, y, z theo trị tuy t đoi không vượt quá
1. Th t v y, giả sả x là so lớn nhat trong các so này và x > 1 thì ta có
z = 4x3
− 3x > x. Ta đi đen mâu thuan.
Neu giả sả x là so nhỏ nhat và x < 1 thì ta cũng có z = 4x3
3x < x,
mâu thuan.
Như v y 1 x, y, z 1 và ta có the thực hi n phép the x = cos α
(0 α π). Khi đó z = cos 3α, y = cos 9α, x = cos 27α. Bây giờ rõ ràng
rang so nghi m của h phương trình ban đau bang so nghi m của phương
trình cos α = cos 27α trên [0, π]. De dàng thay rang so nghi m này đúng
bang 27:
kπ
α = , k = 0, 1, 2, . . . , 13; α =
13
kπ
, k = 1, 2, . . . , 13.
14
50
Tải tài liệu tại sividoc.com
≤
2 2 2 2
2 2 2 2
3 x +
1
= 4 y +
1
= 5 z +
1
3 2 3 2
Nh n xét 3.5. Neu như trong bài toán có đieu ki n x2
+ y2
= 1 thì phép
the x = sin α, y = cos α trong nhieu trường hợp sě tỏ ra hi u quả.
Bài toán 3.14. Giải h phương trình
x2
+ y2
= 1
4xy(2y2
− 1) = 1
Lài giai. Ta đ t x = sin α, y = cos α với 0 α < π thì ta được
π
sin 4α = 1. Nghĩa là 4α =
tìm ra đáp so cuoi cùng
+ k2π với k = 0, 1, 2, 3. Tà đó có the de dàng
2
√
2 −
√
2
,
√
2 +
√
2
!
,
√
2 +
√
2
, −
√
2 −
√
2
!
,
−
√
2 −
√
2
, −
√
2 +
√
2
!
, −
√
2 +
√
2
,
√
2 −
√
2
!
.
Ta thay phép the này giải quyet bài toán rat nhanh gon, trong khi đó
các cách giải bài toán không sả dụng phép the lượng giác phác tạp hơn
nhieu.
Bài toán 3.15. Giải h phương trình
x y z .
xy + yz + zx = 1.
Lài giai. Vì x
3(x2 + 1)
=
y
4(y2 + 1)
=
z
5(z2 + 1)
nên x, y, z có cùng dau,
ngoài ra, neu (x, y, z) là nghi m của h thì (−x, −y, −z) cũng là nghi m.
Như v y ta chỉ can đi tìm các nghi m dương. Đ t x = tan
α β
, y = tan ,
2 2
γ
z = tan
2
(0 < α, β, γ < π, α+β +γ = π), ta được
sin α
3
sin β
=
4
sin γ
= .
5
Tà định lý hàm so sin bây giờ suy ra α, β, γ là các góc của tam giác có
đ® dài các cạnh tương áng là 3, 4, 5. Tam giác này là tam giác vuông có
π
γ = , sin α =
2
3
, sin β =
5
4 α
. Vì the tan
5 2
1 β
= , tan
3 2
1 γ
= , tan
2 2 = 1.
Như v y đáp so của bài toán là
1
,
1
, 1 , −
1
, −
1
, −1 .
(
51
Tải tài liệu tại sividoc.com
r
2
b
a
Chương 4. M t so dạng toán liên
quan
4.1 Phép the lư ng giác
4.1.1 Phép the lư ng giác trong bat đang thfíc
Chúng ta sě xem xét các ví dụ sả dụng phép the lượng giác đe cháng
minh m®t so dạng bat đȁng thác.
Bài toán 4.1. a, b, c, d là các so dương. Cháng minh bat đȁng thác
√
ab +
√
cd ≤
q
(a + d)(b + c).
Lài giai. Viet lại bat đȁng thác dưới dạng
a
a + d
b
·
b + c
+
c
b + c
d
·
a + d ≤ 1.
a
Đ t
a + d
= sin2
α,
b
b + c
= sin2
β 0 < α, β <
π
. Khi đó bat đȁng thác
có the đưa ve dạng sin α sin β + cos α cos β ≤ 1, hay là cos(α − β) ≤ 1.
Như ta đã thay phép the lượng giác đã giúp chúng ta phá được các
dau căn thác và đưa ve dạng đơn giản hơn. Trong ví dụ dưới đây chúng ta
cũng sả dụng phép the lượng giác đe đưa bat đȁng thác ve dạng đơn giản
hơn.
Bài toán 4.2. Cho a, b, c là các so dương, trong đó c là so nhỏ nhat trong
chúng. Cháng minh các bat đȁng thác
c
+
c √
ab ≤
q
(a + c)(b + c) +
q
(a − c)(b − c) ≤ 2
√
ab.
r
52
Tải tài liệu tại sividoc.com
4
a b a b a
—
b
≤
1 − a
Lài giai. Viet lại bat đȁng thác dưới dạng
c
+
c
≤
r
1 +
c
1 +
c
+
r
1 −
c
1
c
2.
c
Đ t
a
c
= sin 2α,
b
= sin 2β, 0 < α, β ≤
π
, bat đȁng thác trở thành
sin 2α+sin 2β ≤ (sin α+cos α)(sin β+cos β)+(cos α−sin α)(cos β−sin β) ≤ 2,
hay
2 sin(α + β) cos(α − β) ≤ 2 cos(α − β) ≤ 2.
Chúng ta cũng lưu ý rang ve phải của bat đȁng thác này
q
(a + c)(b + c) +
q
(a − c)(b − c) ≤ 2
√
ab
là m®t h quả đơn giản của bài toán 3.
Bài toán 4.3. Cháng minh rang với moi so tự nhiên n (n ≥ 2) và với moi
a, ta có
−(1 + a2
)n
≤ (2a)n
+ (1 − a2
)n
≤ (1 + a2
)n
.
Lài giai. Bat đȁng thác can cháng minh tương đương với
−1 ≤
α
2a n
1 + a2
+
2 n
1 + a2
≤ 1
Đ t tan
2
= a với −π < α < π, ta có
2a 1 − a2
1 + a2
= sin α;
1 + a2
= cos α.
Bat đȁng thác can cháng minh có dạng
−1 ≤ sinn
α + cosn
α ≤ 1.
Th t v y ta có
−1 ≤ sin α ≤ 1 ⇒ − sin2
α ≤ sinn
α ≤ sin2
α, với ∀n ≥ 2.
Tương tự, ta cũng có
−1 ≤ cos α ≤ 1 ⇒ − cos2
α ≤ cosn
α ≤ cos2
α, với ∀n ≥ 2.
Do đó −1 ≤ sinn
α + cosn
α ≤ 1, bài toán được cháng minh.
53
Tải tài liệu tại sividoc.com
c
b
n
−
1 2 n
1 2 n
a
k
Bài toán 4.4. Cho a, b, c là các so dương, trong đó c là so nhỏ nhat. Cháng
minh các bat đȁng thác
. −
c
.
√
ab ≤
q
c(a − c) +
q
c(b − c) ≤
√
ab.
4.1.2 Phép the lư ng giác trong dãy so
Cuoi cùng ta đi đen các phép the lượng giác trong dãy so. Ta xem xét
hai bài toán khó mà có the đưa ve các dãy truy hoi. Lời giải các h thác
truy hoi này thu được nhờ vào các phép the lượng giác.
Bài toán 4.5. a1, a2, . . . , an là các so thực sao cho a2
+ a2
+ · · · + a2
= 1.
1 2 n
Tìm giá trị lớn nhat của bieu thác
a1a2 + a2a3 + · · · + an−1an.
Lài giai. Ta xét các so C sao cho bat đȁng thác
a1a2 + a2a3 + · · · + an−1an ≤ C(a2
+ a2
+ · · · + a2
)
đúng với moi so thực a1, a2, . . . , an. So C nhỏ nhat trong các so như v y
sě là đáp so can tìm. Thá nhat, so C can tìm không vượt quá 1, vì
a2
+ a2
+ · · · + a2
− (a1a2 + a2a3 + · · · + an−1an) =
1 2 n
1 2 2 2 2
=
2
[a1 + (a1 − a2) + · · · + (an−1 − an) + an] ≥ 0.
Ta bien đői bieu thác C(a2
+ a2
+ · · · + a2
) − (a1a2 + a2a3 + · · · + an−1an)
bang cách liên tiep tách bình phương đúng. Ta thu được bieu thác có dạng
p1 a1 −
1
2p1
2
a2 +p2 a2 −
1
2p2
2
a3 +...+pn−1
1
an−1 −
1
2pn−1
2
an +pna2
.
De thay p1 = C và pk+1 = C −
4p
với moi k = 1, 2, . . . , n − 1. Bieu thác
thu được không âm với moi a1, a2, . . . , an khi và chỉ khi tat cả các so p1,
p2, . . . , pn không âm. Như v y bài toán đưa ve vi c tìm so C sao cho tat
cả các so hạng của dãy so p1, p2, . . . , pn đeu không âm.
Vì 0 < C ≤ 1 nên ta có the đ t C = cos α, trong đó 0 ≤ α <
đó
π
. Khi
2
1
p2 = cos α −
4 cos α
=
4 cos2
α 1
=
4 cos α
2 cos α sin 2α − sin α
=
2 sin 2α
sin 3α
.
2 sin 2α
54
Tải tài liệu tại sividoc.com
−
k
n
2
Tiep theo
p3
sin 2α
= cos α −
2 sin 3α
=
2 cos α sin 3α − sin 2α
2 sin 3α
sin 4α
= .
2 sin 3α
Bang quy nạp, de dàng cháng minh được rang
sin(k + 1)α
pk = , k = 1, 2, . . . , n.
2 sin kα
Như v y p1, p2, . . . , pn không âm khi và chỉ khi sin α, sin 2α, . . . , sin(n+1)α
π π
không âm. Như v y 0 ≤ α ≤ và giá trị C can tìm bang cos .
n + 1 n + 1
Bài toán 4.6. x1, x2, . . . , xn là các so dương. Goi A là so nhỏ nhat trong
1
các so x1, x2 +
1
1
, x3 +
2
, . . . , xn +
x
1
,
n−1
1
, còn B là so lớn nhat trong
xn
các so này. Cháng minh rang giá trị lớn nhat của A bang giá trị nhỏ nhat
của B và hãy tìm giá trị này.
Lài giai. Đau tiên ta xét tình huong
1
x1 = x2 +
1
1
= x3 +
2
= · · · = xn +
x
1 1
= .
n−1 xn
1
Khi đó các so x1, x2, . . . , xn thoả mãn h thác truy hoi xk+1 = x1 −
x
,
k = 1, 2, . . . , n 1. H thác tương tự ta đã g p ở ví dụ trước. H thác này
cũng có the giải bang phép the lượng giác.
Đau tiên ta cháng minh rang x1 < 2. Th t v y, neu x1 ≥
1
2 thì ta lan
lượt có x2 ≥ 1, x3 ≥ 1, . . . , xn ≥ 1 và lúc đó đȁng thác x1 =
x
không the
xảy ra.
Bây giờ ta có the đ t x1 = 2 cos α 0 < α < . Tương tự như ở bài
π
toán trước, bang quy nạp de dàng cháng minh được rang xk =
(1 ≤ k ≤ n).
sin(k + 1)α
sin kα
1
Vì x1 =
n
nên tà đây ta có
2 cos α =
sin nα
.
sin(n + 1)α
x x
x x
x
55
Tải tài liệu tại sividoc.com
3 n
4 2
Tà đây sin(n + 2)α = 0, tác là α =
π
.
n + 2
Bây giờ ta cháng minh rang giá trị lớn nhat của A và giá trị nhỏ nhat
π
của B bang 2 cos
n + 2
. Ta chỉ can cháng minh bat đȁng thác
π
A ≤ 2 cos
n + 2
≤ B.
1
Giả sả tat cả các so x1, x2 +
x
1
, x3 +
x
1
, . . . , xn +
x
1
, đeu lớn hơn
x
π
1 2 n−1 n
2cos
n + 2
. Trong trường hợp này ta lan lượt có các bat đȁng thác sau
3π
sin
4π
sin sin (n + 1)π
x2 > n + 2 , x >
2π
sin
n + 2
n + 2 , . . . , x >
3π
sin
n + 2
n + 2
nπ
sin
n + 2
Nhưng khi đó
1
< 2 cos
xn
π
n + 2
. Mâu thuan.
π
Như v y ta đã cháng minh được bat đȁng thác A ≤ 2 cos
n + 2
. Bat
π
đȁng thác 2 cos
n + 2
≤ B cháng minh hoàn toàn tương tự.
4.2 M t so dạng toán tfi các đe thi Olympic sfi dnng
phương pháp lư ng giác
Bài toán √
4.7 (IMO - 196
√
5). Tìm tat cả các giá trị của x ∈ [0; 2π] sao cho
2 cos x ≤ | 1 + sin 2x − 1 − sin 2x| ≤ 2.
Lài giai.√ √
Đ t y = | 1 + sin 2x − 1 − sin 2x| ⇒ y2
= 2 − 2| cos 2x|.
Neu cos x ≥ 0 mà 2 cos x ≤ 2 ⇒ | cos 2x| = |2 cos2
x−1| = 1−2 cos2
x.
1 hπ πi 3π 7π
Khi đó đe bài trở thành cos x ≤
Neu cos x < 0
√
2
⇒ x ∈
4
;
2
∪
4
;
4
.
+) Giả sả | cos x| ≤
5π
;
3π
.
1
√
2
ta có cos x ≤ | cos x| ≤
1
√
2
⇒ x ∈
π 3π
;
2 4
.
∪
56
Tải tài liệu tại sividoc.com
r r r
/
r s
r
r s
4 4
sin2
y cos2 x x + y
sin2
y cos2 x
2
!
2
!
| |
+) Giả sả | cos x| >
x ∈
3π
;
5π
.
1
√
2
ta có | cos 2x| = 2 cos x−1 ⇒ y = 2| sin x| ⇒
V y bat phương trình có nghi m là x ∈
π
;
7π
.
Bài toán 4.8 (VMO - 2013). Giải h phương trình
sin2
x +
1
sin2
x
+ cos2 y +
1
cos2 y =
20x
x + y
r
sin2
y +
1
+
r
cos2 x +
1
=
r
20y
Đieu ki n có nghĩa: sin x, cos x, siny, cos y = 0, xy > 0.
Nhân ve với ve của hai phương trình trong h ta được
sin2
x +
1
sin2
x
+
s
sin2
y +
1
+ cos2 y +
1
cos y
+
r
cos2 x +
1
!
= 20
xy
. (3)
(x + y)2
Áp dụng bat đȁng thác Cauchy - Schwarz và AM - GM, ta có
sin2
x +
1
sin2
x
+
s
sin2
y +
1
+ cos2 y +
1
cos y
+
r
cos2 x +
1
!
sin2
y cos2
x
1 2
≥ | sin x. cos x| +
| sin x. cos x|
sin 2x 1 3 2
= + +
2 2| sin 2x| 2| sin 2x|
3 2
5 2
1 + = .
2 2
Hoàn toàn tương tự ta có
s
sin2
y + 1
sin2
y
+
r
cos2 x + 1
cos2 x
5
2
2
Lài giai.
4
4
2
≥
≥ .
57
Tải tài liệu tại sividoc.com
r
≥
sin2
x cos2 x sin2
y cos2 x
4 2 4 2
n n
8m4
− 8m2
+ 1 =
1
a4
+
1
, với m =
1
a +
1
. (1)
un+1 = 8u4
− 8u2
+ 1
2
2 a4 3
2 a42
Xác định so hạng tőng quát vn.
Do đó theo bat đȁng thác AM - GM ta có
V T (3) ≥
4
s
4 sin2
x +
1
. cos2 x +
1
sin2
y +
1
cos2 x +
1
≥ 4
r
4 (
5
)4
2
= 10 20
xy
(x + y)2
= V P (3).
π π
Dau bang xảy ra khi | sin 2x| = 1; x = y ⇔ x = y =
4
+ k
2
, k ∈ Z.
Thả lại, ta thay nghi m này thỏa mãn.
V y nghi m của h phương trình là (x; y) =
π
+ k
π
;
π
+ k
π
; k ∈ Z.
Bài toán 4.9. Cho dãy so (vn)n thỏa mãn đieu ki n
v1 = 5
vn+1 = v4
− 4v2
+ 2
Lài giai. Sả dụng hang đȁng thác sau
Đ t
vn
2
2 a4 2 a
= un ta thu được
u1 = = a + , a = (5 +
5 1 1 1
√
21),
2 2 a 2
(2)
n n
1
Theo (1) thì u = a4
+
1
, u =
1
a42
+
1
,. . .
Bang phương pháp quy nạp, ta thu được
u =
1
a +
1
, a =
1
(5 +
√
21),
2 a 2
u
=
1
a4n−1
+
1
V y vn =
5 +
√
21 4n−1
5 −
√
21 4n−1
2
2
1
n+1
a4n−1
+
58
Tải tài liệu tại sividoc.com
, n = 1, 2, . . .
2
59
Tải tài liệu tại sividoc.com
≤ ≤
— ≤≤ ≤
2
2 2 2 2
3 t
Bài toán 4.10 (VMO - 1984 - Bảng A). Giải phương trình
q
1 +
√
1 − x2
q
(1 + x)3 −
q
(1 − x)3 = 2 +
√
1 − x2.
Lài giai.
Đieu ki n có nghĩa: 1 x 1.
Đ t x = cos t, 0 t π
Phương trình trở thành
√
1 + sin t
q
(1 + cos t)3 −
q
(1 − cos t)3 = 2 + sin t
s
t t
2
3 t √
cos + sin
2 2
cos
2
− sin
2
.2 2 = 2 + sin t
⇔ cos2 t
− sin2 t
1 + cos
t
sin
t
.2
√
2 = 2 + sin t
⇔ cos t(2 + sin t)
√
2 = 2 + sin t ⇔ (2 + sin t)(
√
2 cos t − 1) = 0
⇔ cos t =
1
√
2
⇔ x =
1
1
√
2
.
V y phương trình có nghi m duy nhat x = √
2
.
Bài toán 4.11 (USAMO - 1978). Giả sả x là nghi m của phương trình
(3 + 2
√
2)x
= (
√
2 − 1)x
+ 3.
Cháng minh rang khi đó x cũng là nghi m của phương trình (
√
2 + 1)x
=
π
2 cos .
9
Lài giai. Giả sả x là nghi m của phương trình
(3 + 2
√
2)x
= (
√
2 − 1)x
+ 3 ⇔ (
√
2 + 1)2x
=
Đ t 2t = (
√
2 + 1)x
> 0. Khi đó (1) trở thành
1
(
√
2 + 1)x
+ 3. (1)
4t2
=
1
2t
+ 3 ⇔ 4t3
− 3t =
1
. (2)
⇔
60
Tải tài liệu tại sividoc.com
2 2 9
Trược het ta tìm các nghi m t ∈ (−1; 1) của phương trình (1).
Do t ∈ (−1; 1) nên đ t t = cos α, α ∈ (0; π). Ta có
4 cos3
α − 3 cos α =
1 1
⇔ cos 3α =
π 2π
⇔ α = ± + k .
M t khác α ∈ (0; π) nên α ∈
π
;
5π
;
7π
.
9 9 9
π 5π
cos
Suy ra các nghi m của phương trình (2) là t1 = cos
9
; t2 = cos
7π
.
9
9
; t3 =
π
Ta thay (2) là phương trình b c 3 đã có đủ 3 nghi m t1 = cos
9
; t2 =
cos
5π
9
; t3 = cos
7π
9
trong (−1; 1) nên ta không can xét các nghi m ở ngoài
khoảng (−1; 1) nǎa. Do t2 = cos
5π
9
; t3 = cos
7π
không thỏa mãn đieu kiên
9
t > 0 nên chỉ có t1
√
thỏa mãn nên (
√
2 + 1)x
= 2cos
π
9 hay x là nghi m của
phương trình ( 2 + 1)x
= 2cos
π
9
(đpcm).
Chú ý 4.1. Ngoài ra ta cũng có the sả dụng công thác nhân đôi, nhân ba
xây dựng m®t so h phương trình mà ta dùng phép the lương giác đe giải.
3
61
Tải tài liệu tại sividoc.com
Ket lu n
Lu n văn “Phương pháp lượng giác giải phương trình đa thác và m®t
so dạng toán liên quan” giải quyet nhǎng van đe sau:
- Lu n văn trình bày m®t so kien thác cơ bản liên quan đen đȁng thác
lượng giác và các đȁng thác đại so liên quan...
- Tiep theo, lu n văn cũng đã trình bày chi tiet ve cách giải phương
trình b c ba, b c bon và m®t so dạng phương trình đa thác b c cao...
- Cuoi cùng, lu n văn trình bày các dạng toán liên quan các phương
trình và h phương trình đưa ve giải phương trình đa thác b c cao.
- M®t phan ket quả của lu n văn này đã được trình bày trong K yeu
h®i thảo khoa hoc tại Hưng Yên (xem [2]).
62
Tải tài liệu tại sividoc.com
Tài li u tham khảo
A Tieng Vi t
[1] Trương Ngoc Đac (2015), M®t so dãy so sinh bới các hàm lượng giác,
K yeu HTKH Buôn Ma Thu®c, 14-15/03/2015, trang 58-64.
[2] Mông Thanh Hang (2017), Phương pháp lương giác giải phương trình
đa thúc b¾c cao, K yeu HTKH Hưng Yên, 25-26/02/2017, trang 117-
130.
[3] Nguyen Văn M u, Phạm Thị Bạch Ngoc (2003), M®t so bài toán chon
loc ve lượng giác, NXB Giáo dục.
[4] Nguyen Văn M u, Đàm Văn Nhỉ (2012), Phương pháp toa đ® trong
hình hoc, NXB ĐHQG Hà N®i.
[5] Vũ Dương Thụy, Nguyen Văn Nho (2001), 40 năm Olympic Toán hoc
quoc te, NXB Giáo dục.
B Tieng Anh
[6] Radulescu T-L.T. , Radulescu V.D. , Andreescu T. (2009), Problems
in Real Analysis: Advanced Calculus on the real axis, Springer Sci-
ences+Business Media.
[7] Sausa Paulo Ney, Silva Jorge- Nume (1998), Berkeley Problems in
Mathematics, Springer.

More Related Content

Similar to Phương Pháp Lư Ng Giác Giải Phương Trình Đa Thức Và M T So Dạng Toán.docx

Bài Toán Cực Trị Với Điều Kiện Ràng Buộc Bất Đẳng Thức, Hệ Bất Đẳng Thức.docx
Bài Toán Cực Trị Với Điều Kiện Ràng Buộc Bất Đẳng Thức, Hệ Bất Đẳng Thức.docxBài Toán Cực Trị Với Điều Kiện Ràng Buộc Bất Đẳng Thức, Hệ Bất Đẳng Thức.docx
Bài Toán Cực Trị Với Điều Kiện Ràng Buộc Bất Đẳng Thức, Hệ Bất Đẳng Thức.docxDV Viết Luận văn luanvanmaster.com ZALO 0973287149
 
MA TRẬN + ĐẶC TẢ + ĐỀ KIỂM TRA GIỮA HỌC KÌ 2 MÔN TOÁN – LỚP 8 KẾT NỐI TRI THỨ...
MA TRẬN + ĐẶC TẢ + ĐỀ KIỂM TRA GIỮA HỌC KÌ 2 MÔN TOÁN – LỚP 8 KẾT NỐI TRI THỨ...MA TRẬN + ĐẶC TẢ + ĐỀ KIỂM TRA GIỮA HỌC KÌ 2 MÔN TOÁN – LỚP 8 KẾT NỐI TRI THỨ...
MA TRẬN + ĐẶC TẢ + ĐỀ KIỂM TRA GIỮA HỌC KÌ 2 MÔN TOÁN – LỚP 8 KẾT NỐI TRI THỨ...Nguyen Thanh Tu Collection
 
Một số dạng toán về đa thức qua các kỳ thi Olympic 6732069.pdf
Một số dạng toán về đa thức qua các kỳ thi Olympic 6732069.pdfMột số dạng toán về đa thức qua các kỳ thi Olympic 6732069.pdf
Một số dạng toán về đa thức qua các kỳ thi Olympic 6732069.pdfTieuNgocLy
 
Tich phan %28 nguyen duy khoi%29
Tich phan %28 nguyen duy khoi%29Tich phan %28 nguyen duy khoi%29
Tich phan %28 nguyen duy khoi%29trongphuckhtn
 
Tich phan (nguyen duy khoi)
Tich phan (nguyen duy khoi)Tich phan (nguyen duy khoi)
Tich phan (nguyen duy khoi)roggerbob
 
Tổng quát về tích phân
Tổng quát về tích phân Tổng quát về tích phân
Tổng quát về tích phân Hoàng Hải Huy
 
Chuyên đề Đẳng Thức và Bất đẳng thức - Bồi dưỡng HSG môn Toán lớp 9
Chuyên đề Đẳng Thức và Bất đẳng thức - Bồi dưỡng HSG môn Toán lớp 9Chuyên đề Đẳng Thức và Bất đẳng thức - Bồi dưỡng HSG môn Toán lớp 9
Chuyên đề Đẳng Thức và Bất đẳng thức - Bồi dưỡng HSG môn Toán lớp 9BOIDUONGTOAN.COM
 
Tai lieu on thi tn thpt mon toan www.mathvn.com
Tai lieu on thi tn thpt mon toan   www.mathvn.comTai lieu on thi tn thpt mon toan   www.mathvn.com
Tai lieu on thi tn thpt mon toan www.mathvn.comtrongphuckhtn
 
bo-de-tham-khao-giua-hoc-ky-2-toan-8-nam-2023-2024-phong-gddt-tp-hai-duong.pdf
bo-de-tham-khao-giua-hoc-ky-2-toan-8-nam-2023-2024-phong-gddt-tp-hai-duong.pdfbo-de-tham-khao-giua-hoc-ky-2-toan-8-nam-2023-2024-phong-gddt-tp-hai-duong.pdf
bo-de-tham-khao-giua-hoc-ky-2-toan-8-nam-2023-2024-phong-gddt-tp-hai-duong.pdfLinhTrnTh14
 

Similar to Phương Pháp Lư Ng Giác Giải Phương Trình Đa Thức Và M T So Dạng Toán.docx (20)

Hàm Đơn Đi U, Tựa Đơn Đi U Và M T So Ứng Dụng Của Phép Đơn Đi U Hóa Hàm So.docx
Hàm Đơn Đi U, Tựa Đơn Đi U Và M T So Ứng Dụng Của Phép Đơn Đi U Hóa Hàm So.docxHàm Đơn Đi U, Tựa Đơn Đi U Và M T So Ứng Dụng Của Phép Đơn Đi U Hóa Hàm So.docx
Hàm Đơn Đi U, Tựa Đơn Đi U Và M T So Ứng Dụng Của Phép Đơn Đi U Hóa Hàm So.docx
 
Bài Toán Cực Trị Với Điều Kiện Ràng Buộc Bất Đẳng Thức, Hệ Bất Đẳng Thức.docx
Bài Toán Cực Trị Với Điều Kiện Ràng Buộc Bất Đẳng Thức, Hệ Bất Đẳng Thức.docxBài Toán Cực Trị Với Điều Kiện Ràng Buộc Bất Đẳng Thức, Hệ Bất Đẳng Thức.docx
Bài Toán Cực Trị Với Điều Kiện Ràng Buộc Bất Đẳng Thức, Hệ Bất Đẳng Thức.docx
 
M T So Phương Trình Diophant Liên Quan Đen So Cân Bang.docx
M T So Phương Trình Diophant Liên Quan Đen So Cân Bang.docxM T So Phương Trình Diophant Liên Quan Đen So Cân Bang.docx
M T So Phương Trình Diophant Liên Quan Đen So Cân Bang.docx
 
Luận văn: Phép biến đổi phân tuyến tính, HAY, 9đ
Luận văn: Phép biến đổi phân tuyến tính, HAY, 9đLuận văn: Phép biến đổi phân tuyến tính, HAY, 9đ
Luận văn: Phép biến đổi phân tuyến tính, HAY, 9đ
 
MA TRẬN + ĐẶC TẢ + ĐỀ KIỂM TRA GIỮA HỌC KÌ 2 MÔN TOÁN – LỚP 8 KẾT NỐI TRI THỨ...
MA TRẬN + ĐẶC TẢ + ĐỀ KIỂM TRA GIỮA HỌC KÌ 2 MÔN TOÁN – LỚP 8 KẾT NỐI TRI THỨ...MA TRẬN + ĐẶC TẢ + ĐỀ KIỂM TRA GIỮA HỌC KÌ 2 MÔN TOÁN – LỚP 8 KẾT NỐI TRI THỨ...
MA TRẬN + ĐẶC TẢ + ĐỀ KIỂM TRA GIỮA HỌC KÌ 2 MÔN TOÁN – LỚP 8 KẾT NỐI TRI THỨ...
 
Một số dạng toán về đa thức qua các kỳ thi Olympic 6732069.pdf
Một số dạng toán về đa thức qua các kỳ thi Olympic 6732069.pdfMột số dạng toán về đa thức qua các kỳ thi Olympic 6732069.pdf
Một số dạng toán về đa thức qua các kỳ thi Olympic 6732069.pdf
 
M T So Dạng Toán Cực Tr± Trong L P Hàm Mũ Và Hàm Hyperbolic.docx
M T So Dạng Toán Cực Tr± Trong L P Hàm Mũ Và Hàm Hyperbolic.docxM T So Dạng Toán Cực Tr± Trong L P Hàm Mũ Và Hàm Hyperbolic.docx
M T So Dạng Toán Cực Tr± Trong L P Hàm Mũ Và Hàm Hyperbolic.docx
 
Tich phan %28 nguyen duy khoi%29
Tich phan %28 nguyen duy khoi%29Tich phan %28 nguyen duy khoi%29
Tich phan %28 nguyen duy khoi%29
 
Tich phan (nguyen duy khoi)
Tich phan (nguyen duy khoi)Tich phan (nguyen duy khoi)
Tich phan (nguyen duy khoi)
 
Nghiệm Yếu Của Bài Toán Biên Dirichlet Chứa Toán Tử Laplace Phân Thứ.docx
Nghiệm Yếu Của Bài Toán Biên Dirichlet Chứa Toán Tử Laplace Phân Thứ.docxNghiệm Yếu Của Bài Toán Biên Dirichlet Chứa Toán Tử Laplace Phân Thứ.docx
Nghiệm Yếu Của Bài Toán Biên Dirichlet Chứa Toán Tử Laplace Phân Thứ.docx
 
Luận văn: Định lý bézout và chiều ngược lại, HAY, 9đ
Luận văn: Định lý bézout và chiều ngược lại, HAY, 9đLuận văn: Định lý bézout và chiều ngược lại, HAY, 9đ
Luận văn: Định lý bézout và chiều ngược lại, HAY, 9đ
 
Bat Đang Thức V I Hàm Loi B Ph N Và Ứng Dụng.docx
Bat Đang Thức V I Hàm Loi B Ph N Và Ứng Dụng.docxBat Đang Thức V I Hàm Loi B Ph N Và Ứng Dụng.docx
Bat Đang Thức V I Hàm Loi B Ph N Và Ứng Dụng.docx
 
M T So Ứng Dụng Của Công Thức N I Suy Lagrange Và Hermite.docx
M T So Ứng Dụng Của Công Thức N I Suy Lagrange Và Hermite.docxM T So Ứng Dụng Của Công Thức N I Suy Lagrange Và Hermite.docx
M T So Ứng Dụng Của Công Thức N I Suy Lagrange Và Hermite.docx
 
Tổng quát về tích phân
Tổng quát về tích phân Tổng quát về tích phân
Tổng quát về tích phân
 
Chuyên đề Đẳng Thức và Bất đẳng thức - Bồi dưỡng HSG môn Toán lớp 9
Chuyên đề Đẳng Thức và Bất đẳng thức - Bồi dưỡng HSG môn Toán lớp 9Chuyên đề Đẳng Thức và Bất đẳng thức - Bồi dưỡng HSG môn Toán lớp 9
Chuyên đề Đẳng Thức và Bất đẳng thức - Bồi dưỡng HSG môn Toán lớp 9
 
Ve Ho Chuan Tac Các Hàm Phân Hình.docx
Ve Ho Chuan Tac Các Hàm Phân Hình.docxVe Ho Chuan Tac Các Hàm Phân Hình.docx
Ve Ho Chuan Tac Các Hàm Phân Hình.docx
 
Một Số Lớp Bất Đẳng Thức Lượng Giác Kiểu Klamkin Trong Tam Giác.doc
Một Số Lớp Bất Đẳng Thức Lượng Giác Kiểu Klamkin Trong Tam Giác.docMột Số Lớp Bất Đẳng Thức Lượng Giác Kiểu Klamkin Trong Tam Giác.doc
Một Số Lớp Bất Đẳng Thức Lượng Giác Kiểu Klamkin Trong Tam Giác.doc
 
Toán Tử Sai Phân Và Ứng Dụng Vào Giải Toán Sơ Cấp.docx
Toán Tử Sai Phân Và Ứng Dụng Vào Giải Toán Sơ Cấp.docxToán Tử Sai Phân Và Ứng Dụng Vào Giải Toán Sơ Cấp.docx
Toán Tử Sai Phân Và Ứng Dụng Vào Giải Toán Sơ Cấp.docx
 
Tai lieu on thi tn thpt mon toan www.mathvn.com
Tai lieu on thi tn thpt mon toan   www.mathvn.comTai lieu on thi tn thpt mon toan   www.mathvn.com
Tai lieu on thi tn thpt mon toan www.mathvn.com
 
bo-de-tham-khao-giua-hoc-ky-2-toan-8-nam-2023-2024-phong-gddt-tp-hai-duong.pdf
bo-de-tham-khao-giua-hoc-ky-2-toan-8-nam-2023-2024-phong-gddt-tp-hai-duong.pdfbo-de-tham-khao-giua-hoc-ky-2-toan-8-nam-2023-2024-phong-gddt-tp-hai-duong.pdf
bo-de-tham-khao-giua-hoc-ky-2-toan-8-nam-2023-2024-phong-gddt-tp-hai-duong.pdf
 

More from DV Viết Luận văn luanvanmaster.com ZALO 0973287149

More from DV Viết Luận văn luanvanmaster.com ZALO 0973287149 (20)

Ảnh Hưởng Của Marketing Quan Hệ Đến Lòng Trung Thành Của Khách Hàng.Tình Huốn...
Ảnh Hưởng Của Marketing Quan Hệ Đến Lòng Trung Thành Của Khách Hàng.Tình Huốn...Ảnh Hưởng Của Marketing Quan Hệ Đến Lòng Trung Thành Của Khách Hàng.Tình Huốn...
Ảnh Hưởng Của Marketing Quan Hệ Đến Lòng Trung Thành Của Khách Hàng.Tình Huốn...
 
Phát triển nguồn nhân lực tại Uỷ ban nhân dân huyện Trà Bồng, tỉnh Quảng Ngãi...
Phát triển nguồn nhân lực tại Uỷ ban nhân dân huyện Trà Bồng, tỉnh Quảng Ngãi...Phát triển nguồn nhân lực tại Uỷ ban nhân dân huyện Trà Bồng, tỉnh Quảng Ngãi...
Phát triển nguồn nhân lực tại Uỷ ban nhân dân huyện Trà Bồng, tỉnh Quảng Ngãi...
 
Báo cáo tốt Nghiệp tài chính hợp nhất tại tổng công ty Indochina gol...
Báo cáo tốt Nghiệp  tài chính hợp nhất tại tổng công ty Indochina gol...Báo cáo tốt Nghiệp  tài chính hợp nhất tại tổng công ty Indochina gol...
Báo cáo tốt Nghiệp tài chính hợp nhất tại tổng công ty Indochina gol...
 
Tạo động lực thúc đẩy nhân viên làm việc tại ngân hàng TMCP Ngoại Thương Việt...
Tạo động lực thúc đẩy nhân viên làm việc tại ngân hàng TMCP Ngoại Thương Việt...Tạo động lực thúc đẩy nhân viên làm việc tại ngân hàng TMCP Ngoại Thương Việt...
Tạo động lực thúc đẩy nhân viên làm việc tại ngân hàng TMCP Ngoại Thương Việt...
 
Phát triển công nghiệp trên địa bàn Thành phố Tam Kỳ, Tỉnh Quảng Na...
Phát triển công nghiệp trên địa bàn Thành phố Tam Kỳ, Tỉnh Quảng Na...Phát triển công nghiệp trên địa bàn Thành phố Tam Kỳ, Tỉnh Quảng Na...
Phát triển công nghiệp trên địa bàn Thành phố Tam Kỳ, Tỉnh Quảng Na...
 
Giải pháp phát triển cho vay xuất nhập khẩu tại ngân hàng NN&PTNN ch...
Giải pháp phát triển cho vay xuất nhập khẩu tại ngân hàng NN&PTNN ch...Giải pháp phát triển cho vay xuất nhập khẩu tại ngân hàng NN&PTNN ch...
Giải pháp phát triển cho vay xuất nhập khẩu tại ngân hàng NN&PTNN ch...
 
Hoàn thiện công tác lập báo cáo tài chính hợp nhất tại tổng công ...
Hoàn thiện công tác lập báo cáo tài chính hợp nhất tại tổng công ...Hoàn thiện công tác lập báo cáo tài chính hợp nhất tại tổng công ...
Hoàn thiện công tác lập báo cáo tài chính hợp nhất tại tổng công ...
 
Luận Văn Thạc Sĩ Quản trị thành tích nhân viên tại Cục Hải quan TP Đà Nẵng.doc
Luận Văn Thạc Sĩ  Quản trị thành tích nhân viên tại Cục Hải quan TP Đà Nẵng.docLuận Văn Thạc Sĩ  Quản trị thành tích nhân viên tại Cục Hải quan TP Đà Nẵng.doc
Luận Văn Thạc Sĩ Quản trị thành tích nhân viên tại Cục Hải quan TP Đà Nẵng.doc
 
Hoàn thiện công tác quản lý thuế thu nhập cá nhân tại cục thuế Tỉ...
Hoàn thiện công tác quản lý thuế thu nhập cá nhân tại cục thuế Tỉ...Hoàn thiện công tác quản lý thuế thu nhập cá nhân tại cục thuế Tỉ...
Hoàn thiện công tác quản lý thuế thu nhập cá nhân tại cục thuế Tỉ...
 
Đề Tài Phát triển bền vững nông nghiệp Huyện Ba Tơ, Tỉnh Quảng Ngãi....
Đề Tài Phát triển bền vững nông nghiệp Huyện Ba Tơ, Tỉnh Quảng Ngãi....Đề Tài Phát triển bền vững nông nghiệp Huyện Ba Tơ, Tỉnh Quảng Ngãi....
Đề Tài Phát triển bền vững nông nghiệp Huyện Ba Tơ, Tỉnh Quảng Ngãi....
 
Hoàn thiện công tác bảo trợ xã hội trên địa bàn huyện Phong Điền, tỉnh Thừa T...
Hoàn thiện công tác bảo trợ xã hội trên địa bàn huyện Phong Điền, tỉnh Thừa T...Hoàn thiện công tác bảo trợ xã hội trên địa bàn huyện Phong Điền, tỉnh Thừa T...
Hoàn thiện công tác bảo trợ xã hội trên địa bàn huyện Phong Điền, tỉnh Thừa T...
 
Đề Tài Luận VănPhát triển sản phẩm du lịch tại thành phố Đà Nẵng.doc
Đề Tài Luận VănPhát triển sản phẩm du lịch tại thành phố Đà Nẵng.docĐề Tài Luận VănPhát triển sản phẩm du lịch tại thành phố Đà Nẵng.doc
Đề Tài Luận VănPhát triển sản phẩm du lịch tại thành phố Đà Nẵng.doc
 
Đào tạo nghề cho lao động thuộc diện thu hồi đất trên địa bàn Thàn...
Đào tạo nghề cho lao động thuộc diện thu hồi đất trên địa bàn Thàn...Đào tạo nghề cho lao động thuộc diện thu hồi đất trên địa bàn Thàn...
Đào tạo nghề cho lao động thuộc diện thu hồi đất trên địa bàn Thàn...
 
Tóm Tắt Luận Văn Thạc Sĩ Quản Trị Kinh Doanh Xây dựng chính sách Marketing tạ...
Tóm Tắt Luận Văn Thạc Sĩ Quản Trị Kinh Doanh Xây dựng chính sách Marketing tạ...Tóm Tắt Luận Văn Thạc Sĩ Quản Trị Kinh Doanh Xây dựng chính sách Marketing tạ...
Tóm Tắt Luận Văn Thạc Sĩ Quản Trị Kinh Doanh Xây dựng chính sách Marketing tạ...
 
Đề Tài Nghiên cứu rủi ro cảm nhận đối với mua hàng thời trang trực tuyến.docx
Đề Tài Nghiên cứu rủi ro cảm nhận đối với mua hàng thời trang trực tuyến.docxĐề Tài Nghiên cứu rủi ro cảm nhận đối với mua hàng thời trang trực tuyến.docx
Đề Tài Nghiên cứu rủi ro cảm nhận đối với mua hàng thời trang trực tuyến.docx
 
Giải pháp nâng cao động lực thúc đẩy người lao động tại công ty khai...
Giải pháp nâng cao động lực thúc đẩy người lao động tại công ty khai...Giải pháp nâng cao động lực thúc đẩy người lao động tại công ty khai...
Giải pháp nâng cao động lực thúc đẩy người lao động tại công ty khai...
 
Giải pháp phát triển dịch vụ ngân hàng điện tử tại ngân hàng đầu ...
Giải pháp phát triển dịch vụ ngân hàng điện tử tại ngân hàng đầu ...Giải pháp phát triển dịch vụ ngân hàng điện tử tại ngân hàng đầu ...
Giải pháp phát triển dịch vụ ngân hàng điện tử tại ngân hàng đầu ...
 
Giải pháp phát triển dịch vụ ngân hàng điện tử tại ngân hàng đầu ...
Giải pháp phát triển dịch vụ ngân hàng điện tử tại ngân hàng đầu ...Giải pháp phát triển dịch vụ ngân hàng điện tử tại ngân hàng đầu ...
Giải pháp phát triển dịch vụ ngân hàng điện tử tại ngân hàng đầu ...
 
Quản trị quan hệ khách hàng tại Chi nhánh Viettel Đà Nẵng – Tập đoàn Viễn thô...
Quản trị quan hệ khách hàng tại Chi nhánh Viettel Đà Nẵng – Tập đoàn Viễn thô...Quản trị quan hệ khách hàng tại Chi nhánh Viettel Đà Nẵng – Tập đoàn Viễn thô...
Quản trị quan hệ khách hàng tại Chi nhánh Viettel Đà Nẵng – Tập đoàn Viễn thô...
 
Đề Tài Đánh giá thành tích đội ngũ giảng viên trường Đại Học Phạm ...
Đề Tài Đánh giá thành tích đội ngũ giảng viên trường Đại Học Phạm ...Đề Tài Đánh giá thành tích đội ngũ giảng viên trường Đại Học Phạm ...
Đề Tài Đánh giá thành tích đội ngũ giảng viên trường Đại Học Phạm ...
 

Recently uploaded

SÁNG KIẾN “THIẾT KẾ VÀ SỬ DỤNG INFOGRAPHIC TRONG DẠY HỌC ĐỊA LÍ 11 (BỘ SÁCH K...
SÁNG KIẾN “THIẾT KẾ VÀ SỬ DỤNG INFOGRAPHIC TRONG DẠY HỌC ĐỊA LÍ 11 (BỘ SÁCH K...SÁNG KIẾN “THIẾT KẾ VÀ SỬ DỤNG INFOGRAPHIC TRONG DẠY HỌC ĐỊA LÍ 11 (BỘ SÁCH K...
SÁNG KIẾN “THIẾT KẾ VÀ SỬ DỤNG INFOGRAPHIC TRONG DẠY HỌC ĐỊA LÍ 11 (BỘ SÁCH K...Nguyen Thanh Tu Collection
 
BỘ ĐỀ PHÁT TRIỂN THEO CẤU TRÚC ĐỀ MINH HỌA BGD NGÀY 22-3-2024 KỲ THI TỐT NGHI...
BỘ ĐỀ PHÁT TRIỂN THEO CẤU TRÚC ĐỀ MINH HỌA BGD NGÀY 22-3-2024 KỲ THI TỐT NGHI...BỘ ĐỀ PHÁT TRIỂN THEO CẤU TRÚC ĐỀ MINH HỌA BGD NGÀY 22-3-2024 KỲ THI TỐT NGHI...
BỘ ĐỀ PHÁT TRIỂN THEO CẤU TRÚC ĐỀ MINH HỌA BGD NGÀY 22-3-2024 KỲ THI TỐT NGHI...Nguyen Thanh Tu Collection
 
TỔNG HỢP ĐỀ THI CHÍNH THỨC KỲ THI TUYỂN SINH VÀO LỚP 10 THPT MÔN NGỮ VĂN NĂM ...
TỔNG HỢP ĐỀ THI CHÍNH THỨC KỲ THI TUYỂN SINH VÀO LỚP 10 THPT MÔN NGỮ VĂN NĂM ...TỔNG HỢP ĐỀ THI CHÍNH THỨC KỲ THI TUYỂN SINH VÀO LỚP 10 THPT MÔN NGỮ VĂN NĂM ...
TỔNG HỢP ĐỀ THI CHÍNH THỨC KỲ THI TUYỂN SINH VÀO LỚP 10 THPT MÔN NGỮ VĂN NĂM ...Nguyen Thanh Tu Collection
 
NQA Lợi ích Từ ISO và ESG Tăng Trưởng và Bền Vững ver01.pdf
NQA Lợi ích Từ ISO và ESG Tăng Trưởng và Bền Vững ver01.pdfNQA Lợi ích Từ ISO và ESG Tăng Trưởng và Bền Vững ver01.pdf
NQA Lợi ích Từ ISO và ESG Tăng Trưởng và Bền Vững ver01.pdfNguyễn Đăng Quang
 
Chuong trinh dao tao Su pham Khoa hoc tu nhien, ma nganh - 7140247.pdf
Chuong trinh dao tao Su pham Khoa hoc tu nhien, ma nganh - 7140247.pdfChuong trinh dao tao Su pham Khoa hoc tu nhien, ma nganh - 7140247.pdf
Chuong trinh dao tao Su pham Khoa hoc tu nhien, ma nganh - 7140247.pdfhoangtuansinh1
 
Chàm - Bệnh án (da liễu - bvdlct ctump) .pptx
Chàm - Bệnh án (da liễu - bvdlct ctump) .pptxChàm - Bệnh án (da liễu - bvdlct ctump) .pptx
Chàm - Bệnh án (da liễu - bvdlct ctump) .pptxendkay31
 
10 ĐỀ KIỂM TRA + 6 ĐỀ ÔN TẬP CUỐI KÌ 2 VẬT LÝ 11 - KẾT NỐI TRI THỨC - THEO C...
10 ĐỀ KIỂM TRA + 6 ĐỀ ÔN TẬP CUỐI KÌ 2 VẬT LÝ 11 - KẾT NỐI TRI THỨC - THEO C...10 ĐỀ KIỂM TRA + 6 ĐỀ ÔN TẬP CUỐI KÌ 2 VẬT LÝ 11 - KẾT NỐI TRI THỨC - THEO C...
10 ĐỀ KIỂM TRA + 6 ĐỀ ÔN TẬP CUỐI KÌ 2 VẬT LÝ 11 - KẾT NỐI TRI THỨC - THEO C...Nguyen Thanh Tu Collection
 
30 ĐỀ PHÁT TRIỂN THEO CẤU TRÚC ĐỀ MINH HỌA BGD NGÀY 22-3-2024 KỲ THI TỐT NGHI...
30 ĐỀ PHÁT TRIỂN THEO CẤU TRÚC ĐỀ MINH HỌA BGD NGÀY 22-3-2024 KỲ THI TỐT NGHI...30 ĐỀ PHÁT TRIỂN THEO CẤU TRÚC ĐỀ MINH HỌA BGD NGÀY 22-3-2024 KỲ THI TỐT NGHI...
30 ĐỀ PHÁT TRIỂN THEO CẤU TRÚC ĐỀ MINH HỌA BGD NGÀY 22-3-2024 KỲ THI TỐT NGHI...Nguyen Thanh Tu Collection
 
Thong bao 337-DHPY (24.4.2024) thi sat hach Ngoai ngu dap ung Chuan dau ra do...
Thong bao 337-DHPY (24.4.2024) thi sat hach Ngoai ngu dap ung Chuan dau ra do...Thong bao 337-DHPY (24.4.2024) thi sat hach Ngoai ngu dap ung Chuan dau ra do...
Thong bao 337-DHPY (24.4.2024) thi sat hach Ngoai ngu dap ung Chuan dau ra do...hoangtuansinh1
 
Sáng kiến “Sử dụng ứng dụng Quizizz nhằm nâng cao chất lượng ôn thi tốt nghiệ...
Sáng kiến “Sử dụng ứng dụng Quizizz nhằm nâng cao chất lượng ôn thi tốt nghiệ...Sáng kiến “Sử dụng ứng dụng Quizizz nhằm nâng cao chất lượng ôn thi tốt nghiệ...
Sáng kiến “Sử dụng ứng dụng Quizizz nhằm nâng cao chất lượng ôn thi tốt nghiệ...Nguyen Thanh Tu Collection
 
BỘ ĐỀ KIỂM TRA CUỐI KÌ 2 VẬT LÝ 11 - KẾT NỐI TRI THỨC - THEO CẤU TRÚC ĐỀ MIN...
BỘ ĐỀ KIỂM TRA CUỐI KÌ 2 VẬT LÝ 11 - KẾT NỐI TRI THỨC - THEO CẤU TRÚC ĐỀ MIN...BỘ ĐỀ KIỂM TRA CUỐI KÌ 2 VẬT LÝ 11 - KẾT NỐI TRI THỨC - THEO CẤU TRÚC ĐỀ MIN...
BỘ ĐỀ KIỂM TRA CUỐI KÌ 2 VẬT LÝ 11 - KẾT NỐI TRI THỨC - THEO CẤU TRÚC ĐỀ MIN...Nguyen Thanh Tu Collection
 
Sáng kiến Dạy học theo định hướng STEM một số chủ đề phần “vật sống”, Khoa họ...
Sáng kiến Dạy học theo định hướng STEM một số chủ đề phần “vật sống”, Khoa họ...Sáng kiến Dạy học theo định hướng STEM một số chủ đề phần “vật sống”, Khoa họ...
Sáng kiến Dạy học theo định hướng STEM một số chủ đề phần “vật sống”, Khoa họ...Nguyen Thanh Tu Collection
 
QUẢN LÝ HOẠT ĐỘNG GIÁO DỤC KỸ NĂNG SỐNG CHO HỌC SINH CÁC TRƯỜNG TRUNG HỌC CƠ ...
QUẢN LÝ HOẠT ĐỘNG GIÁO DỤC KỸ NĂNG SỐNG CHO HỌC SINH CÁC TRƯỜNG TRUNG HỌC CƠ ...QUẢN LÝ HOẠT ĐỘNG GIÁO DỤC KỸ NĂNG SỐNG CHO HỌC SINH CÁC TRƯỜNG TRUNG HỌC CƠ ...
QUẢN LÝ HOẠT ĐỘNG GIÁO DỤC KỸ NĂNG SỐNG CHO HỌC SINH CÁC TRƯỜNG TRUNG HỌC CƠ ...ThunTrn734461
 
Sơ đồ tư duy môn sinh học bậc THPT.pdf
Sơ đồ tư duy môn sinh học bậc THPT.pdfSơ đồ tư duy môn sinh học bậc THPT.pdf
Sơ đồ tư duy môn sinh học bậc THPT.pdftohoanggiabao81
 
30 ĐỀ PHÁT TRIỂN THEO CẤU TRÚC ĐỀ MINH HỌA BGD NGÀY 22-3-2024 KỲ THI TỐT NGHI...
30 ĐỀ PHÁT TRIỂN THEO CẤU TRÚC ĐỀ MINH HỌA BGD NGÀY 22-3-2024 KỲ THI TỐT NGHI...30 ĐỀ PHÁT TRIỂN THEO CẤU TRÚC ĐỀ MINH HỌA BGD NGÀY 22-3-2024 KỲ THI TỐT NGHI...
30 ĐỀ PHÁT TRIỂN THEO CẤU TRÚC ĐỀ MINH HỌA BGD NGÀY 22-3-2024 KỲ THI TỐT NGHI...Nguyen Thanh Tu Collection
 
ôn tập lịch sử hhhhhhhhhhhhhhhhhhhhhhhhhh
ôn tập lịch sử hhhhhhhhhhhhhhhhhhhhhhhhhhôn tập lịch sử hhhhhhhhhhhhhhhhhhhhhhhhhh
ôn tập lịch sử hhhhhhhhhhhhhhhhhhhhhhhhhhvanhathvc
 
Kiểm tra chạy trạm lí thuyết giữa kì giải phẫu sinh lí
Kiểm tra chạy trạm lí thuyết giữa kì giải phẫu sinh líKiểm tra chạy trạm lí thuyết giữa kì giải phẫu sinh lí
Kiểm tra chạy trạm lí thuyết giữa kì giải phẫu sinh líDr K-OGN
 
bài 5.1.docx Sinh học di truyền đại cương năm nhất của học sinh y đa khoa
bài 5.1.docx Sinh học di truyền đại cương năm nhất của học sinh y đa khoabài 5.1.docx Sinh học di truyền đại cương năm nhất của học sinh y đa khoa
bài 5.1.docx Sinh học di truyền đại cương năm nhất của học sinh y đa khoa2353020138
 
Trích dẫn trắc nghiệm tư tưởng HCM5.docx
Trích dẫn trắc nghiệm tư tưởng HCM5.docxTrích dẫn trắc nghiệm tư tưởng HCM5.docx
Trích dẫn trắc nghiệm tư tưởng HCM5.docxnhungdt08102004
 

Recently uploaded (19)

SÁNG KIẾN “THIẾT KẾ VÀ SỬ DỤNG INFOGRAPHIC TRONG DẠY HỌC ĐỊA LÍ 11 (BỘ SÁCH K...
SÁNG KIẾN “THIẾT KẾ VÀ SỬ DỤNG INFOGRAPHIC TRONG DẠY HỌC ĐỊA LÍ 11 (BỘ SÁCH K...SÁNG KIẾN “THIẾT KẾ VÀ SỬ DỤNG INFOGRAPHIC TRONG DẠY HỌC ĐỊA LÍ 11 (BỘ SÁCH K...
SÁNG KIẾN “THIẾT KẾ VÀ SỬ DỤNG INFOGRAPHIC TRONG DẠY HỌC ĐỊA LÍ 11 (BỘ SÁCH K...
 
BỘ ĐỀ PHÁT TRIỂN THEO CẤU TRÚC ĐỀ MINH HỌA BGD NGÀY 22-3-2024 KỲ THI TỐT NGHI...
BỘ ĐỀ PHÁT TRIỂN THEO CẤU TRÚC ĐỀ MINH HỌA BGD NGÀY 22-3-2024 KỲ THI TỐT NGHI...BỘ ĐỀ PHÁT TRIỂN THEO CẤU TRÚC ĐỀ MINH HỌA BGD NGÀY 22-3-2024 KỲ THI TỐT NGHI...
BỘ ĐỀ PHÁT TRIỂN THEO CẤU TRÚC ĐỀ MINH HỌA BGD NGÀY 22-3-2024 KỲ THI TỐT NGHI...
 
TỔNG HỢP ĐỀ THI CHÍNH THỨC KỲ THI TUYỂN SINH VÀO LỚP 10 THPT MÔN NGỮ VĂN NĂM ...
TỔNG HỢP ĐỀ THI CHÍNH THỨC KỲ THI TUYỂN SINH VÀO LỚP 10 THPT MÔN NGỮ VĂN NĂM ...TỔNG HỢP ĐỀ THI CHÍNH THỨC KỲ THI TUYỂN SINH VÀO LỚP 10 THPT MÔN NGỮ VĂN NĂM ...
TỔNG HỢP ĐỀ THI CHÍNH THỨC KỲ THI TUYỂN SINH VÀO LỚP 10 THPT MÔN NGỮ VĂN NĂM ...
 
NQA Lợi ích Từ ISO và ESG Tăng Trưởng và Bền Vững ver01.pdf
NQA Lợi ích Từ ISO và ESG Tăng Trưởng và Bền Vững ver01.pdfNQA Lợi ích Từ ISO và ESG Tăng Trưởng và Bền Vững ver01.pdf
NQA Lợi ích Từ ISO và ESG Tăng Trưởng và Bền Vững ver01.pdf
 
Chuong trinh dao tao Su pham Khoa hoc tu nhien, ma nganh - 7140247.pdf
Chuong trinh dao tao Su pham Khoa hoc tu nhien, ma nganh - 7140247.pdfChuong trinh dao tao Su pham Khoa hoc tu nhien, ma nganh - 7140247.pdf
Chuong trinh dao tao Su pham Khoa hoc tu nhien, ma nganh - 7140247.pdf
 
Chàm - Bệnh án (da liễu - bvdlct ctump) .pptx
Chàm - Bệnh án (da liễu - bvdlct ctump) .pptxChàm - Bệnh án (da liễu - bvdlct ctump) .pptx
Chàm - Bệnh án (da liễu - bvdlct ctump) .pptx
 
10 ĐỀ KIỂM TRA + 6 ĐỀ ÔN TẬP CUỐI KÌ 2 VẬT LÝ 11 - KẾT NỐI TRI THỨC - THEO C...
10 ĐỀ KIỂM TRA + 6 ĐỀ ÔN TẬP CUỐI KÌ 2 VẬT LÝ 11 - KẾT NỐI TRI THỨC - THEO C...10 ĐỀ KIỂM TRA + 6 ĐỀ ÔN TẬP CUỐI KÌ 2 VẬT LÝ 11 - KẾT NỐI TRI THỨC - THEO C...
10 ĐỀ KIỂM TRA + 6 ĐỀ ÔN TẬP CUỐI KÌ 2 VẬT LÝ 11 - KẾT NỐI TRI THỨC - THEO C...
 
30 ĐỀ PHÁT TRIỂN THEO CẤU TRÚC ĐỀ MINH HỌA BGD NGÀY 22-3-2024 KỲ THI TỐT NGHI...
30 ĐỀ PHÁT TRIỂN THEO CẤU TRÚC ĐỀ MINH HỌA BGD NGÀY 22-3-2024 KỲ THI TỐT NGHI...30 ĐỀ PHÁT TRIỂN THEO CẤU TRÚC ĐỀ MINH HỌA BGD NGÀY 22-3-2024 KỲ THI TỐT NGHI...
30 ĐỀ PHÁT TRIỂN THEO CẤU TRÚC ĐỀ MINH HỌA BGD NGÀY 22-3-2024 KỲ THI TỐT NGHI...
 
Thong bao 337-DHPY (24.4.2024) thi sat hach Ngoai ngu dap ung Chuan dau ra do...
Thong bao 337-DHPY (24.4.2024) thi sat hach Ngoai ngu dap ung Chuan dau ra do...Thong bao 337-DHPY (24.4.2024) thi sat hach Ngoai ngu dap ung Chuan dau ra do...
Thong bao 337-DHPY (24.4.2024) thi sat hach Ngoai ngu dap ung Chuan dau ra do...
 
Sáng kiến “Sử dụng ứng dụng Quizizz nhằm nâng cao chất lượng ôn thi tốt nghiệ...
Sáng kiến “Sử dụng ứng dụng Quizizz nhằm nâng cao chất lượng ôn thi tốt nghiệ...Sáng kiến “Sử dụng ứng dụng Quizizz nhằm nâng cao chất lượng ôn thi tốt nghiệ...
Sáng kiến “Sử dụng ứng dụng Quizizz nhằm nâng cao chất lượng ôn thi tốt nghiệ...
 
BỘ ĐỀ KIỂM TRA CUỐI KÌ 2 VẬT LÝ 11 - KẾT NỐI TRI THỨC - THEO CẤU TRÚC ĐỀ MIN...
BỘ ĐỀ KIỂM TRA CUỐI KÌ 2 VẬT LÝ 11 - KẾT NỐI TRI THỨC - THEO CẤU TRÚC ĐỀ MIN...BỘ ĐỀ KIỂM TRA CUỐI KÌ 2 VẬT LÝ 11 - KẾT NỐI TRI THỨC - THEO CẤU TRÚC ĐỀ MIN...
BỘ ĐỀ KIỂM TRA CUỐI KÌ 2 VẬT LÝ 11 - KẾT NỐI TRI THỨC - THEO CẤU TRÚC ĐỀ MIN...
 
Sáng kiến Dạy học theo định hướng STEM một số chủ đề phần “vật sống”, Khoa họ...
Sáng kiến Dạy học theo định hướng STEM một số chủ đề phần “vật sống”, Khoa họ...Sáng kiến Dạy học theo định hướng STEM một số chủ đề phần “vật sống”, Khoa họ...
Sáng kiến Dạy học theo định hướng STEM một số chủ đề phần “vật sống”, Khoa họ...
 
QUẢN LÝ HOẠT ĐỘNG GIÁO DỤC KỸ NĂNG SỐNG CHO HỌC SINH CÁC TRƯỜNG TRUNG HỌC CƠ ...
QUẢN LÝ HOẠT ĐỘNG GIÁO DỤC KỸ NĂNG SỐNG CHO HỌC SINH CÁC TRƯỜNG TRUNG HỌC CƠ ...QUẢN LÝ HOẠT ĐỘNG GIÁO DỤC KỸ NĂNG SỐNG CHO HỌC SINH CÁC TRƯỜNG TRUNG HỌC CƠ ...
QUẢN LÝ HOẠT ĐỘNG GIÁO DỤC KỸ NĂNG SỐNG CHO HỌC SINH CÁC TRƯỜNG TRUNG HỌC CƠ ...
 
Sơ đồ tư duy môn sinh học bậc THPT.pdf
Sơ đồ tư duy môn sinh học bậc THPT.pdfSơ đồ tư duy môn sinh học bậc THPT.pdf
Sơ đồ tư duy môn sinh học bậc THPT.pdf
 
30 ĐỀ PHÁT TRIỂN THEO CẤU TRÚC ĐỀ MINH HỌA BGD NGÀY 22-3-2024 KỲ THI TỐT NGHI...
30 ĐỀ PHÁT TRIỂN THEO CẤU TRÚC ĐỀ MINH HỌA BGD NGÀY 22-3-2024 KỲ THI TỐT NGHI...30 ĐỀ PHÁT TRIỂN THEO CẤU TRÚC ĐỀ MINH HỌA BGD NGÀY 22-3-2024 KỲ THI TỐT NGHI...
30 ĐỀ PHÁT TRIỂN THEO CẤU TRÚC ĐỀ MINH HỌA BGD NGÀY 22-3-2024 KỲ THI TỐT NGHI...
 
ôn tập lịch sử hhhhhhhhhhhhhhhhhhhhhhhhhh
ôn tập lịch sử hhhhhhhhhhhhhhhhhhhhhhhhhhôn tập lịch sử hhhhhhhhhhhhhhhhhhhhhhhhhh
ôn tập lịch sử hhhhhhhhhhhhhhhhhhhhhhhhhh
 
Kiểm tra chạy trạm lí thuyết giữa kì giải phẫu sinh lí
Kiểm tra chạy trạm lí thuyết giữa kì giải phẫu sinh líKiểm tra chạy trạm lí thuyết giữa kì giải phẫu sinh lí
Kiểm tra chạy trạm lí thuyết giữa kì giải phẫu sinh lí
 
bài 5.1.docx Sinh học di truyền đại cương năm nhất của học sinh y đa khoa
bài 5.1.docx Sinh học di truyền đại cương năm nhất của học sinh y đa khoabài 5.1.docx Sinh học di truyền đại cương năm nhất của học sinh y đa khoa
bài 5.1.docx Sinh học di truyền đại cương năm nhất của học sinh y đa khoa
 
Trích dẫn trắc nghiệm tư tưởng HCM5.docx
Trích dẫn trắc nghiệm tư tưởng HCM5.docxTrích dẫn trắc nghiệm tư tưởng HCM5.docx
Trích dẫn trắc nghiệm tư tưởng HCM5.docx
 

Phương Pháp Lư Ng Giác Giải Phương Trình Đa Thức Và M T So Dạng Toán.docx

  • 1. THÁI NGUYÊN, 06/2017 Tải tài liệu tại sividoc.com Viết đề tài giá sinh viên – ZALO:0973.287.149-TEAMLUANVAN.COM ĐẠI HOC THÁI NGUYÊN TRƯ NG ĐẠI HOC KHOA HOC ——————–o0o——————– MÔNG THANH HANG PHƯƠNG PHÁP LƯ NG GIÁC GIẢI PHƯƠNG TRÌNH ĐA THỨC VÀ M T SO DẠNG TOÁN LU N VĂN THẠC SỸ TOÁN HOC Chuyên nghành: PHƯƠNG PHÁP TOÁN SƠ CAP Mã so 60 46 01 13
  • 2. THÁI NGUYÊN, 06/2017 Tải tài liệu tại sividoc.com Viết đề tài giá sinh viên – ZALO:0973.287.149-TEAMLUANVAN.COM ĐẠI HOC THÁI NGUYÊN TRƯ NG ĐẠI HOC KHOA HOC ——————–o0o——————– MÔNG THANH HANG PHƯƠNG PHÁP LƯ NG GIÁC GIẢI PHƯƠNG TRÌNH ĐA THỨC VÀ M T SO DẠNG TOÁN LU N VĂN THẠC SỸ TOÁN HOC Chuyên nghành: PHƯƠNG PHÁP TOÁN SƠ CAP Mã so 60 46 01 13 Ngư i hư ng dȁn khoa hoc GS. TS. LÊ TH± THANH NHÀN
  • 3. 1 Tải tài liệu tại sividoc.com Mnc lnc M đau . . . . . . . . . . . . . . . . . . . . . . . . . . . . . 2 Chương 1. M t so đang thfíc lư ng giác và đang thfíc đại so sinh b i h thfíc lư ng giác 4 1.1 M®t so tính chat của đa thác lượng giác . . . . . . . . . . 4 1.2 M®t so đong nhat thác dạng đại so - lượng giác . . . . . . . 9 1.3 Đa thác Chebyshev...................................................................... 17 1.3.1 Các định nghĩa................................................................. 17 1.3.2 Tính chat của các đa thác Chebyshev............................. 17 Chương 2. Phương pháp lư ng giác giải phương trình b c ba và b c bon 20 2.1 Giải phương trình b c ba ..........................................................20 2.1.1 Giải và bi n lu n phương trình b c ba ...........................20 2.1.2 Phương trình b c ba nh n các yeu to trong tam giác là nghi m........................................................................28 2.2 Giải phương trình b c bon ........................................................32 2.3 M®t so h phương trình đưa ve phương trình b c ba và b c bon......................................................................................................37 Chương 3. Phương pháp lư ng giác giải phương trình đa thfíc b c cao 39 3.1 Phương trình đa thác b c cao ...................................................39 3.2 H phương trình đa thác b c cao ..............................................49 Chương 4. M t so dạng toán liên quan 51 4.1 Phép the lượng giác..........................................................................51 4.1.1 Phép the lượng giác trong bat đȁng thác........................ 51 4.1.2 Phép the lượng giác trong dãy so.......................................53 4.2 M®t so dạng toán tà các đe thi Olympic sả dụng phương pháp lượng giác.................................................................................55 Ket lu n............................................................................. 60 Tài li u tham khảo............................................................ 61
  • 4. 2 Tải tài liệu tại sividoc.com M đau Các chuyên đe đa thác và lượng giác và nhǎng van đe liên quan là m®t phan quan trong của đại so và giải tích toán hoc. Các hoc sinh thường phải đoi m t với nhieu dạng toán loại khó liên quan đen hai chuyên đe này. Các dạng toán ve phương trình đa thác luôn luôn xuat hi n trong chương trình toán tà b c THCS đen THPT... Trong hau het các kỳ thi hoc sinh giỏi quoc gia, Olympic Toán khu vực và quoc te, Olympic sinh viên giǎa các trường đại hoc và cao đȁng, các bài toán liên quan đen đa thác rat hay được đe c p và thu®c loại khó và rat khó. Các bài toán ve khảo sát phương trình và bat phương trình đa thác bang phương pháp lượng giác là m®t dạng chuyên đe chon loc can thiet cho giáo viên và hoc sinh b c trung hoc phő thông và năm đau b c đại hoc. Sả dụng lượng giác ta có the thiet l p được nhieu đong nhat thác đại so mới, đe tà đó cho phép giải các phương trình b c ba, b c bon và m®t so dạng phương trình đa thác b c cao với h so thực m®t cách trực tiep, không can vi n trợ đen so phác. Chính vì v y, và cũng đe đáp áng cho nhu cau giảng dạy và hoc t p, tác giả chon đe tài lu n văn ve "Phương pháp lượng giác giải phương trình đa thác và m®t so dạng toán". Đây là chuyên đe có ý nghĩa thực tien trong công vi c giảng dạy, nó cho ta sự nhìn nh n nhat quán ve các bài toán giải và bi n lu n phương trình đa thác và các dạng toán liên quan đen bat đȁng thác và cực trị m®t so lớp đa thác m®t bien. Cau trúc lu n văn gom 4 chương: Chương 1. M®t so đȁng thác lượng giác và đȁng thác đại so sinh bởi h thác lượng giác. Chương 2. Phương pháp lượng giác giải phương trình b c ba và b c bon. Chương 3. Phương pháp lượng giác giải phương trình b c cao. Chương 4. M®t so dạng toán liên quan. M®t so dạng ví dụ và bài t p được chon loc là các đe ra của các kỳ thi
  • 5. 3 Tải tài liệu tại sividoc.com hoc sinh giỏi quoc gia và Olympic quoc te. M®t so các bài toán minh hoạ khác được trích tà các tài li u tham khảo [1-5]. Tôi xin bày tỏ lòng cảm ơn sâu sac tới GS.TS. Lê Thị Thanh Nhàn người thay đã trực tiep hướng dan và giúp đơ đe tác giả hoàn thành bản lu n văn này. Tôi xin chân thành cảm ơn các thay, cô giáo trong khoa Toán - Tin, phòng Đào tạo trường Đại hoc Khoa hoc - Đại hoc Thái Nguyên, Trường THPT Sơn Dương, huy n Sơn Dương, tỉnh Tuyên Quang và bạn bè đong nghi p đã giúp đơ tạo đieu ki n cho tôi hoàn thành bản lu n văn này. Thái Nguyên, 01 tháng 05 năm 2017 Mông Thanh Hang
  • 6. 4 Tải tài liệu tại sividoc.com Σ ∈ { } ∈ { } ≤ { } Chương 1. M t so đang thfíc lư ng giác và đang thfíc đại so sinh b i h thfíc lư ng giác 1.1 M t so tính chat của đa thfíc lư ng giác Định nghĩa 1.1 (xem [3]). Bieu thác n Ln(x) = a0 + (ak cos kx + bk sin kx), (1.1) k=1 trong đó: a0, ak, bk ∈ R (k ∈ {1, 2, . . . , n}); |an| + |bn| /= 0 (n ∈ N∗ ), được goi là đa thác lượng giác b c n (cap n) với các h so a0, ak, bk (k ∈ {1, 2, . . . , n}). Định nghĩa 1.2 (xem [3]). Neu trong đa thác (1.1) tat cả các h so bk (k 1, 2, . . . , n ) đeu bang 0 thì ta có đa thác lượng giác cap n thuan cos: Cn(x) = a0 + a1 cos x + a2 cos 2x + · · · + an cos nx (an 0) (1.2) Neu trong (1.1) tat cả các h so ak (k 1, 2, . . . , n ) đeu bang 0 thì ta có đa thác lượng giác cap n thuan sin: Sn(x) = b0 + b1 sin x + b2 sin 2x + · · · + bn sin nx (bn /= 0). (1.3) Tính chat 1.1. Cho Sn(x) và Sm ∗ (x) là hai đa thác lượng giác. Khi đó: a) Sn(x) + Sm ∗ (x) là đa thác lượng giác b c k với k max n, m . b) Sn(x).Sm ∗ (x) là đa thác lượng giác b c n + m. Tính chat 1.2. Với moi đa thác lượng giác Ln(x) dạng (1.1) luôn ton tại các đa thác đại so Pn(t) và Qn−1(t) sao cho Ln(x) = Pn(cos x) + sin xQn−1(cos x).
  • 7. 5 Tải tài liệu tại sividoc.com Σ Σ Σ Tính chat 1.3. Với moi Sn(x) dạng (1.3) luôn luôn ton tại đa thác đại so Qn−1(t) đe Ln(x) = b0 + sin x.Qn−1(cos x). Tính chat 1.4. Với moi đa thác Cn(x) dạng (1.2) ta đeu có Cn(x) = Pn(cos x), trong đó Pn(t) là đa thác b c n đoi với t và có h so b c cao nhat là an2n−1 . Ngược lại, với moi đa thác Pn(t) với h so chính bang 1 thì tà phép đ t ȁn phụ t = cos x ta đeu bien đői ve được đa thác Cn(x) dạng (1.2) với an = 21−n . Bài toán 1.1. Cho đa thác k f(x) = a0 + (aj cos jx + bj sin jx) (k ≥ 1) (1.4) j=1 và cho so α thoả mãn đieu ki n nα = 2π với n > k. Cháng minh rang f(x + α) + f(x + 2α) + . . . + f(x + nα) = na0. (1.5) Lài giai. Nh n xét rang tő hợp tuyen tính của các đa thác dạng (1.4) cũng là m®t đa thác có dạng đó. Vì v y không mat tính tőng quát ta chỉ can cháng minh (1.5) cho trường hợp đa thác dạng f (x) = sin mx và f (x) = cos mx là đủ. M t khác, ta có n cos(α + kβ) = 0, k=1 n sin(α + kβ) = 0 k=1 đúng với moi α ∈ R, 0 /= β < 2π và nβ . 2π. Tà đó ta có ngay đȁng thác (1.5) là đúng. Bài toán 1.2. Cho đa thác f(x) = b0 + b1 sin x + b2 sin 2x + · · · + bn sin nx, bn = / 0, thoả mãn đieu ki n Cháng minh rang |f(x)| ≤ | sin x|, ∀x ∈ R. |b1 + 2b2 + 3b3 + · · · + nbn| ≤ 1. (1.6)
  • 8. 6 Tải tài liệu tại sividoc.com . .. . . . . .. = |f′(0)| = . lim f(x) − f(0) . ≤ lim . f(x) − f(0) . x→0 sin x đieu phải cháng minh. x . x→0 . x . P = 1 − √ 2 − R cos 2 α − β + 4 Lài giai. Ta có |b1 + 2b2 + 3b3 + · · · + nbn| = f(x) sin x f(x) = lim = lim ≤ 1, Bài toán 1.3. Cho các so thực a, b, A, B sao cho đa thác lượng giác f(x) = 1 − a cos x − b sin x − A cos 2x − B sin 2x thoả mãn đieu ki n f(x) ≥ 0, ∀x ∈ R. Cháng minh rang a2 + b2 ≤ 2, A2 + B2 ≤ 1. Lài giai. Đ t √ a2 + b2 = r; √ A2 + B2 = R. Khi đó ton tại α, β đe Tà đó suy ra a = r cos α; b = r sin α, a cos x + b sin x = r cos(x − α), A = R cos 2β; B = R sin 2β, A cos 2x + B sin 2x = R cos 2(x − β). f(x) = 1 − r cos(x − α) − R cos 2(x − β). Đ t f(α + π 4 ) = P, f(α − π ) = Q. 4 Khi đó, ta có các đȁng thác r π r Q = 1 − √ 2 − R cos 2 π α − β − 4 . Neu r2 > 2 thì 1 − √ r < 0. x x→0 x x→0 2 ,
  • 9. 7 Tải tài liệu tại sividoc.com 4 4 ≤ ≤ ≥ ∀ ∈ − — − Σ i i n n Trị tuy t đoi của hi u hai góc 2(α − β + π ) và 2(α − β − π ) bang π 4 4 nên các cosin của chúng trái dau. Bởi v y, trong hai bieu thác R cos 2 α − β + π R cos 2 α − β − π có m®t bieu thác không âm. Tà đó dan đen trong hai so P và Q có m®t so âm. V y ít nhat m®t trong hai giá trị f(α + π ) và f(α − π ) là so âm. Đieu đó là vô lý (do giả 4 4 thiet f(x) 0, x R). V y r2 2 suy ra a2 + b2 2. Tương tự ta có f(β) = 1 − r cos(β − α) − R cos 0 = 1 − r cos(β − α) − R; f(β + π) = 1 − r cos(β − α + π) − R. Neu xảy ra trường hợp R > 1 thì 1 R < 0 và do hi u của 2 góc β α + π và β α bang π nên l p lu n tương tự như trên ta thu được m®t trong hai so f(β) và f(β + π) là so âm, vô lý. V y A2 + B2 ≤ 1, đieu phải cháng minh. Nh n xét 1.1. Bài toán trên là truờng hợp đ c bi t của định lý ve đa thác lượng giác nh n giá trị không âm: Neu đa thác n f(x) = 1 + (ak cos kx + bk sin kx) k=1 không âm với moi x ∈ R thì a2 + b2 ≤ 2, ∀i ∈ {1, 2, . . . , (n − 1)}, a2 + b2 ≤ 1. Bài toán 1.4. Cháng minh rang với m = 2n − 1, đa thác lưong giác f(x) = cos 2n x + a1 cos(2n − 1)x + a2 cos(2n − 2)x + . . . + am cos x (1.7) không the chỉ nh n giá trị cùng dau. Lài giai. Giả sả f(x) chỉ nh n giá trị dương. Khi đó 1 f1(x) := (f(x) + f(x + π)) > 0 2 và
  • 10. 8 Tải tài liệu tại sividoc.com ∈ 4 Σ ∈ ∀ ∈ n với moi x ∈ R. Do cos(x + kπ) = (−1)k cos x nên đa thác f1(x) = cos 2n x + a2 cos(2n − 2)x + . . . + am−2 cos 2x dương với moi x R. Do đó đa thác 1 1 f2(x) := 2 (f1(x) + f1(x + 2 π)) dương với moi x ∈ R. Tương tự như trên ta cũng thu được f2(x) = cos 2n x + a4 cos(2n − 4)x + . . . + am−4 cos 4x. V y f(x) = 1 f 2 2(x) + f2 x + 1 π dương với moi x ∈ R. L p lại quá trình trên, sau hǎu hạn bước ta thu được đa thác cos 2n x > 0 với moi x ∈ R. Đieu đó không xảy ra. Nh n xét 1.2. Neu sả dụng đ c trưng tuan hoàn của các nguyên hàm F (x) của f(x) dạng (1.7) thì F (x) không the là hàm thực sự đơn đi u và do đó đạo hàm của nó (chính là f(x)) không the luôn luôn cùng dau. Bài toán 1.5. Cho đa thác n fn(x) = a0 + (ak cos kx + bk sin kx), k=1 trong đó các so thực a0, ak, bk R thoả mãn đieu ki n fn(x) > 0, x R, a2 + b2 = 1 (i = 1, 2, . . . , n). i i Cháng minh rang fn(x) − n a0 ≤ 1, ∀x ∈ R. Lài giai. Ta có fn(x) ≤ a0 + Σ q a2 + b2 = a0 + n. (1.8) Xét nguyên hàm của fn(x) i i i=1 F (x) = a0x + Σ sin ix − cos ix . ai bi i i i=1 n
  • 11. 9 Tải tài liệu tại sividoc.com Do F′(x) = fn(x) > 0 với moi x ∈ R nên F(x) là hàm tăng trên R. Suy ra F(2π) > F(0) tác a0 > 0. Ket hợp với (8) ta thu được fn(x) − n a0 ≤ 1, ∀x ∈ R. 1.2 M t so đong nhat thfíc dạng đại so - lư ng giác Nh n xét rang đȁng thác cơ bản đe dan đen sự phong phú của h thong các đong nhat thác lượng giác là công thác sin2 t + cos2 t = 1, ∀t ∈ R. (1.9) Gan với h thác (1.9) là đong nhat thác Lagrange (2x)2 + (1 − x2 )2 = (1 + x2 )2 , ∀x ∈ R. (1.10) Hai công thác (đong nhat thác) (1.9) và (1.10) là hai cách viet của m®t t h thác. Neu ta thay x = tan 2 ngược lại. vào (1.10) thì de dàng thu được (1.9) và Như v y là moi công thác lượng giác sě tương áng với m®t đong nhat thác đại so tương áng. Đieu đó cũng th t de hieu neu chúng ta nhớ lại quá trình dan dat đen định nghĩa các hàm so lượng giác cơ bản đoi với góc nhon được mô tả dựa theo Định lý Pytago: Trong tam giác vuông ABC với cạnh huyen BC ta luôn có h thúc (AB)2 + (AC)2 = (BC)2 . Tuy nhiên, với so lượng các công thác bien đői lượng giác quá nhieu, bản thân các h thác lượng giác tạo thành m®t chuyên đe có tính đ®c l p tương đoi, dan tách hȁn cơ sở đại so của nó, đã làm cho chúng ta quên đi m®t lượng lớn các h thác đại so có cùng xuat sá tà m®t h thác lượng giác quen biet. Đ c bi t, trong chương trình toán b c phő thông hi n nay, các hàm so lượng giác ngược, hàm lượng giác hyperbolic,... không nam trong phan kien thác bat bu®c thì nhǎng bài toán liên quan đen chúng sě là m®t thách thác lớn đoi với hoc sinh. Ta nhac lại công thác Euler quen biet eiα = cos α + i sin α, α ∈ R.
  • 12. 10 Tải tài liệu tại sividoc.com /∈ − 2 a 2 a2 2 a 2 a 2 a5 2 a 2 a3 2 a2 − Khi đó cos α = sin α = eiα + e−iα , 2 eiα e−iα . 2i Rõ ràng khi khảo sát hàm so cos t thì ít ai nghĩ trong đau rang nó có dạng 1 a + 1 vì khi đó a không còn là m®t so thực. Nhưng neu ta chú ý đen bieu thác eα + e−α 2 , α ∈ R, thì đó chính là cos(iα) (= cosh α) và vì v y, ve m t hình thác, ta sě có nhieu bien đői thu được tà các công thác liên quan đen bien x [ 1, 1] giong như công thác đoi với hàm cos t. Ví dn 1.1. H thác đại so áng với công thác cos 2t = 2 cos2 t − 1 chính là công thác 1 a2 + 1 = 2 h1 a + 1 i2 −1. Ví dn 1.2. H thác đại so áng với công thác cos 3t = 4 cos3 t − 3 cos t chính là công thác 1 a3 + 1 = 4 h1 a + 1 i3 −3 h1 a + 1 i , 2 a3 2 a 2 a hay 4x3 − 3x = 1 a3 + 1 2 a3 với x = 1 a + 1 , a /= 0. Ví dn 1.3. H thác đại so áng với công thác cos 5t + cos t = 2 cos 3t cos 2t chính là công thác 1 a5 + 1 + 1 a + 1 = 2 h1 a3 + 1 ih1 a2 + 1 i .
  • 13. 11 Tải tài liệu tại sividoc.com 2 − 2 a 2 q3 2 q 2 Tà ví dụ trên, sả dụng ket quả khai trien các hàm lượng giác cos 3t và cos 2t, ta thu được đong nhat thác đại so dạng b c 5. 1 a5 + 1 = −m + 2(4m3 − 3m)(2m2 − 1), 2 trong đó a5 m = 1 a + 1 . Ví dn 1.4. Cho so thực m với |m| > 1. Tính giá trị của bieu thác M = 8x3 − 6x, trong đó x = 1 q 3 m + √ m2 − 1 + q 3 m − √ m2 − 1 . Lài giai. Đe ý rang, do |m| > 1 nên ton tại so thực q đe có h thác m = 1 q3 + 1 . Ta chỉ can chon là đủ. Khi đó q = q 3 m + √ m2 − 1 1 q + 1 = 1 q 3 m + √ m2 − 1 + q 3 m − √ m2 − 1 = x. Theo Ví dụ 1.2 thì nên M = 2m. 4x3 − 3x = m Ví dn 1.5. Không dùng máy tính, tìm giá trị đúng của góc nhon x thoả mãn cos x = q 1 + ( √ 6 + 1 √ 2 − √ 3 − 2)2 Lài giai. Xét A = √ 6 + √ 2 − √ 3 − 2. Ta có A = ( √ 3 − √ 2)( √ 2 − 1) = √ 3 √ 2 √ 2 + 1 , .
  • 14. 12 Tải tài liệu tại sividoc.com √ r — − hay √ 3 √ 2 π π A = 2 − 2 2 1 cos 6 − cos 4 = π π sin + sin π = tan . 24 V y nên 2 + 2 4 6 1 + A2 = 1 + tan2 π = 1 π . Suy ra 24 cos2 24 hay cos x = cos2 π 24 π = cos , 24 π x = ± 24 + k2π, k ∈ Z. π Do x là góc nhon nên x = . 24 Cách 2. Tà h thác đã cho 1 ta thu được cos x = q 1 + ( √ 6 + √ 2 − √ 3 − 2)2 1 + ( √ 6 + √ 2 √ 3 2)2 = 1 cos2 x = 1 + tan2 x. Do đó tan2 x = ( √ 6 + √ 2 − √ 3 − 2)2 , hay tan x = √ 6 + √ 2 − √ 3 − 2 > 0, do x là góc nhon. Tiep theo ta sả dụng h thác góc nhân đôi đoi với hàm so tang ho c hàm so cosin, ta thu được công thác tính góc nhon x. Bây giờ ta chuyen sang xét các h thác đại so liên quan đen hàm so sin t. Tà công thác Euler, ta thu được h thác eit − e−it i sin t = . 2 Tà đây suy ra bieu thác i sin(it) nh n giá trị thực. Đieu này gợi ý cho ta cách chuyen đői các đong nhat thác đoi với hàm so sin sang các đong nhat thác đại so. ,
  • 15. 13 Tải tài liệu tại sividoc.com 2 a5 2 a 2 a3 2 a2 2 a Ví dn 1.6. Xét công thác khai trien sin 3t = 3 sin t − 4 sin3 t. Tà đây ta thu được công thác (hình thác) i sin i(3t) = 3(i sin it) + 4(i sin it)3 . H thác đại so áng với công thác trên chính là đong nhat thác 1 a3 − 1 = 3 h1 a − 1 i +4 h1 a − 1 i3 , hay 2 a3 2 a 2 a 4x3 + 3x = 1 a3 − 1 với 2 a3 x = 1 a − 1 , a 0. 2 a Ví dn 1.7. Xét công thác bien đői sin 5t + sin t = 2 sin 3t(1 − 2 sin2 t). (1.11) Ta viet lại công thác (1.11) dưới dạng i sin i(5t) + i sin it = 2i sin i(3t)(1 + 2(i sin it)2 . H thác đại so áng với công thác trên chính là đong nhat thác 1 a5 − 1 + 1 a − 1 = 2 h1 a3 − 1 ih 1 + 1 a − 1 2i . Tà ví dụ trên, sả dụng ket quả khai trien các hàm lượng giác sin 3t và sin 2t, ta thu được đong nhat thác đại so dạng như ở ví dụ sau đây. Ví dn 1.8. 1 a5 − 1 = −m + 2(4m3 + 3m)(2m2 + 1), 2 trong đó a5 m = 1 a − 1 .
  • 16. 14 Tải tài liệu tại sividoc.com 2 √ √ √ √ √ 2 q3 2 q 2 √ (1 + z2)(1 + x2) − √ 1 + z2 − √ 1 + x2 Ví dn 1.9. Cho so thực m. Tính giá trị của bieu thác M = x3 + 3 x, 4 trong đó x = 1 q 3 m + √ m2 + 1 + q 3 m − √ m2 + 1 . Lài giai. Đe ý rang, với moi m đeu ton tại so thực q đe m = 1 q3 − 1 . Ta chỉ can chon là đủ. Khi đó q = q 3 m + √ m2 + 1 1 q − 1 = 1 q 3 m + √ m2 + 1 + q 3 m − √ m2 + 1 = x. Theo Ví dụ 1.6 thì 1 4x3 + 3x = m nên M = m. 4 Tà nhǎng ket quả nh n được, ta có the giải và bi n lu n được nhieu dạng phương trình đại so b c cao và công thác tính giá trị của m®t so bieu thác cháa căn thác. Bài toán 1.6. Cho các so dương x, y, z thỏa mãn đieu ki n x+y +z = xyz. Cháng minh rang (1 + y2)(1 + z2) − 1 + y2 − √ 1 + z2 yz + zx (1 + x2)(1 + y2) − 1 + x2 − 1 + y2 xy = 0. Lài giai. Đ t x = tan α, y = tan β, z = tan γ +
  • 17. 15 Tải tài liệu tại sividoc.com 2 √ √ √ √ √ √ √ √ với Do x + y + z = xyz, nên α, β, γ ∈ 0; π . tan α + tan β + tan γ = tan αtan βtan γ ⇔ tan α + tan β = tan γ(tan αtan β − 1) tan α + tan β ⇔ 1 − tan αtan β = − tan γ (de thay tan α tan β = / 1) ⇔ α + β = −γ + kπ ⇔ α + β + γ = kπ, (k ∈ Z) Do α + β + γ ∈ 0; 3π , suy ra 0 < kπ < 3π , mà k ∈ Z nên k = 1. 2 2 V y nên α + β + γ = π. Ta suy ra (1 + y2)(1 + z2) − 1 + y2 − √ 1 + z2 yz (1 + tan2 β)(1 + tan2 γ) − 1 + tan2 β − 1 + tan2 γ tan βtan γ 1 1 1 1 cos β cos γ − cos β − cos γ = sin β sin γ cos β cos γ = 1 − (cos β + cos γ) . sin β sin γ Tương tự, ta cũng có (1 + z2)(1 + x2) − √ 1 + z2 − √ 1 + x2 zx = 1 − (cos γ + cos α) sin γ sin α và (1 + x2)(1 + y2) − √ 1 + x2 − 1 + y2 xy = 1 − (cos α + cos β) . sin α sin β Khi đó ve trái của đȁng thác can cháng minh bang 1 − (cos β + cos γ) + 1 − (cos γ + cos α) + 1 − (cos α + cos β) sin β sin γ sin γ sin α sin α sin β = sin α + sin β + sin γ − sin(α + γ) − sin(α + β) − sin(β + γ) sin α sin β sin γ = 0, đieu phải cháng minh. =
  • 18. 16 Tải tài liệu tại sividoc.com Bài toán 1.7. Cho xy /= −1, yz /= −1, zx /= −1. Cháng minh rang x − y 1 + xy + y − z 1 + yz + z − x 1 + zx = x − y 1 + xy . y − z 1 + yz . z − x . 1 + zx Lài giai. Đ t x = tan α, y = tan β, z = tan γ, với α, β, γ ∈ − π ; π . Khi đó x − y 1 + xy + y − z 1 + yz 2 2 + z − x 1 + zx = tan α − tan β 1 + tan αtan β + tan β − tan γ 1 + tan βtan γ + tan γ − tan α 1 + tan αtan γ = tan(α − β) + tan(β − γ) + tan(γ − α) và x − y y − z z − x 1 + xy . 1 + yz . 1 + zx = tan(α − β) tan(β − γ) tan(γ − α). Ta cháng minh đong nhat thác tan(α − β) + tan(β − γ) + tan(γ − α) = tan(α − β) tan(β − γ) tan(γ − α). Th t v y, ta có nên tan a + tan b 1 − tan a tan b = tan(a + b) Suy ra tan a + tan b = tan(a + b)(1 − tan a tan b). tan(α − β) + tan(β − γ) + tan(γ − α) = tan(α − β + β − γ)[1 − tan(α − β) tan(β − γ)] + tan(γ − α) = tan(α − γ) + tan(γ − α) + tan(α − β) tan(β − γ) tan(γ − α) = tan(α − β) tan(β − γ) tan(γ − α). Do đó x − y 1 + xy + y − z 1 + yz + z − x 1 + zx = x − y 1 + xy . y − z 1 + yz . z − x , 1 + zx đieu phải cháng minh.
  • 19. 17 Tải tài liệu tại sividoc.com ∈ ∈ − 1.3 Đa thfíc Chebyshev 1.3.1 Các định nghĩa Định nghĩa 1.3 (xem [3]). Các đa thác Tn(x) (n N) được xác định như sau T0(x) = 1; T1(x) = x, Tn+1(x) = 2xTn(x) − Tn−1(x), ∀n > 1 được goi là các đa thác Chebyshev (loại 1). Định nghĩa 1.4 (xem [3]). Các đa thác Un(x) (n ∈ N) xác định như sau U0(x) = 0; U1(x) = 1, Un+1(x) = 2xUn(x) − Un−1(x), ∀n > 1 được goi là các đa thác Chebyshev (loại 2). 1.3.2 Tính chat của các đa thfíc Chebyshev Tính chat 1.5. Tn(x) = cos(n arccos x) với moi x ∈ [−1, 1] Tính chat 1.6. Tn(x) Z[x] b c n có h so b c cao nhat bang 2n−1 và là hàm chȁn khi n chȁn; là hàm lẻ khi n lẻ. Tính chat 1.7. Tn(x) có đúng n nghi m phân bi t trên [-1, 1 ] là 2k + 1 xk = cos 2n π (k = 0, 1, . . . , n − 1). kπ Tính chat 1.8. |Tn(x)| ≤ 1, ∀x ∈ [−1, 1] và |Tn(x)| = 1 khi x = cos n , k ∈ Z. Tính chat 1.9. Đa thác T ∗(x) = 21−n Tn(x) là đa thác b c n với h so b c cao nhat bang 1 và có đ® l ch so với 0 trên [ 1, 1] là nhỏ nhat trong tat cả các đa thác b c n với h so b c cao nhat bang 1. Tính chat 1.10. Un(x) = sin(n arccos x) √ 1 − x2 với moi x ∈ (−1, 1).
  • 20. 18 Tải tài liệu tại sividoc.com − n n n sin t 1 sin nt Tính chat 1.11. U (x) = T ′ (x) = , cos t = x, đa thác b c n − 1 có h so b c cao nhat bang 2n−1 và là hàm chȁn khi n lẻ; là hàm lẻ khi n chȁn. Tính chat 1.12. Tn(x) có đúng n nghi m phân bi t trên [-1, 1 ] là 2k + 1 xk = cos 2n π (k = 0, 1, . . . , n − 1). Tính chat 1.13. |Un(x)| ≤ n, ∀x ∈ [−1, 1] và |Tn ′ (x)| ≤ n2 , ∀x ∈ [−1, 1]. Xét các hàm so sinh x = 1 (ex e−x ), cosh x = 2 1 (ex + e−x ). 2 Khi đó với |x| > 1 thì Tn(x) = cosh(nt); Un(x) = trong đó x = cosh t. sinh(nt) , (1.12) sinh t Bài toán 1.8. Cháng minh rang đa thác Un(x) có đúng n − 1 nghi m thực khác nhau trong (−1, 1). Lài giai. Sả dụng Tính chat 1.3 và do Tn(x) có đúng n nghi m thực phân bi t trên [−1, 1] nên theo Định lý Rolle ta có ngay đieu phải cháng minh. Bài toán 1.9. Cháng minh rang Un(x) = xUn−1(x) + Tn−1(x), ∀n ∈ N∗; x ∈ R. Lài giai. Ta cháng minh cho trường hợp x ∈ (−1, 1). Đ t x = cos t (0 < t < π) và sả dụng các tính chat của Tn(x) và Un(x) ta thu được đieu phải cháng minh. Với |x| > 1 thì ta sả dụng ket quả của đȁng thác (1.12). Bài toán 1.10. Cháng minh rang Tn+1(x) = xTn(x) − (1 − x2 )Un(x), ∀x ∈ R, n ∈ N. (1.13) Lài giai. Sả dụng phương pháp quy nạp, ta sě thu được (1.13)
  • 21. 19 Tải tài liệu tại sividoc.com | | ∈ Bài toán 1.11. Cháng minh rang (1 − x2 )Tn ′′ (x) − xTn ′ (x) + n2 Tn(x) = 0, ∀n ∈ N, x ∈ R. Lài giai. Với x ∈ [−1, 1], ta sả dụng tính chat 1.2 của Tn(x) và thay trực tiep bieu thác của Tn(x) vào đieu ki n bài ra ta thay đȁng thác đúng. Với x > 1 thì sả dụng (1.13) và các tính chat của các hàm so sinh x và cosh x. Bài toán 1.12. Cháng minh rang với m, n ∈ N; n ≥ m và x ∈ R thì Tn+m(x) + Tn−m(x) = 2Tn(x)Tm(x). Lài giai. Sả dụng định nghĩa và phương pháp quy nạp ho c sả dụng các công thác cos(n + m)x + cos(n − m)x = 2 cos nx cos mx và cosh(n + m)x + cosh(n − m)x = 2 cosh(nx) cosh(mx). Bài toán 1.13. Cháng minh rang Tm(Tn(x)) = Tmn(x), ∀x ∈ R, m, n ∈ N. (1.14) Lài giai. Ta cháng minh (1.14) bang phương pháp quy nạp theo m. Với n co định tuỳ ý và m = 0, ta có T0(Tn(x)) = 1 = T0n(x) (theo định nghĩa Tn(x)). V y (1.14) đúng với m = 0; n N. Giả sả (1.14) đúng tới m. Khi đó Tm+l(Tn(x)) = 2Tn(x)Tm(Tn(x)) = 2Tn(x)Tmn(x) − T(m−l)n(x) (theo giả thiet quy nạp) = Tn+mn(x) + Tmn−n(x) − Tmn−n(x) (theo Bài toán1.3 ) = T(m+1)n(x). V y Tm(Tn(x)) = Tmn(x), ∀x ∈ R, m, n ∈ N.
  • 22. 20 Tải tài liệu tại sividoc.com 0 √ 0 0 ∆ = (ax0 + b)2 − 4a(ax2 + bx0 + c) > 0 0 Chương 2. Phương pháp lư ng giác giải phương trình b c ba và b c bon 2.1 Giải phương trình b c ba Trong phan này ta nêu phương pháp giải phương trình b c ba với h so thực tùy ý : ax3 + bx2 + cx + d = 0, a /= 0. (2.1) 2.1.1 Giải và bi n lu n phương trình b c ba Bài toán 2.1. Giải phương trình (2.1) khi biet m®t nghi m x = x0. Lài giai. Theo giả thiet thì ax3 + bx2 + cx0 + d = 0 0 0 (2.1) ⇔ ax3 + bx2 + cx + d = ax3 + bx2 + cx0 + d 0 0 3 3 2 2 ⇔ a(x x0) + b(x 2 — x0) + c(x − x0) = 0 ⇔ (x − x0)[ax + (ax0 + b)x + ax0 + bx0 + c] = 0. 1) Neu ∆ = (ax0 + b)2 − 4a(ax2 + bx0 + c) < 0 thì phương trình (2.1) có nghi m duy nhat x = x0. 2) Neu ∆ ≥ 0 thì phương trình (2.1) có các nghi m x = x0 x = −(ax0 + b) ± ∆ 2a H quả 2.1. 1) Neu x0 là nghi m của phương trình (2.1) thì đieu ki n can và đủ đe phương trình (2.1) có ba nghi m phân bi t là ax2 + (ax0 + b)x0 + ax2 + bx0 + c 0 2 −
  • 23. 21 Tải tài liệu tại sividoc.com − 0 0 3 c 0 x1 + x2 + x3 = − b d b 3 2 √ 2) Neu x0 là nghi m của (2.1) thì có the phân tích : ax3 + bx2 + cx + d = (x − x0)f(x). (2.2) trong đó f(x) là m®t tam thác b c hai xác định. 3) Neu x1, x2, x3 là các nghi m của phương trình (2.1) thì ax3 + bx2 + cx + d = a(x − x1)(x − x2)(x − x3) và có công thác Viete a c x1x2 + x2x3 + x3x1 = a x1x2x3 = a Bài toán 2.2. Giải phương trình với ax3 + bx2 + cx + d = 0 (2.3) ac3 = db3 . (2.4) (Khi đó phương trình (2.3) (2.4) có tên goi là phương trình quy hoi b c ba) Lài giai. Tà (2.4), suy ra 1) Neu c = 0 thì b = 0 và (2.4) ⇔ ax3 + d = 0 ⇔ x = r 3 d − a , d 2) Neu c /= 0 thì b /= 0 và c = ( )3 . c a b Đ t = −x0 thì c = −bx0, d = −ax3 . The vào (2.3), ta được ax3 + bx2 − bx0x − ax3 = 0 ⇔ a(x − x0) + bx(x − x0) = 0 2 ⇔ (x − x0)[ax + (ax0 + b)x + ax ] = 0. V y x = x0 = − b là m®t nghi m. Neu ∆ = (ax0 + b)2 − 4a2 x2 ≥ 0 thì phương trình còn có nghi m 0 x = −(ax0 + b) ± ∆ . 2a
  • 24. 22 Tải tài liệu tại sividoc.com — ± − . . 1 0 . . 1 3 1 3 2 Bài toán 2.3. Giải phương trình: 4x3 3x = m với m = 1. Lài giai. Khi m = 1 thì phương trình có dạng 4x3 − 3x = 1 ⇔ (x − 1)(2x + 1)2 = 0 1 nên phương trình có nghi m đơn x = 1 và nghi m kép x = − 2 . Khi m = −1 thì phương trình có dạng 4x3 − 3x = −1 ⇔ (x + 1)(2x − 1)2 = 0 1 nên phương trình có nghi m đơn x = −1 và nghi m kép x = 2 . Bài toán 2.4. Giải phương trình: 4x3 − 3x = m với |m| < 1. Lài giai. Đ t m = cos α = cos(α ± 2π). Khi đó cos α = cos(3. α ) = 4 cos3 α 3 cos α. 3 Do v y phương trình có ba nghi m x1 α = cos 3 x2,3 = cos α ± 2π 3 Bài toán 2.5. Giải phương trình 4x3 − 3x = m, |m| > 1. Lài giai. Ta cháng minh phương trình có nghi m duy nhat. Th t v y, phương trình không có nghi m trong [−1, 1] vì neu x = x0 ∈ [−1, 1] là nghi m thì ta đ t x0 = cos β. Khi đó 4x3 − 3x0 = |cos 3β| ≤ 1 /= m. Giả sả phương trình có nghi m x = x1 với |x1| > 1. Khi đó 4x3 −3x1 = m. V y có phương trình 4x3 − 3x = 4x3 − 3x1 ⇔ 4(x − x1) − 3(x − x1) = 0 2 ⇔ (x − x1)(4x + 4x1x + 4x − 3) = 0 ∆ = 4x2 − 4(4x2 − 3) = 12 − 12x2 < 0. 1 1 1 V y x = x1 là nghi m duy nhat.
  • 25. 23 Tải tài liệu tại sividoc.com 0 0 2 a3 2 a3 a) Neu p = 0 thì phương trình có nghi m duy nhat y = 3 q. 3 Đ t m = 1 (a3 + 1 ) với a3 = m ± √ m2 − 1. Khi đó, phương trình có nghi m duy nhat 1 1 x = (a + ) = 2 a 1 q 3 m + √ m2 − 1 + q 3 m − √ m2 − 1 . Bài toán 2.6. Giải phương trình 4x3 + 3x = m. Lài giai. Nh n xét rang neu x = x0 là nghi m của phương trình thì đó là nghi m duy nhat. Th t v y, xét x > x0 khi đó 4x3 + 3x > 4x3 + 3x0 = m. Tương tự, với x < x0 thì 4x3 + 3x < 4x3 + 3x0 = m. 1 1 Đ t x = (a − ). 2 a Khi đó de dàng kiem tra đȁng thác 4x3 + 3x = 1 a3 − 1 . (2.5) 2 a3 Suy ra cách giải phương trình như sau. Đ t m = 1 (a3 − 1 ) với a3 = m ± √ m2 + 1. Khi đó theo (2.5) ta có nghi m duy nhat của phương trình : x = (a − ) = q 1 1 1 3 m + √ m2 + 1 + q 3 m − √ m2 + 1 . 2 a 2 Bài toán 2.7. Giải và bi n lu n phương trình t3 + at2 + bt + c = 0. a Lài giai. Đ t t = y − 3 . Khi đó ta có the viet phương trình dưới dạng (y − a )3 + a(y − a )2 + b(y − a ) + c = 0 3 3 3 ⇔ y − py = q, a2 2a3 ab trong đó p = 3 − b; q = − 27 + 3 − c. √ 2
  • 26. 24 Tải tài liệu tại sividoc.com ± − 3 2 d3 3 b) Neu p > 0. Đ t y = 2 r p x. Khi đó ta được phương trình 4x3 −3x = 3 √ 3q m với m = 2p √ p . -) |m| ≤ 1. Đ t m = cos α. Khi đó phương trình có ba nghi m: x = cos α ; x = cos α ± 2π . 3 3 -) |m| > 1. Đ t m = 1 (d3 + 1 ) với d3 = m √ m2 1. 2 d3 Khi đó phương trình có nghi m duy nhat 1 1 x = (d + ) = 2 d 1 q 3 m + √ m2 − 1 + q 3 m − √ m2 − 1 . c) Neu p < 0. Đ t y = 2 r − p x ta được phương trình 4x3 + 3x = m. Đ t m = 1 (d3 − 1 ) với d3 = m ± √ m2 + 1. Khi đó phương trình có nghi m duy nhat x = (d − ) = q 1 1 1 3 m + √ m2 + 1 + q 3 m − √ m2 + 1 . 2 d 2 Bài toán 2.8. Cho x1, x2, x3 là nghi m của phương trình x3 + ax2 + x + b = 0 (2.6) và b 0. Cháng minh rang 1 (x1 − x 1 )(x2 − x 1 ) + (x2 − x 1 )(x3 − x 1 ) + (x3 − x 1 )(x1 − x ) = 4. Lài giai. Vì x1, x2, x3 là nghi m của phương trình (2.6) nên x1x2 + x2x3 + x3x1 = 1 hay x1x2x3 = −b 0 x1x2 + x2x3 + x3x1 = 1 x1, x2, x3 0 2 1 2 2 3 3 1
  • 27. 25 Tải tài liệu tại sividoc.com 1 − 1 2 2 3 3 Đ t x = tan α, x = tan β, x = tan γ với α, β, γ ∈ − π ; π . 1 2 3 2 2 Ta có tan α tan β + tan β tan γ + tan γ tan α = 1 ⇔ 1 − tan α tan β = (tan α + tan β) tan γ (2.7) Neu tan α tan β = 1 thì tan α + tan β = 0 ho c tan γ = 0. lý. Neu tan α + tan β = 0, thì tan α = − tan β, khi đó − tan2 β = 1, vô Neu tan γ = 0 thì x3 = 0, vô lý. V y tan α tan β /= 1. Khi đó tà (2.7), ta có tan α + tan β 1 − tan α tan β . tan γ = 1 π ⇔ tan(α + β) = tan( 2 − γ) π ⇔ α + β + γ = 2 + kπ, (k ∈ Z) và 1 (x1 − x 1 )(x2 − x 1 ) + (x2 − x 1 )(x3 − x 1 ) + (x3 − x 1 )(x1 − x ) = (tan α − cot α)(tan β − cot β) + (tan β − cot β)(tan γ − cot γ) + (tan γ − cot γ)(tan α − cot α) = 4 cot 2α cot 2β + 4 cot 2γ(cot 2α + cot 2β) = 4cot 2α cot 2β − 4 cot(2α + 2β)(cot 2α + cot 2β) ( do 2γ = π − (2α + 2β + k2π)) = 4cot 2α cot 2β 4 cot 2α cot 2β − 1 (cot 2α + cot 2β) = 4. cot 2α + cot 2β V y 1 1 1 1 1 1 (x1 − x )(x2 − x ) + (x2 − x )(x3 − x ) + (x3 − x )(x1 − x ) = 4, đieu phải cháng minh. 1 2 2 3 3 1
  • 28. 26 Tải tài liệu tại sividoc.com — − ≤ 4 − 4 8 3 Bài toán 2.9. Cho phương trình x3 px2 + qx p = 0 với p, q là các so dương. Cháng minh rang neu phương trình đã cho có ba nghi m phân bi t x1, x2, x3 lớn hơn ho c bang 1, thì ta luôn có p ≥ 1 + √ 2 ! (q + 3). (2.8) Lài giai. Giả sả x1 < x2 < x3. Theo định lý Vieete, ta có x1 + x2 + x3 = p x1x2 + x2x3 + x1x3 = q x1x2x3 = p Đ t x1 = tan A, x2 = tan B, x3 = tan C với A, B, C là ba góc của tam giác và π A, B, C < 4 π 2 . Không giảm tőng quát, ta giả sả A = min{A, B, C} π π thì ≤ A ≤ . Khi đó (2.8) tương đương với 2 + √ 2 tan A tan B tan C ≥ 2 + √ 2 (tan A tan B + tan B tan C + tan C tan A + 3) 8 ⇔ 1 ≥ (cot A + cot B + cot C + 3 cot A cot B cot C) 8 ⇔ 8 − 4 √ 2 ≥ cot A + cot B + cot C + 3 cot A cot B cot C. (2.9) Đe cháng minh (2.9), ta chú ý cot A + cot B + cot C + 3 cot A cot B cot C 2 sin A = cot A + cos(B − C) + cos A + 3 cot A cos(B − C) + cos(B + C) cos(B − C) − cos(B + C) cot A + 2 sin A 1 + cos A + 3 cot A 1 − cos A , 1 + cos A 1 − tan2 A 1 cot A = tan A 1 = A = 2 tan 2 1 tan2 A 2 2 , A 2 tan 2 ≤
  • 29. 27 Tải tài liệu tại sividoc.com ≤ 2 2 2 2 sin A = 1 + cos A 2.2sin A A cos 2 2 A A = 2 tan 2 2 cos2 2 và 1 − tan2 A 2 sin2 A 3 cot A 1 − cos A = 3 1 + cos A 2 A 2 tan 2 2 2 cos2 A 2 1 − tan2 A = 3 A 2 2 tan 2 tan2 A 2 1 − tan2 A tan A tan A − tan3 A = 3 nên ta có 2 2 = 3 2 2 , 2 2 cot A + cot B + cot C + 3 cot A cot B cot C 1 − tan2 A tan A − tan3 A hay ≤ A 2 2 tan 2 A + 2 tan + 3 2 2 cot A + cot B + cot C + 3 cot A cot B cot C ≤ 1 + 3 tan A − 3 tan3 A A 2 tan 2 π π 2 2 2 π A π M t khác vì 4 ≤ A ≤ 3 ⇒ 8 ≤ nên ta có 2 6 Xét hàm so √ 2 − 1 ≤ tan A 1 √ 3 . 3 3 f(t) = − 2 t 1 + 3t + 2t , t ∈ 1 2 − 1; √ 3 . ≤ √
  • 30. 28 Tải tài liệu tại sividoc.com 2 r − — − — − − √ 1 π − — − 2 ≤ 0, Ta có f′(t) = 3 t2 2 1 9t4 + 6t2 1 + 3 − 2t2 = 2t2 = − (3t2 − 1)2 2t với ∀t 0 nên f(t) nghịch bien trên 2 − 1; √ 3 . Suy ra f(t) ≤ f( √ 2 − 1) = 8 − 4 √ 2, ta có đieu phải cháng minh. cos(B − C) = 1 3π B = C = , Dau "=" xảy ra khi A tan = 2 √ 2 − 1 hay 8 A = 4 . 2.1.2 Phương trình b c ba nh n các yeu to trong tam giác là nghi m Trong phan này ta xét các ∆ABC với đ® dài ba cạnh BC = a, CA = b, AB = c. Ký hi u p là nảa chu vi, r và R là bán kính các đường tròn n®i, ngoại tiep, ra, rb, rc tương áng là bán kính các đường tròn bàng tiep, ha, hb, hc tương áng là đ® dài các đường cao,. . . Bài toán 2.10 (Phương trình b c ba theo đ® dài các cạnh của tam giác). Cháng minh rang a, b, c là ba nghi m của phương trình b c ba dưới đây: x3 − 2px2 + (p2 + r2 + 4Rr)x − 4Rrp = 0. (2.10) A Lài giai. Tà h thác tan = 2 r p − a và a = 2R sin A, suy ra A hay 2 tan a = 2R 2 1 + tan2 A 2 r a = 4R p − a 1 + p − a = 4Rr p − a . r2 + (p − a)2 Như v y, ta có quan h a(a2 2pa + p2 + r2 ) = 4Rr(p a) hay a3 2pa2 + (p2 + r2 + 4Rr)a 4Rrp = 0. Do đó a là m®t nghi m của x3 2px2 +(p2 + r2 + 4Rr)x 4Rrp = 0. Tương tự, b và c cũng là nghi m của phương trình này.
  • 31. 29 Tải tài liệu tại sividoc.com — − − — − − − Bài toán 2.11. Cháng minh rang 1 1 1 , , a b c là nghi m của phương trình 3 p2 + r2 + 4Rr 2 1 1 x x 4pRr 1 + 2Rr x − 4pRr = 0. (2.11) Lài giai. Thay x bởi , ta có ngay đieu phải cháng minh. x Bài toán 2.12. Cháng minh rang p a, p b, p c là các nghi m của phương trình x3 − px2 + r(r + 4R)x − pr2 = 0. Lài giai. Theo công thác di n tích ta có S2 = p(p − a)(p − b)(p − c) = p2 r2 ⇒ (p − a)(p − b)(p − c) = pr2 . p − a + p − b + p − c = 3p − (a + b + c) = p. Giờ ta cháng minh (p − a)(p − b) + (p − b)(p − c) + (p − c)(p − a) = r(r + 4R) ⇔ (p − a)(p − b) + (p − b)(p − c) + (p − c)(p − a) = ⇔ (p − a)(p − b) + (p − b)(p − c) + (p − c)(p − a) = abc + (p − a)(p − b)(p − c) S abc S ( + ) p S p p p boxTac abc + (p − a)(p − b)(p − c) p p abc + p3 − p2 a − p2 b − p2 c + pab + pbc + pca − abc p = p2 − pa − pb − pc + ab + bc + ca = 3p2 − pa − pb − pc + ab + bc + ca − 2p2 = 3p2 − pa − pb − pc + ab + bc + ca − p(a + b + c) = p2 − pa − pb + ab + p2 − pb − pc + bc + p2 − pc − pa + ca = (p − a)(p − b) + (p − b)(p − c) + (p − c)(p − a). V y đȁng thác được cháng minh. Ta có p a, p b, p c là ba nghi m của phuơng trình x3 − px2 + r(r + 4R)x − pr2 = 0. =
  • 32. 30 Tải tài liệu tại sividoc.com 3 − Bài toán 2.13. 1 , p − a 1 , p − b 1 p − c là ba nghi m của phương trình x3 − 4R + r x2 + 1 x − 1 = 0. pr r2 pr2 1 Lài giai. Thay x bởi , ta có đieu phải cháng minh. x Bài toán 2.14 (Phương trình b c ba theo các đường cao). Chúng minh rang ha, hb, hc là các nghi m của phương trình p2 + r2 + 4Rr x x2 + 2R 2p2 r R x − 2p2 r2 = 0. R 1 Lài giai. Theo công thúc tính di n tích tam giác, ta có S = 2 aha = pr ⇒ ha = 2pr a . Tương tự ta có hb = 2pr b , hc = 2pr c . Nên ha + hb + hc = 1 1 2pr( + a b 1 + ) và c ha + hb + hc = 2pr p2 + r2 + 4Rr p2 + r2 + 4Rr = . 4pRr 2R h h + h h + h h 2pr 2pr 2pr 2pr 2pr 2pr = + + = 4p2 r2 ( 1 1 1 + + ), a b b c c a a b b c c a 2 2 1 2p2 r ab bc ca hahb + hbhc + hcha = 4p r = . 2Rr R hahbhc = 8p3 r3 1 abc . Ta có hahbhc = 8p3 r3 1 = 4pRr 2p2 r2 . R V¾y ha, hb, hc là nghi m của phương trình x3 − Bài toán 2.15. p2 + r2 + 4Rr x2 + 2R 2p2 r R x − 2p2 r2 = 0. R 1 1 1 , , ha hb hc
  • 33. 31 Tải tài liệu tại sividoc.com là nghi m của phương trình 3 1 2 p2 + r2 + 4Rr 2R x − r x + 1 4p2r2 x − 4p2r2 = 0. Lài giai. Thay x bởi x ta có ngay đieu phải cháng minh. Bài toán 2.16 (Phương trình b c ba theo bán kính đường tròn bàng tiep). Cháng minh rang ra, rb, rc là nghi m của phương trình x3 − (4R + r)x2 + p2 x − p2 r = 0. Lài giai. Tà S = pr = ra(p − a) = rb(p − b) = rc(p − c) suy ra ra = hay pr p − a , rb = pr p − b , rc = p3r3 pr p − c rarbrc = (p − a)(p − b)(p − c) p4r3 p4r3 2 Ta có = = p(p − a)(p − b)(p − c) S2 = p r. 1 1 1 ra + rb + rc = pr( p − a + p − b + p − c ) 4R + r = pr pr = 4R + r. và rarb +rbrc +rcra = p2 r2 ( 1 (p − a)(p − b) 1 + (p − b)(p − c) 1 + ) (p − c)(p − a) = p2 r2 1 r2 = p2 . V y ra, rb, rc là nghi m của phương trình x3 − (4R + r)x2 + p2 x − p2 r = 0. Bài toán 2.17. Cháng minh rang là nghi m của phương trình 1 1 1 , , ra rb rc x3 − 1 x2 + 4R + r x − 1 = 0. r p2r 1 p2r Lài giai. Thay x bởi x ta có ngay đieu phải cháng minh.
  • 34. 32 Tải tài liệu tại sividoc.com d 2 2.2 Giải phương trình b c bon Trong phan này ta sě nêu phương pháp chung đe phân tích m®t đa thác b c bon tőng quát thành tích của hai tam thác b c hai. Đoi với m®t so dạng đa thác b c bon đ c bi t có nhǎng phép bien đői phù hợp và đơn giản hơn, không đòi hỏi phải v n dụng toàn b® thu t toán tőng quát. Bài toán 2.18. Giải phương trình ax4 + bx3 + cx2 + dx + e = 0 (2.12) với ad2 = eb2 , a, e /= 0. (2.13) (Phương trình (2.12)-(2.13) có tên goi : phương trình hoi quy b c bon). Lài giai. Viet (2.13) dưới dạng e a = ( )2 . b d Đ t b = α có d = bα; e = a(α)2 . The vào phương trình (2.12), ta được: ax4 + bx3 + cx2 + bαx + a(α)2 = 0 ⇔ a(x 2 — α ) + bx(x 2 2 + α) + cα2 = 0 2 ⇔ a(x + α) + bx(x + α) + (c − 2aα)x = 0 Nh n xét x = 0 không thỏa mãn phương trình. Chia hai ve của phương trình cho x2 ta được: với t = x2 + α x at2 + bt + c − 2aα = 0 V y phương trình (2.12)-(2.13) đưa ve h : at2 + bt + c − 2aα = 0 x2 − tx + α = 0 (2.14) Giải phương trình thá nhat của h (2.14) ta tính được t và tà phương trình thá hai của h (2.14) ta tính được x. 4 2
  • 35. 33 Tải tài liệu tại sividoc.com 2 2 2 2 2 2 2 − 2 Bài toán 2.19. Giải phương trình (x − a)4 + (x − b)4 = c (2.15) Lài giai Đ t x = t + a + b ; α = a − b . Khi đó 2 2 (2.15) ⇔ (t + α)4 + (t − α)4 = c ⇔ [(t + α) − (t − α) 2 ]2 + 2(t2 — α ) = c 2 ⇔ 16α t + 2(t − α ) = c. Ta nh n được phương trình trùng phương: 2t4 + 12α2 t2 + 2α4 − c = 0. Bài toán 2.20. Giải phương trình x4 = ax2 + bx + c với b2 = 4(a + 2)(c + 1) Lài giai Tam thác b c hai với ∆ = b2 − 4(a + 2)(c + 1) = 0 có dạng f(x) = (a + 2)x2 + bx + c + 1 và f(x) = (a + 2)x2 + bx + c + 1 có nghi m kép và f(x) = (a + 2)(x + b )2 neu a 2 2(a + 2) c + 1 neu a = −2 Phương trình đã cho được viet dưới dạng x4 + 2x2 + 1 = (a + 2)x2 + bx + c + 1 2 2 ⇔ (x + 1) = (a + 2)x + bx + c + 1 1) Neu a + 2 = 0(⇔ a = −2) thì b = 0 và ta được phương trình (x2 + 1)2 = c + 1 2 ) Neu a + 2 < 0 thì phương trình vô nghi m vì có Ve trái ≥ 0 Ve phải < 0
  • 36. 34 Tải tài liệu tại sividoc.com h i / ≥ ⇔ h i ⇔± 2 3) Neu a + 2 > 0(⇔ a > −2) thì c > −1 thì ta có the viet (x2 + 1)2 = ( √ a + 2x ± √ c + 1)2 (dau + áng với b >√ 0, dau - án √ g với b < 0) và ta được các phương trình b c hai x2 + 1 = ±( a + 2x ± c + 1) Tiep theo giải và bi n lu n theo a, c. Bài toán 2.21. Giải phương trình x4 = ax2 + bx + c, b /= 0 Lài giai. Goi α là so thực thỏa mãn h thác b2 = 4(a + 2α)(c + α2 ) (2.16) (ton tại ít nhat m®t giá trị α thỏa mãn (2.16)) vì (2.16) là phương trình b c ba đoi với α) Khi đó tam thác b c hai f(x) = (a + 2α)x2 + bx + (c + α2 ) có nghi m kép và f(x) = (a + 2α) x + b 2 neu a + 2α = 0 2(a + 2α) c + α2 neu a + 2α = 0 Viet phương trình đã cho dưới dạng x4 + 2αx2 + α2 = f(x) 2 ⇔ (x + α) = f(x) (2.17) 1) Neu a + 2α = 0 thì (2.17) (x2 + α)2 = c + α2 2) Neu a + 2α < 0 thì Ve trái 0; Ve phải <0, nên phương trình vô nghi m. 3) Neu a + 2α > 0 thì (2.17) x2 + α = √ a + 2α x + b . 2(a + 2α) Bài toán 2.22. Giải phương trình t4 + αt3 + βt2 + γt + δ = 0
  • 37. 35 Tải tài liệu tại sividoc.com − / − 1 a ⇔ Lài giai. Đ t t = x − α . Khi đó ta được phương trình: 4 (x − α )4 + α(x − α )3 + β(x − α )2 + γ(x − α ) + δ = 0 4 4 4 4 2 6α2 4 2α3 1 1 4 ⇔ x = ax + bx + c với a = 42 ; b = − 42 + 2 αβ − γ; c = 42 (3α − 42 βα2 +43 αγ 44 δ). Áp dụng bài toán trên ta tìm được nghi m của phương trình. H quả 2.2. Moi đa thác b c bon có nghi m thực đeu phân tích được thành tích của hai tam thác b c hai với h so thực. Bài toán 2.23. Cho α = 0. Khai trien bieu thác (1 α √ x)8 +(1+α √ x)8 = P(x) ta được P(x) là m®t đa thác b c bon. Giải phương trình P(x) = 0 Lài giai. Đ t α2 x = t, ta được phương trình 1 + 28t + 70t2 + 28t3 + t4 = 0 ⇔ (t + 1 1 )2 1 + 28(t + 1 ) + 68 = 0. t Đ t u = t + t ta được phương trình u2 + 28u + 68 = 0 ⇔ u1,2 = −14 ± √ 128 ⇒ t = 1 2 (u1,2 ± q u2 ,2 −4). Bài toán 2.24. Giải phương trình a(ax2 + bx + c)2 + b(ax2 + bx + c) + c = x. Lài giai. Đ t ax2 + bx + c = y ta được h ax2 + bx + c = y ax2 + bx + c = y ay2 + by + c = x a(x2 − y2 ) + (b + 1)(x − y) = 0 ⇔ ax2 + bx + c = y y = x ho c (x − y)[a(x + y) + b + 1] = 0 ax + (b − 1)x + c = 0 y = − b + 1 − x ax2 + bx + c = − b + 1 a — x. 2 ⇔
  • 38. 36 Tải tài liệu tại sividoc.com — ⇔ − √ √ √ " − − √ 2 2 Giải moi phương trình b c hai ta được nghi m của h , tà đó ta suy ra nghi m của phương trình. Bài toán 2.25. Giải phương trình x4 = 3x2 + 10x + 4. Lài giai. Viet phương trình dưới dạng (x2 + α)2 = (3 + 2α)x2 + 10x + 4 + α2 . Chon α đe ∆′ = 25 (3 + 2α)(4 + α2 ) = 0 2α3 + 3α2 + 8α 13 = 0 Ta thay α = 1 thỏa mãn. V y có the viet phương trình dưới dạng (x2 +1)2 = 5x2 +10x+5 ⇔ (x +1)2 = [ √ 5(x+1)]2 ⇔ x2 + 1 = √ 5(x + 1) x2 + 1 = − √ 5(x + 1) Giải tàng trường hợp suy ra nghi m của phương trình đã cho là x1,2 = 5 ± 1 + 4 5 . 2 Bài toán 2.26. Giải phương trình x2 Lài giai. Đieu ki n x = 1. Viet phương trình đã cho dưới dạng x2 + (x + 1)2 = 3. x 2 x2 x2 2 x2 (x − x + 1 ) = 3 − 2. x + 1 ⇔ ( x + 1 ) + 2. x + 1 − 3 = 0 x2 x 1 = 0 ⇔ x2 + 3x + 3 = 0 (vô nghi m) ⇔ x1, x2 = 1 ± √ 5 . 2 V y phương trình đã cho có hai nghi m x1 , x2 = 1 ± √ 5 . 2 Bài toán 2.27 (VMO 1973). Giải phương trình x4 = 4x + 1. Lài giai.Viet phương trình đã cho dưới dạng x4 + 2x2 + 1 = 4x + 1 + 2x2 + 1 (x2 + 1)2 = 2(x + 1)2 ⇔ (x2 + 1)2 − [ √ 2(x + 1)]2 = 0 ⇔ [x + 1 + √ 2(x + 1)][x + 1 − √ 2(x + 1)] = 0 ⇔ x2 + 1 − √ 2(x + 1) = 0 ⇔ x 1,2 = √ 2p 4 √ 2 − 2 . 2 " 2
  • 39. 37 Tải tài liệu tại sividoc.com ≤ 2 2 2 2 2 2 2 2 3 x + 1 = 4 y + 1 = 5 z + 1 2.3 M t so h phương trình đưa ve phương trình b c ba và b c bon Nh n xét rang, neu như x2 +y2 = 1 thì ta sả dụng phép the x = sin α, y = cos α. Trong nhieu trường hợp, phép the này sě tỏ ra hi u quả. Bài toán 2.28. Giải h phương trình ( x2 + y2 = 1 . 4xy(2y2 − 1) = 1 Lài giai. Đ t x = sin α, y = cos α với 0 α < π thì ta được sin 4α = 1. π Nghĩa là 4α = + k2π với k = 0, 1, 2, 3. Tà đó có the de dàng suy ra 2 √ 2 − √ 2 , √ 2 + √ 2 ! , √ 2 + √ 2 , − √ 2 − √ 2 ! , − √ 2 − √ 2 , − √ 2 + √ 2 ! , − √ 2 + √ 2 , √ 2 − √ 2 ! . Ta thay phép the này giải quyet bài toán rat nhanh gon, trong khi đó các cách giải bài toán không sả dụng phép the lượng giác sě phác tạp hơn nhieu. Neu ta có h thác dạng xy + yz + zx = 1 thì ton tại α, β, γ sao cho x = tan α , y = tan 2 β γ , z = tan 2 2 α và α + β + γ = π. β Th t v y, đ t x = tan 2 , y = tan 2 , (−π < α, β < π), thì z = cot = tan − . α + β π α + β 2 2 2 Tiep theo, đ t γ = π − (α + β). Bài toán 2.29. Giải h phương trình x y z xy + yz + zx = 1
  • 40. 38 Tải tài liệu tại sividoc.com − 3 2 3 2 xy + yz + zx = 1 Lài giai. Vì x 3(x2 + 1) = y 4(y2 + 1) = z 5(z2 + 1) nên x, y, z có cùng dau, ngoài ra, neu (x, y, z) là nghi m của h thì (−x, −y, −z) cũng là nghi m. Như v y, ta chỉ can đi tìm các nghi m dương. Đ t x = tan α β , y = tan , 2 2 γ z = tan 2 (0 < α, β, γ < π, α+β +γ = π), ta được sin α 3 sin β = 4 sin γ = . 5 Tà định lý hàm so sin bây giờ suy ra α, β, γ là các góc của tam giác có đ® dài các cạnh tương áng là 3, 4, 5. Tam giác này là tam giác vuông có π γ = , sin α = 2 3 , sin β = 5 4 α . Vì the tan 5 2 1 β = , tan 3 2 1 γ = , tan 2 2 = 1. Như v y đáp so của bài toán là 1 , 1 , 1 , − 1 , − 1 , −1 . Bài toán 2.30. Cháng minh rang giǎa 7 so thực bat kỳ luôn tìm được hai so x và y sao cho 0 < x − y 1 + xy 1 √ 3 . Lài giai. Goi các so đã cho là a1, a2, . . . , a7. Với moi so thực a, ton tại so α thu®c khoảng − π , π sao cho a = tan α. Giả sả a = tan α , 2 2 1 1 a2 = tan α2, . . . , a7 = tan α7. π Trong 7 so α1, α2, . . . , α7 ton tại hai so có hi u không vượt quá 6 . Giả sả hai so này là α và β, trong đó α > β. Khi đó tan α − tan β π 1 0 < 1 + tan α tan β = tan(α − β) ≤ tan 6 = √ 3 . Như v y các so x = tan α và y = tan β là các so can tìm. Lời giải của bài toán trở nên hien nhiên sau khi ta chon được m®t phép the lượng giác thích hợp. Bản chat của phép the lượng giác nam ở cho các bien so có m t trong bài toán được xét như giá trị của các hàm lượng giác nào đó. Trong đó các hàm so lượng giác can được chon đe bieu thác thu được càng gon càng tot. Bài viet này nói ve các phép the lượng giác. Bài toán 2.31. Giải h phương trình 1 x2 1 + x2 = 2y 1 + y2 = 1 − z2 . 1 + z2 ≤
  • 41. 39 Tải tài liệu tại sividoc.com 1 = cos 1 5 5 5 2 3 3 3 Chương 3. Phương pháp lư ng giác giải phương trình đa thfíc b c cao Trong chương này ta xét m®t so dạng phương trình b c cao và h phương trình tương áng giải được bang phương pháp lượng giác ho c sả dụng các đȁng thác đại so sinh bởi các h thác lượng giác. 3.1 Phương trình đa thfíc b c cao Tiep theo, ta xét m®t so áng dụng khác của đȁng thác đại so - lượng giác. 1 Bài toán 3.1. Giải phương trình T5(x) = Chebyshev loại 1 b c 5. Nh n xét 3.3. Đe ý rang, tà đȁng thác 2 , trong đó T5(x) là đa thác ta suy ra cos 5t + cos t = 2 cos 3t cos 2t, T5(x) = 16x5 − 20x3 + 5x, x ∈ [−1, 1]. Lài giai. Đe ý rang T5(x) = cos(5 arccos x), x ∈ [−1, 1] và T5(x) < −1 khi x < −1, T5(x) > 1 khi x > 1. V y nên chỉ can tìm nghi m của phương trình đã cho trong (−1, 1). Ta có = cos = cos ± 2π = cos ± 4π . 1 π π π Tà đó, suy ra phương trình đã cho có 5 nghi m phân bi t π x , x = cos π3 ± 2π , x = cos π3 ± 4π . 2,3 4,5
  • 42. 40 Tải tài liệu tại sividoc.com − 5 2 + ) a 2 a5 Bài toán 3.2. Giải phương trình 16x5 20x3 + 5x = 5 . 2 Nh n xét 3.4. Đe ý rang, tà đȁng thác cos 5t + cos t = 2 cos 3t cos 2t, ta suy ra 16x5 − 20x3 + 5x = T5(x), x ∈ [−1, 1]. Lài giai. Đe ý rang T5(x) = cos(5 arccos x), x ∈ [−1, 1] và T5(x) < −1 khi x < −1, T5(x) > 1 khi x > 1. V y nên chỉ can tìm nghi m của phương trình đã cho trong (1, +∞). 5 1 1 Sả dụng các đȁng thác = 2 + và 4 2 T 1 (a 1 2 1 a5 + 1 , suy ra phương trình có nghi m x1 = 1 √ 5 2 1 2 + √ 5 2 . Ta cháng minh phương trình đã cho có nghi m duy nhat trong (1, +∞). Th t v y, ta thay T′(x) = 80x4 − 60x2 + 5 > 0, ∀x ∈ (1, +∞) nên ve trái của phương trình đã cho là hàm đong bien trong (1, +∞). H quả 3.1. Xét phương trình 16x5 − 20x3 + 5x = m. (∗) Khi đó - với m ∈ (−1, 1) thi phương trình (*) có 5 nghi m phân bi t, - với m > 1 ho c m < −1 thi phương trình (*) có 1 nghi m duy nhat, - với m = √1 thì phương trình có 1 nghi m đơn x = 1 và 2 nghi m kép x1,2 = −1 ± 5 . 4 - với m = √ −1 thì phương trình có 1 nghi m đơn x = −1 và 2 nghi m kép x1,2 = 1 ± 5 . 4 =
  • 43. 41 Tải tài liệu tại sividoc.com 2 2 2 Bài toán 3.3. Cháng minh rang phương trình 64x6 − 96x4 + 36x2 − 3 = 0 có nghi m thực x0 thỏa mãn đieu ki n q 2 + √ 2 + √ 2 < x0 < q 2 + √ 2 + √ 3 . Lài giai. Tà công thác cos2 α = 1 + cos 2α 2 (với 0 ≤ α ≤ π), ta suy ra α 1 + cos ‚ .1 + r 1 + cos α q α cos 4 Khi α = = 2 π , ta có 2 = , 2 = 1 2 + √ 2 + 2 cos α. 2 2 4 Khi α = π , ta có π cos 16 = 1 r 2 + q 2 + √ 2. 6 M t khác, ta cũng có π cos 24 = 1 r 2 + q 2 + √ 3. Suy ra cos 6t = 4 cos3 2t − 3 cos 2t = 4(2 cos2 t − 1)3 − 3(2 cos2 t − 1) = 32 cos6 t − 48 cos4 t + 18 cos2 t − 1 64 cos6 t − 96 cos4 t + 36 cos2 t − 3 = 2 cos 6t − 1. Tà phương trình 64x6 − 96x4 + 36x2 − 3 = 0, ta xét x ∈ [1; 1] và đ t x = cos t, ta có 2 cos 6t − 1 = 0 ⇒ cos 6t = π 1 π 2 ⇒ t = 18 . Do đó x0 = cos 18 là m®t nghi m của phương trình. M t khác π π π < < 24 18 16 2 ,
  • 44. 42 Tải tài liệu tại sividoc.com 2 (x + n) + — − hay π cos 24 π > cos 18 π > cos . 16 V y phương trình 64x6 −96x4 +36x2 −3 = 0 luôn có m®t nghi m x0 thỏa mãn đieu ki n π = cos 18 q 2 + √ 2 + √ 2 < x0 < q 2 + √ 2 + √ 3 . Bài toán 3.4. Cho b® so m, n, p ∈ R. Giải phương trình x3 + m3 x3 + n3 3 3 x − m x − n x − p (x + m)3 + 3 x3+p3 (x+p)3 — 2 + 2 x + m x + n x + p = 0. Lài giai. Nh n xét rang x3 + m3 1 3 (x − m)2 (x + m)3 = 4 + 4 (x + m)2 . Vì v y phương trình đã cho tương đương với phương trình sau 1 3 (x − m)2 1 3 (x − n)2 1 3 (x − p)2 4 + 4 (x + m)2 + 4 + 4 (x + n)2 + 4 + 4 (x + p)2 − − 3 + 3 x − m x − n x − p = 0. (1) 2 2 x + m x + n x + p Đ t và đe ý rang x − m x + m = a, x − n x + n = b, x − p = c x + p 1 + 3 a2 + 1 + 3 b2 + 1 + 3 c2 − 4 4 4 4 4 4 3 3 − 4 + 2 abc = 0 có the bien đői được ve dạng (ab + c)2 = (1 − a2 )(1 − b2 ). (2) Thay các giá trị a, b, c theo bien x, m, n, p ta được (ab + c)2 = 4[x3 + (mn mp np)x]2 (x + m)2(x + n)2(x + p)2 , 2
  • 45. 43 Tải tài liệu tại sividoc.com Σ ∈ 1 − a2 = 4mx , 1 − b2 = 4nx . V y (2) có dạng (x + m)2 (x + n)2 x2 [x2 +2(x+p) √ mn+mn−mp−np][x2 −2(x+p) √ mn+mn−mp−np] = 0. Giải ra ta được các nghi m của phương trình là x1 = x2 = 0, x3,4 = ±( √ mp − √ np) − √ mn, x5,6 = √ mn ± ( √ mp + √ np. π Bài toán 3.5. Cho 0 < α < Q(x) ∈ R[x] b c n thì đa thác n + 2 . Cháng minh rang với moi đa thác P (x) = (x2 − 2x cos α + 1)Q(x) không the có tat cả các h so đeu không âm. Lài giai. Giả sả và Khi đó Q(x) = a0xn + a1xn−1 + · · · + an−1x + an P(x) = b0xn+2 + b1xn+1 + · · · + bn+1x + bn+2. Suy ra b0 = a0, b1 = a1 − 2a0 cos α, b2 = a2 + a0 − 2a1 cos α, · · · bn+1 = an−1 − 2an cos α, bn+2 = an. bk = ak + ak−2 − 2ak−1 cos α, an+2 = an+1 = 0, a−1 = a−2 = 0 và n+2 bk sin kα = 0. k=0 Mà sin kα > 0 vì α 0, π nên ton tại h so b n + 2 j < 0.
  • 46. 44 Tải tài liệu tại sividoc.com · · · 2 √ √ 0 0 1 1 n n . . . . . . . . . . .. Bài toán 3.6. Cho a0, a1, . . . , an là n + 1 so đôi m®t khác nhau. Giải h phương trình sau x0 + x1a0 + x2a2 · · · + xnan = 0 x0 + x1a1 + x2a2 + · · · + xnan = 0 x0 + x1an + x2a2 + · · · + xnan = 0 Lài giai. Xét đa thác f(y) = xnyn + xn−1yn−1 + · · · + x1y + x0. Ta có deg f ≤ n. Tà h (1) ta có f(a0) = f(a1) = · · · = f(an) = 0, nên f(y) có n + 1 nghi m phân bi t, do đó f(y) ≡ 0. Tà đó suy ra x0 = x1 = · · · = xn = 0. Thả lại ta thay x0 = x1 = = xn = 0 thoả mãn h đã cho. V y h có nghi m duy nhat (x0, x1, . . . , xn) = (0, 0, . . . , 0). Bài toán 3.7. Giải phương trình √ 1 − x2 = 4x3 − 3x. Lài giai. Đ t x = cos α, 0 ≤ α ≤ π. Ta được sin α = cos 3α, hay cos 3α − cos π − α = 0. Tà đó 2 sin π − 2α sin α + π = 0. 4 π Đen đây ta có the de dàng tìm được α = 8 4 5π ho c α = 8 ho c α = 3π . Vì 4 π cos = 8 1 + cos π 4 = 2 2 + 2 , 2 ‚ . , (1)
  • 47. 45 Tải tài liệu tại sividoc.com − 4 2 = 2 − 2 , 2 ‚ . ,1 + cos 5π √ √ cos 3π √ 2 4 = − 2 , nên t p hợp (√ 2 + √ 2 , − √ 2 − √ 2 , − √ 2 ) là t p nghi m của phương 2 2 2 trình. Bài toán 3.8. Giải phương trình x 35 x + √ x2 − 1 = 1 Lài giai. Chú ý rang x > 1. Đ t x = . 12 , 0 < α < π . Phương trình có the viet lại thành sin α 2 1 1 35 + = . sin α cos α 12 2 t2 − 1 Đ t sin α + cos α = t thì 1 + 2 sin α cos α = t Thay vào ta được , suy ra sin α cos α = . 2 2t 35 = 2 7 5 ⇔ 35t − 24t − 35 = 0 ⇔ t = ∨ t = − . t2 − 1 12 5 7 7 Vì t = sin α + cos α > 0 nên ta loại nghi m thá hai. Xét t = 5 12 thì tà đây ta tính được sin α cos α = trình . Như v y sin α, cos α là nghi m của phương 25 X2 − 7 X + 12 = 0. 5 25 3 4 Tà đây ta tính được sin α = 5 5 ho c sin α = 5 . Tương áng ta được nghi m 5 của phương trình là x = 3 và x = . 4 Bài toán 3.9. Giải phương trình 8x3 − 4x − 1 = √ 3 6x + 1. 5π cos 8 = −
  • 48. 46 Tải tài liệu tại sividoc.com ( − − ( − − 3 | | ≤ ∈ | | | − | | | − x 2 — − − 2 Lài giai. Phương trình đã cho tương đương (2x)3 − 4x − 1 = √ 3 2x + 4x + 1. Đ t u = 2x, v = √ 3 2x + 4x + 1, ta có h phương trình u3 4x 1 = v v3 − 4x − 1 = u ⇔ u3 v3 = v u u3 − 4x − 1 = v ⇔ (u − v)(u2 + uv + v2 + 1) = 0 u3 − 4x − 1 = v u = v u3 − 4x − 1 = u ⇔ 8x − 6x = 1. (1) Neu x > 1 thì 8x3 6x = 2 x (4x2 3) > 2 nên (1) vô nghi m. Do v y ta phải có x 1. Đieu này cho phép ta đ t x = cos t với t [0, π]. Khi đó phương trình (1) có the viet lại thành cos 3t = 1 2 ⇔ t1 = π 9 ∨ t2 = 5π 7π 9 ∨ t3 = 9 . π 5π 7π V y phương trình đã cho có ba nghi m x = cos , x = cos 9 , x = cos . 9 9 Bài toán 3.10. Tìm x ∈ (0; 1) thỏa mãn đieu ki n 32x(x2 − 1)(2x2 − 1)2 = 1 − 1 . Lài giai. Vì x ∈ (0; 1) nên ta có the đ t x = cos α với α ∈ 0; π . Ta có 32 cos α(cos2 α 1)(2 cos2 α 1)2 = 1 1 cos α ⇔ − 32 cos α sin α cos2 1 2α = 1 − cos α ⇔ 8 sin 2 2α cos2 2α = 1 − cos α ⇔ 2 sin 4α = 1 − cos α ⇔ cos α = cos 8α. Do đó α = k2π ho c α = 7 l2π 9 (với k, l ∈ Z.) Vì rang α ∈ 0; π nên k = 1, l = 1 và l = 2. ( ( ⇔ 2 2
  • 49. 47 Tải tài liệu tại sividoc.com | | ≤ − − Với t = √ 2 thì π 2 π 2 3 — 2 V y phương trình đã cho có ba nghi m thu®c (0; 1) đó là x = cos 2π ; x = cos 7 2π 4π ; x = cos . 9 9 Bài toán 3.11. Giải phương trình x3 + q (1 − x2)3 = x q 2(1 − x2). Lài giai. Đieu ki n đehcác bieiu thác có nghĩa: −1 ≤ x ≤ 1. sin3 α + cos3 α = √ 2 sin α cos α ⇔ (sin α + cos α)3 − 3 sin α cos α(sin α + cos α) − √ 2 sin α cos α = 0. Đ t sin α + cos α = √ 2 sin π + α = t với đieu ki n t √ 2. 4 t2 − 1 Suy ra sin α cos α = 2 và phương trình trở thành t − 3 t2 1 2 t − t2 1 2 = 0 2 ⇔ t3 + √ 2t2 − 3t − √ 2 = 0 ⇔ (t − √ 2)(t + √ 2 − 1)(t + √ 2 + 1) = 0 Suy ra t = √ 2 ho c t = 1 − √ 2 do |t| ≤ √ 2 . √ 2 sin π + α = √ 2 ⇔ sin π + α = 1 hay α = 4 4 π + k2π. 4 Vì α ∈ h − π ; π i π nên α = π √ 2 và do đó x = sin = . hay Với t = 1 2√2 , suy ra 4 4 2 sin α + cos α = 1 − √ 2 x + √ 1 − x2 = 1 − √ 2, Đ t x = sin α (với α ∈ − ; ), thì phương trình trở thành √
  • 50. 48 Tải tài liệu tại sividoc.com 2 — ∈ 2 t 2 2 2 2 2 2 2 2 2 2 s 3 − tác x ≤ 1 − √ 2 ⇔ x ≤ 1 − √ 2 1 − x2 = (1 − √ 2 − x)2 x2 − (1 − √ 2)x + (1 − √ 2) = 0 ⇔ x = 1 − √ 2 + √ 2 √ 2 − 1 . V y phương trình đã cho có 2 nghi m là x = √ 2 ; x = 1 − √ 2 + √ 2 √ 2 − 1 . 2 2 Bài toán 3.12. Giải phương trình q 1 + √ 1 − x2 hq (1 + x)3 − q (1 − x)3 i = 2 + √ 1 − x2. Lài giai. Đieu ki n có nghĩa: 1 ≤ x ≤ 1. Đ t x = cos t (với t [0; π]), phương trình trở thành hay √ 1 + sin t q (1 + cos t)3 − q (1 − cos t)3 = 2 + √ 1 − cos2 t s sin t + cos t 2 s 2 cos2 t 2 sin2 2 3 2 = 2 + √ sin2 t (3.1) t Vì t ∈ [0; π] nên 2 ∈ h 0; πi . Do đó t t sin 2 ≥ 0, cos 2 ≥ 0. V y nên phương trình (3.1) tương đương với 2 √ 2 sin t + cos t cos3 t − sin3 t = 2 + sin t ⇔ 2 √ 2 sin t + cos t cos t − sin t ×
  • 51. 49 Tải tài liệu tại sividoc.com 2 2 2 — − ≤ ≤ — ≤ ≤ x + 3y = 4y3 × cos2 t + sin t t cos 2 2 + sin2 t = 2 + sin t 2 ⇔ 2 √ 2 cos t 1 + 1 sin t = 2 + sin t ⇔ √ 2 cos t(2 + sin t) = 2 + sin t ⇔ ( √ 2 cos t − 1)(2 + sin t) = 0 ⇔ cos t = √ 2 ⇔ x = √ 2 2 √ 2 V y phương trình có nghi m là x = . 2 3.2 H phương trình đa thfíc b c cao Phép the lượng giác thường có ích trong phép giải các h phương trình hoán vị vòng quanh. Ta xem xét m®t ví dụ như v y. Bài toán 3.13. H phương trình sau có bao nhiêu nghi m? y + 3z = 4z3 . z + 3x = 4x3 Lài giai. Ta viet lại h dưới dạng x = 4y3 − 3y y = 4z3 − 3z . z = 4x3 − 3x Ta cháng minh rang tat cả các so x, y, z theo trị tuy t đoi không vượt quá 1. Th t v y, giả sả x là so lớn nhat trong các so này và x > 1 thì ta có z = 4x3 − 3x > x. Ta đi đen mâu thuan. Neu giả sả x là so nhỏ nhat và x < 1 thì ta cũng có z = 4x3 3x < x, mâu thuan. Như v y 1 x, y, z 1 và ta có the thực hi n phép the x = cos α (0 α π). Khi đó z = cos 3α, y = cos 9α, x = cos 27α. Bây giờ rõ ràng rang so nghi m của h phương trình ban đau bang so nghi m của phương trình cos α = cos 27α trên [0, π]. De dàng thay rang so nghi m này đúng bang 27: kπ α = , k = 0, 1, 2, . . . , 13; α = 13 kπ , k = 1, 2, . . . , 13. 14
  • 52. 50 Tải tài liệu tại sividoc.com ≤ 2 2 2 2 2 2 2 2 3 x + 1 = 4 y + 1 = 5 z + 1 3 2 3 2 Nh n xét 3.5. Neu như trong bài toán có đieu ki n x2 + y2 = 1 thì phép the x = sin α, y = cos α trong nhieu trường hợp sě tỏ ra hi u quả. Bài toán 3.14. Giải h phương trình x2 + y2 = 1 4xy(2y2 − 1) = 1 Lài giai. Ta đ t x = sin α, y = cos α với 0 α < π thì ta được π sin 4α = 1. Nghĩa là 4α = tìm ra đáp so cuoi cùng + k2π với k = 0, 1, 2, 3. Tà đó có the de dàng 2 √ 2 − √ 2 , √ 2 + √ 2 ! , √ 2 + √ 2 , − √ 2 − √ 2 ! , − √ 2 − √ 2 , − √ 2 + √ 2 ! , − √ 2 + √ 2 , √ 2 − √ 2 ! . Ta thay phép the này giải quyet bài toán rat nhanh gon, trong khi đó các cách giải bài toán không sả dụng phép the lượng giác phác tạp hơn nhieu. Bài toán 3.15. Giải h phương trình x y z . xy + yz + zx = 1. Lài giai. Vì x 3(x2 + 1) = y 4(y2 + 1) = z 5(z2 + 1) nên x, y, z có cùng dau, ngoài ra, neu (x, y, z) là nghi m của h thì (−x, −y, −z) cũng là nghi m. Như v y ta chỉ can đi tìm các nghi m dương. Đ t x = tan α β , y = tan , 2 2 γ z = tan 2 (0 < α, β, γ < π, α+β +γ = π), ta được sin α 3 sin β = 4 sin γ = . 5 Tà định lý hàm so sin bây giờ suy ra α, β, γ là các góc của tam giác có đ® dài các cạnh tương áng là 3, 4, 5. Tam giác này là tam giác vuông có π γ = , sin α = 2 3 , sin β = 5 4 α . Vì the tan 5 2 1 β = , tan 3 2 1 γ = , tan 2 2 = 1. Như v y đáp so của bài toán là 1 , 1 , 1 , − 1 , − 1 , −1 . (
  • 53. 51 Tải tài liệu tại sividoc.com r 2 b a Chương 4. M t so dạng toán liên quan 4.1 Phép the lư ng giác 4.1.1 Phép the lư ng giác trong bat đang thfíc Chúng ta sě xem xét các ví dụ sả dụng phép the lượng giác đe cháng minh m®t so dạng bat đȁng thác. Bài toán 4.1. a, b, c, d là các so dương. Cháng minh bat đȁng thác √ ab + √ cd ≤ q (a + d)(b + c). Lài giai. Viet lại bat đȁng thác dưới dạng a a + d b · b + c + c b + c d · a + d ≤ 1. a Đ t a + d = sin2 α, b b + c = sin2 β 0 < α, β < π . Khi đó bat đȁng thác có the đưa ve dạng sin α sin β + cos α cos β ≤ 1, hay là cos(α − β) ≤ 1. Như ta đã thay phép the lượng giác đã giúp chúng ta phá được các dau căn thác và đưa ve dạng đơn giản hơn. Trong ví dụ dưới đây chúng ta cũng sả dụng phép the lượng giác đe đưa bat đȁng thác ve dạng đơn giản hơn. Bài toán 4.2. Cho a, b, c là các so dương, trong đó c là so nhỏ nhat trong chúng. Cháng minh các bat đȁng thác c + c √ ab ≤ q (a + c)(b + c) + q (a − c)(b − c) ≤ 2 √ ab. r
  • 54. 52 Tải tài liệu tại sividoc.com 4 a b a b a — b ≤ 1 − a Lài giai. Viet lại bat đȁng thác dưới dạng c + c ≤ r 1 + c 1 + c + r 1 − c 1 c 2. c Đ t a c = sin 2α, b = sin 2β, 0 < α, β ≤ π , bat đȁng thác trở thành sin 2α+sin 2β ≤ (sin α+cos α)(sin β+cos β)+(cos α−sin α)(cos β−sin β) ≤ 2, hay 2 sin(α + β) cos(α − β) ≤ 2 cos(α − β) ≤ 2. Chúng ta cũng lưu ý rang ve phải của bat đȁng thác này q (a + c)(b + c) + q (a − c)(b − c) ≤ 2 √ ab là m®t h quả đơn giản của bài toán 3. Bài toán 4.3. Cháng minh rang với moi so tự nhiên n (n ≥ 2) và với moi a, ta có −(1 + a2 )n ≤ (2a)n + (1 − a2 )n ≤ (1 + a2 )n . Lài giai. Bat đȁng thác can cháng minh tương đương với −1 ≤ α 2a n 1 + a2 + 2 n 1 + a2 ≤ 1 Đ t tan 2 = a với −π < α < π, ta có 2a 1 − a2 1 + a2 = sin α; 1 + a2 = cos α. Bat đȁng thác can cháng minh có dạng −1 ≤ sinn α + cosn α ≤ 1. Th t v y ta có −1 ≤ sin α ≤ 1 ⇒ − sin2 α ≤ sinn α ≤ sin2 α, với ∀n ≥ 2. Tương tự, ta cũng có −1 ≤ cos α ≤ 1 ⇒ − cos2 α ≤ cosn α ≤ cos2 α, với ∀n ≥ 2. Do đó −1 ≤ sinn α + cosn α ≤ 1, bài toán được cháng minh.
  • 55. 53 Tải tài liệu tại sividoc.com c b n − 1 2 n 1 2 n a k Bài toán 4.4. Cho a, b, c là các so dương, trong đó c là so nhỏ nhat. Cháng minh các bat đȁng thác . − c . √ ab ≤ q c(a − c) + q c(b − c) ≤ √ ab. 4.1.2 Phép the lư ng giác trong dãy so Cuoi cùng ta đi đen các phép the lượng giác trong dãy so. Ta xem xét hai bài toán khó mà có the đưa ve các dãy truy hoi. Lời giải các h thác truy hoi này thu được nhờ vào các phép the lượng giác. Bài toán 4.5. a1, a2, . . . , an là các so thực sao cho a2 + a2 + · · · + a2 = 1. 1 2 n Tìm giá trị lớn nhat của bieu thác a1a2 + a2a3 + · · · + an−1an. Lài giai. Ta xét các so C sao cho bat đȁng thác a1a2 + a2a3 + · · · + an−1an ≤ C(a2 + a2 + · · · + a2 ) đúng với moi so thực a1, a2, . . . , an. So C nhỏ nhat trong các so như v y sě là đáp so can tìm. Thá nhat, so C can tìm không vượt quá 1, vì a2 + a2 + · · · + a2 − (a1a2 + a2a3 + · · · + an−1an) = 1 2 n 1 2 2 2 2 = 2 [a1 + (a1 − a2) + · · · + (an−1 − an) + an] ≥ 0. Ta bien đői bieu thác C(a2 + a2 + · · · + a2 ) − (a1a2 + a2a3 + · · · + an−1an) bang cách liên tiep tách bình phương đúng. Ta thu được bieu thác có dạng p1 a1 − 1 2p1 2 a2 +p2 a2 − 1 2p2 2 a3 +...+pn−1 1 an−1 − 1 2pn−1 2 an +pna2 . De thay p1 = C và pk+1 = C − 4p với moi k = 1, 2, . . . , n − 1. Bieu thác thu được không âm với moi a1, a2, . . . , an khi và chỉ khi tat cả các so p1, p2, . . . , pn không âm. Như v y bài toán đưa ve vi c tìm so C sao cho tat cả các so hạng của dãy so p1, p2, . . . , pn đeu không âm. Vì 0 < C ≤ 1 nên ta có the đ t C = cos α, trong đó 0 ≤ α < đó π . Khi 2 1 p2 = cos α − 4 cos α = 4 cos2 α 1 = 4 cos α 2 cos α sin 2α − sin α = 2 sin 2α sin 3α . 2 sin 2α
  • 56. 54 Tải tài liệu tại sividoc.com − k n 2 Tiep theo p3 sin 2α = cos α − 2 sin 3α = 2 cos α sin 3α − sin 2α 2 sin 3α sin 4α = . 2 sin 3α Bang quy nạp, de dàng cháng minh được rang sin(k + 1)α pk = , k = 1, 2, . . . , n. 2 sin kα Như v y p1, p2, . . . , pn không âm khi và chỉ khi sin α, sin 2α, . . . , sin(n+1)α π π không âm. Như v y 0 ≤ α ≤ và giá trị C can tìm bang cos . n + 1 n + 1 Bài toán 4.6. x1, x2, . . . , xn là các so dương. Goi A là so nhỏ nhat trong 1 các so x1, x2 + 1 1 , x3 + 2 , . . . , xn + x 1 , n−1 1 , còn B là so lớn nhat trong xn các so này. Cháng minh rang giá trị lớn nhat của A bang giá trị nhỏ nhat của B và hãy tìm giá trị này. Lài giai. Đau tiên ta xét tình huong 1 x1 = x2 + 1 1 = x3 + 2 = · · · = xn + x 1 1 = . n−1 xn 1 Khi đó các so x1, x2, . . . , xn thoả mãn h thác truy hoi xk+1 = x1 − x , k = 1, 2, . . . , n 1. H thác tương tự ta đã g p ở ví dụ trước. H thác này cũng có the giải bang phép the lượng giác. Đau tiên ta cháng minh rang x1 < 2. Th t v y, neu x1 ≥ 1 2 thì ta lan lượt có x2 ≥ 1, x3 ≥ 1, . . . , xn ≥ 1 và lúc đó đȁng thác x1 = x không the xảy ra. Bây giờ ta có the đ t x1 = 2 cos α 0 < α < . Tương tự như ở bài π toán trước, bang quy nạp de dàng cháng minh được rang xk = (1 ≤ k ≤ n). sin(k + 1)α sin kα 1 Vì x1 = n nên tà đây ta có 2 cos α = sin nα . sin(n + 1)α x x x x x
  • 57. 55 Tải tài liệu tại sividoc.com 3 n 4 2 Tà đây sin(n + 2)α = 0, tác là α = π . n + 2 Bây giờ ta cháng minh rang giá trị lớn nhat của A và giá trị nhỏ nhat π của B bang 2 cos n + 2 . Ta chỉ can cháng minh bat đȁng thác π A ≤ 2 cos n + 2 ≤ B. 1 Giả sả tat cả các so x1, x2 + x 1 , x3 + x 1 , . . . , xn + x 1 , đeu lớn hơn x π 1 2 n−1 n 2cos n + 2 . Trong trường hợp này ta lan lượt có các bat đȁng thác sau 3π sin 4π sin sin (n + 1)π x2 > n + 2 , x > 2π sin n + 2 n + 2 , . . . , x > 3π sin n + 2 n + 2 nπ sin n + 2 Nhưng khi đó 1 < 2 cos xn π n + 2 . Mâu thuan. π Như v y ta đã cháng minh được bat đȁng thác A ≤ 2 cos n + 2 . Bat π đȁng thác 2 cos n + 2 ≤ B cháng minh hoàn toàn tương tự. 4.2 M t so dạng toán tfi các đe thi Olympic sfi dnng phương pháp lư ng giác Bài toán √ 4.7 (IMO - 196 √ 5). Tìm tat cả các giá trị của x ∈ [0; 2π] sao cho 2 cos x ≤ | 1 + sin 2x − 1 − sin 2x| ≤ 2. Lài giai.√ √ Đ t y = | 1 + sin 2x − 1 − sin 2x| ⇒ y2 = 2 − 2| cos 2x|. Neu cos x ≥ 0 mà 2 cos x ≤ 2 ⇒ | cos 2x| = |2 cos2 x−1| = 1−2 cos2 x. 1 hπ πi 3π 7π Khi đó đe bài trở thành cos x ≤ Neu cos x < 0 √ 2 ⇒ x ∈ 4 ; 2 ∪ 4 ; 4 . +) Giả sả | cos x| ≤ 5π ; 3π . 1 √ 2 ta có cos x ≤ | cos x| ≤ 1 √ 2 ⇒ x ∈ π 3π ; 2 4 . ∪
  • 58. 56 Tải tài liệu tại sividoc.com r r r / r s r r s 4 4 sin2 y cos2 x x + y sin2 y cos2 x 2 ! 2 ! | | +) Giả sả | cos x| > x ∈ 3π ; 5π . 1 √ 2 ta có | cos 2x| = 2 cos x−1 ⇒ y = 2| sin x| ⇒ V y bat phương trình có nghi m là x ∈ π ; 7π . Bài toán 4.8 (VMO - 2013). Giải h phương trình sin2 x + 1 sin2 x + cos2 y + 1 cos2 y = 20x x + y r sin2 y + 1 + r cos2 x + 1 = r 20y Đieu ki n có nghĩa: sin x, cos x, siny, cos y = 0, xy > 0. Nhân ve với ve của hai phương trình trong h ta được sin2 x + 1 sin2 x + s sin2 y + 1 + cos2 y + 1 cos y + r cos2 x + 1 ! = 20 xy . (3) (x + y)2 Áp dụng bat đȁng thác Cauchy - Schwarz và AM - GM, ta có sin2 x + 1 sin2 x + s sin2 y + 1 + cos2 y + 1 cos y + r cos2 x + 1 ! sin2 y cos2 x 1 2 ≥ | sin x. cos x| + | sin x. cos x| sin 2x 1 3 2 = + + 2 2| sin 2x| 2| sin 2x| 3 2 5 2 1 + = . 2 2 Hoàn toàn tương tự ta có s sin2 y + 1 sin2 y + r cos2 x + 1 cos2 x 5 2 2 Lài giai. 4 4 2 ≥ ≥ .
  • 59. 57 Tải tài liệu tại sividoc.com r ≥ sin2 x cos2 x sin2 y cos2 x 4 2 4 2 n n 8m4 − 8m2 + 1 = 1 a4 + 1 , với m = 1 a + 1 . (1) un+1 = 8u4 − 8u2 + 1 2 2 a4 3 2 a42 Xác định so hạng tőng quát vn. Do đó theo bat đȁng thác AM - GM ta có V T (3) ≥ 4 s 4 sin2 x + 1 . cos2 x + 1 sin2 y + 1 cos2 x + 1 ≥ 4 r 4 ( 5 )4 2 = 10 20 xy (x + y)2 = V P (3). π π Dau bang xảy ra khi | sin 2x| = 1; x = y ⇔ x = y = 4 + k 2 , k ∈ Z. Thả lại, ta thay nghi m này thỏa mãn. V y nghi m của h phương trình là (x; y) = π + k π ; π + k π ; k ∈ Z. Bài toán 4.9. Cho dãy so (vn)n thỏa mãn đieu ki n v1 = 5 vn+1 = v4 − 4v2 + 2 Lài giai. Sả dụng hang đȁng thác sau Đ t vn 2 2 a4 2 a = un ta thu được u1 = = a + , a = (5 + 5 1 1 1 √ 21), 2 2 a 2 (2) n n 1 Theo (1) thì u = a4 + 1 , u = 1 a42 + 1 ,. . . Bang phương pháp quy nạp, ta thu được u = 1 a + 1 , a = 1 (5 + √ 21), 2 a 2 u = 1 a4n−1 + 1 V y vn = 5 + √ 21 4n−1 5 − √ 21 4n−1 2 2 1 n+1 a4n−1 +
  • 60. 58 Tải tài liệu tại sividoc.com , n = 1, 2, . . . 2
  • 61. 59 Tải tài liệu tại sividoc.com ≤ ≤ — ≤≤ ≤ 2 2 2 2 2 3 t Bài toán 4.10 (VMO - 1984 - Bảng A). Giải phương trình q 1 + √ 1 − x2 q (1 + x)3 − q (1 − x)3 = 2 + √ 1 − x2. Lài giai. Đieu ki n có nghĩa: 1 x 1. Đ t x = cos t, 0 t π Phương trình trở thành √ 1 + sin t q (1 + cos t)3 − q (1 − cos t)3 = 2 + sin t s t t 2 3 t √ cos + sin 2 2 cos 2 − sin 2 .2 2 = 2 + sin t ⇔ cos2 t − sin2 t 1 + cos t sin t .2 √ 2 = 2 + sin t ⇔ cos t(2 + sin t) √ 2 = 2 + sin t ⇔ (2 + sin t)( √ 2 cos t − 1) = 0 ⇔ cos t = 1 √ 2 ⇔ x = 1 1 √ 2 . V y phương trình có nghi m duy nhat x = √ 2 . Bài toán 4.11 (USAMO - 1978). Giả sả x là nghi m của phương trình (3 + 2 √ 2)x = ( √ 2 − 1)x + 3. Cháng minh rang khi đó x cũng là nghi m của phương trình ( √ 2 + 1)x = π 2 cos . 9 Lài giai. Giả sả x là nghi m của phương trình (3 + 2 √ 2)x = ( √ 2 − 1)x + 3 ⇔ ( √ 2 + 1)2x = Đ t 2t = ( √ 2 + 1)x > 0. Khi đó (1) trở thành 1 ( √ 2 + 1)x + 3. (1) 4t2 = 1 2t + 3 ⇔ 4t3 − 3t = 1 . (2) ⇔
  • 62. 60 Tải tài liệu tại sividoc.com 2 2 9 Trược het ta tìm các nghi m t ∈ (−1; 1) của phương trình (1). Do t ∈ (−1; 1) nên đ t t = cos α, α ∈ (0; π). Ta có 4 cos3 α − 3 cos α = 1 1 ⇔ cos 3α = π 2π ⇔ α = ± + k . M t khác α ∈ (0; π) nên α ∈ π ; 5π ; 7π . 9 9 9 π 5π cos Suy ra các nghi m của phương trình (2) là t1 = cos 9 ; t2 = cos 7π . 9 9 ; t3 = π Ta thay (2) là phương trình b c 3 đã có đủ 3 nghi m t1 = cos 9 ; t2 = cos 5π 9 ; t3 = cos 7π 9 trong (−1; 1) nên ta không can xét các nghi m ở ngoài khoảng (−1; 1) nǎa. Do t2 = cos 5π 9 ; t3 = cos 7π không thỏa mãn đieu kiên 9 t > 0 nên chỉ có t1 √ thỏa mãn nên ( √ 2 + 1)x = 2cos π 9 hay x là nghi m của phương trình ( 2 + 1)x = 2cos π 9 (đpcm). Chú ý 4.1. Ngoài ra ta cũng có the sả dụng công thác nhân đôi, nhân ba xây dựng m®t so h phương trình mà ta dùng phép the lương giác đe giải. 3
  • 63. 61 Tải tài liệu tại sividoc.com Ket lu n Lu n văn “Phương pháp lượng giác giải phương trình đa thác và m®t so dạng toán liên quan” giải quyet nhǎng van đe sau: - Lu n văn trình bày m®t so kien thác cơ bản liên quan đen đȁng thác lượng giác và các đȁng thác đại so liên quan... - Tiep theo, lu n văn cũng đã trình bày chi tiet ve cách giải phương trình b c ba, b c bon và m®t so dạng phương trình đa thác b c cao... - Cuoi cùng, lu n văn trình bày các dạng toán liên quan các phương trình và h phương trình đưa ve giải phương trình đa thác b c cao. - M®t phan ket quả của lu n văn này đã được trình bày trong K yeu h®i thảo khoa hoc tại Hưng Yên (xem [2]).
  • 64. 62 Tải tài liệu tại sividoc.com Tài li u tham khảo A Tieng Vi t [1] Trương Ngoc Đac (2015), M®t so dãy so sinh bới các hàm lượng giác, K yeu HTKH Buôn Ma Thu®c, 14-15/03/2015, trang 58-64. [2] Mông Thanh Hang (2017), Phương pháp lương giác giải phương trình đa thúc b¾c cao, K yeu HTKH Hưng Yên, 25-26/02/2017, trang 117- 130. [3] Nguyen Văn M u, Phạm Thị Bạch Ngoc (2003), M®t so bài toán chon loc ve lượng giác, NXB Giáo dục. [4] Nguyen Văn M u, Đàm Văn Nhỉ (2012), Phương pháp toa đ® trong hình hoc, NXB ĐHQG Hà N®i. [5] Vũ Dương Thụy, Nguyen Văn Nho (2001), 40 năm Olympic Toán hoc quoc te, NXB Giáo dục. B Tieng Anh [6] Radulescu T-L.T. , Radulescu V.D. , Andreescu T. (2009), Problems in Real Analysis: Advanced Calculus on the real axis, Springer Sci- ences+Business Media. [7] Sausa Paulo Ney, Silva Jorge- Nume (1998), Berkeley Problems in Mathematics, Springer.